gleim 2015 | part 1 | online mcqs | unit 001 · pdf fileb. statement of changes in equity. c....

170
Gleim 2015 | Part 1 | Online MCQs | Unit 001 www.facebook.com/CMA.Arabwebsoft Page 1 دى عرب ويب سوفت حصري | لمنتQuestion: 2 Notes to financial statements are beneficial in meeting the disclosure requirements of financial reporting. The notes should not be used to A. Describe significant accounting policies. B. Describe depreciation methods employed by the company. C. Describe principles and methods peculiar to the industry in which the company operates, when these principles and methods are predominantly followed in that industry. D. Correct an improper presentation in the financial statements. Answer (D) is correct. Financial statement notes should not be used to correct improper presentations. The financial statements should be presented correctly on their own. Notes should be used to explain the methods used to prepare the financial statements and the amounts shown. The first footnote typically describes significant accounting policies. Question: 3 An objective of financial reporting is A. Providing information useful to investors, creditors, donors, and other users for decision making. A primary objective of external financial reporting is A. Direct measurement of the value of a business enterprise. B. Provision of information that is useful to present and potential investors, creditors, and others in making rational financial decisions regarding the enterprise. Answer (B) is correct. According to the FASB’s Conceptual Framework, the objectives of external financial reporting are to provide information that (1) is useful to present and potential investors, creditors, and others in making rational financial decisions regarding the enterprise; (2) helps those parties in assessing the amounts, timing, and uncertainty of prospective cash receipts from dividends or interest and the proceeds from sale, redemption, or maturity of securities or loans; and (3) concerns the economic resources of an enterprise, the claims thereto, and the effects of transactions, events, and circumstances that change its resources and claims thereto. C. Establishment of rules for accruing liabilities. D. Direct measurement of the enterprise’s stock price.

Upload: hanhu

Post on 05-Mar-2018

219 views

Category:

Documents


1 download

TRANSCRIPT

Gleim 2015 | Part 1 | Online MCQs | Unit 001

www.facebook.com/CMA.Arabwebsoft Page 1

حصري | لمنتدى عرب ويب سوفت

Question: 2 Notes to financial statements are beneficial in meeting the disclosure requirements of financial reporting. The notes should not be used to

A. Describe significant accounting policies.

B. Describe depreciation methods employed by the company.

C. Describe principles and methods peculiar to the industry in which the

company operates, when these principles and methods are predominantly

followed in that industry.

D. Correct an improper presentation in the financial statements.

Answer (D) is correct. Financial statement notes should not be used to correct improper

presentations. The financial statements should be presented correctly on

their own. Notes should be used to explain the methods used to prepare

the financial statements and the amounts shown. The first footnote

typically describes significant accounting policies.

Question: 3 An objective of financial reporting is

A. Providing information useful to investors, creditors, donors, and other users

for decision making.

A primary objective of external financial reporting is

A. Direct measurement of the value of a business enterprise.

B. Provision of information that is useful to present and potential investors,

creditors, and others in making rational financial decisions regarding the

enterprise.

Answer (B) is correct. According to the FASB’s Conceptual Framework, the objectives of

external financial reporting are to provide information that (1) is useful to

present and potential investors, creditors, and others in making rational

financial decisions regarding the enterprise; (2) helps those parties in

assessing the amounts, timing, and uncertainty of prospective cash

receipts from dividends or interest and the proceeds from sale,

redemption, or maturity of securities or loans; and (3) concerns the

economic resources of an enterprise, the claims thereto, and the effects of

transactions, events, and circumstances that change its resources and

claims thereto.

C. Establishment of rules for accruing liabilities.

D. Direct measurement of the enterprise’s stock price.

Gleim 2015 | Part 1 | Online MCQs | Unit 001

www.facebook.com/CMA.Arabwebsoft Page 2

حصري | لمنتدى عرب ويب سوفت

Answer (A) is correct. The objective is to report financial information that is useful in making

decisions about providing resources to the reporting entity. Primary users

of financial information are current or prospective investors and creditors

who cannot obtain it directly. Their decisions depend on expected

returns.

B. Assessing the adequacy of internal control.

C. Evaluating management results compared with standards.

D. Providing information on compliance with established procedures.

Question: 4 The management of ABC Corporation is analyzing the financial statements of XYZ Corporation because ABC is strongly considering purchasing a block of XYZ ordinary shares that would give ABC significant influence over XYZ. Which financial statement should ABC primarily use to assess the amounts, timing, and certainty of future cash flows of XYZ Company?

A. Income statement.

B. Statement of changes in equity.

C. Statement of cash flows.

Answer (C) is correct. A statement of cash flows provides information about the cash receipts

and cash payments of an entity during a period. This information helps

investors, creditors, and other users to assess the entity’s ability to

generate cash and cash equivalents and the needs of the entity to use

those cash flows. Historical cash flow data indicate the amount, timing,

and certainty of future cash flows. It is also a means of verifying past

cash flow assessments and of determining the relationship between

profits and net cash flows and the effects of changing prices.

D. Statement of financial position.

Question: 5 An entity that sprays chemicals in residences to eliminate or prevent infestation of insects requires that customers prepay for 3 months’ service at the beginning of each new quarter. Select the term that appropriately describes this situation from the viewpoint of the entity.

A. Deferred income.

Answer (A) is correct. The future inflow of economic benefits is not sufficiently certain given

that the entity has not done what is required to be entitled to those

benefits. Thus, the receipt of cash in anticipation of goods to be delivered

or services to be performed must be recognized as a liability, usually

called deferred (or unearned) revenue or deferred (or unearned) income.

B. Earned income.

C. Accrued income.

D. Prepaid expense.

Gleim 2015 | Part 1 | Online MCQs | Unit 001

www.facebook.com/CMA.Arabwebsoft Page 3

حصري | لمنتدى عرب ويب سوفت

Question: 6 Which of the following is true regarding the comparison of managerial and financial accounting?

A. Managerial accounting is generally more precise.

B. Managerial accounting has a past focus, and financial accounting has a

future focus.

C. The emphasis on managerial accounting is relevance, and the emphasis on

financial accounting is timeliness.

D. Managerial accounting need not follow generally accepted accounting

principles (GAAP), while financial accounting must follow them.

Answer (D) is correct. Managerial accounting assists management decision making, planning,

and control. Financial accounting addresses accounting for an entity’s

assets, liabilities, revenues, expenses, and other elements of financial

statements. Financial statements are the primary method of

communicating to external parties information about the entity’s results

of operations, financial position, and cash flows. For general-purpose

financial statements to be useful to external parties, they must be

prepared in conformity with accounting principles that are generally

accepted in the United States. However, managerial accounting

information is primarily directed to specific internal users. Hence, it

ordinarily need not follow such guidance.

Question: 7 The financial statements included in the annual report to the shareholders are least useful to which one of the following?

A. Stockbrokers.

B. Bankers preparing to lend money.

C. Competing businesses.

D. Managers in charge of operating activities.

Answer (D) is correct. Accrual-basis amounts used in financial reporting are not useful to

managers making day-to-day operating decisions. The practice of

management accounting fulfills the needs of these users.

Question: 8 The accounting measurement that is not consistent with the going concern concept is

A. Historical cost.

B. Realization.

C. The transaction approach.

D. Liquidation value.

Answer (D) is correct. Financial accounting principles assume that a business entity is a going

concern in the absence of evidence to the contrary. The concept justifies

the use of depreciation and amortization schedules, and the recording of

Gleim 2015 | Part 1 | Online MCQs | Unit 001

www.facebook.com/CMA.Arabwebsoft Page 4

حصري | لمنتدى عرب ويب سوفت

assets and liabilities using attributes other than liquidation value.

Question: 9 The primary purpose of the statement of financial position is to reflect

A. The fair value of the firm’s assets at some moment in time.

B. The status of the firm’s assets in case of forced liquidation of the firm.

C. The success of a company’s operations for a given amount of time.

D. Items of value, debt, and net worth.

Answer (D) is correct. The balance sheet presents three major financial accounting elements:

assets (items of value), liabilities (debts), and equity (net worth).

According to the FASB’s Conceptual Framework, assets are probable

future economic benefits resulting from past transactions or events.

Liabilities are probable future sacrifices of economic benefits arising

from present obligations as a result of past transactions or events. Equity

is the residual interest in the assets after deduction of liabilities.

Question: 10 Prepaid expenses are valued on the statement of financial position at the

A. Cost to acquire the asset.

B. Face amount collectible at maturity.

C. Cost to acquire minus accumulated amortization.

D. Cost less expired or used portion.

Answer (D) is correct. Prepaid expenses, such as supplies, prepaid rent, and prepaid insurance,

are reported on the balance sheet at cost minus the expired or used

portion. These are typically current assets.

Question: 11 A statement of financial position allows investors to assess all of the following except the

A. Efficiency with which enterprise assets are used.

B. Liquidity and financial flexibility of the enterprise.

C. Capital structure of the enterprise.

D. Net realizable value of enterprise assets.

Answer (D) is correct. Assets are usually measured at original historical cost in a statement of

financial position, although some exceptions exist. For example, some

short-term receivables are reported at their net realizable value. Thus, the

statement of financial position cannot be relied upon to assess NRV.

Gleim 2015 | Part 1 | Online MCQs | Unit 001

www.facebook.com/CMA.Arabwebsoft Page 5

حصري | لمنتدى عرب ويب سوفت

Question: 12 The accounting equation (assets – liabilities = equity) reflects the

A. Entity point of view.

B. Fund theory.

C. Proprietary point of view.

Answer (C) is correct. The equation is based on the proprietary theory. Equity in an enterprise is

what remains after the economic obligations of the enterprise are

deducted from its economic resources.

D. Enterprise theory.

Question: 13 Long-term obligations that are or will become callable by the creditor because of the debtor’s violation of a provision of the debt agreement at the balance sheet date should be classified as

A. Long-term liabilities.

B. Current liabilities unless the debtor goes bankrupt.

C. Current liabilities unless the creditor has waived the right to demand

repayment for more than 1 year from the balance sheet date.

Answer (C) is correct. Long-term obligations that are or will become callable by the creditor

because of the debtor’s violation of a provision of the debt agreement at

the balance sheet date normally are classified as current liabilities.

However, the debt need not be reclassified if the violation will be cured

within a specified grace period or if the creditor formally waives or

subsequently loses the right to demand repayment for a period of more

than a year from the balance sheet date (also, reclassification is not

required if the debtor expects and has the ability to refinance the

obligation on a long-term basis).

D. Contingent liabilities until the violation is corrected.

Question: 14 When classifying assets as current and noncurrent for reporting purposes,

A. The amounts at which current assets are carried and reported must reflect

realizable cash values.

B. Prepayments for items such as insurance or rent are included in an “other

assets” group rather than as current assets as they will ultimately be

expensed.

C. The time period by which current assets are distinguished from noncurrent

assets is determined by the seasonal nature of the business.

D. Assets are classified as current if they are reasonably expected to be

realized in cash or consumed during the normal operating cycle.

Answer (D) is correct.

Gleim 2015 | Part 1 | Online MCQs | Unit 001

www.facebook.com/CMA.Arabwebsoft Page 6

حصري | لمنتدى عرب ويب سوفت

For financial reporting purposes, current assets consist of cash and other

assets or resources expected to be realized in cash, sold, or consumed

during the longer of 1 year or the normal operating cycle of the business.

Question: 15 Abernathy Corporation uses a calendar year for financial and tax reporting purposes and has $100 million of mortgage bonds due on January 15, Year 2. By January 10, Year 2, Abernathy intends to refinance this debt with new long-term mortgage bonds and has entered into a financing agreement that clearly demonstrates its ability to consummate the refinancing. This debt is to be

A. Classified as a current liability on the statement of financial position at

December 31, Year 1.

B. Classified as a long-term liability on the statement of financial position at

December 31, Year 1.

Answer (B) is correct. Short-term obligations expected to be refinanced should be reported as

current liabilities unless the firm both plans to refinance and has the

ability to refinance the debt on a long-term basis. The ability to refinance

on a long-term basis is evidenced by a post-balance-sheet date issuance

of long-term debt or a financing arrangement that will clearly permit

long-term refinancing.

C. Retired as of December 31, Year 1.

D. Considered off-balance-sheet debt.

Question: 16 Lister Company intends to refinance a portion of its short-term debt in Year 2 and is negotiating a long-term financing agreement with a local bank. This agreement would be noncancelable and would extend for a period of 2 years. The amount of short-term debt that Lister Company can exclude from its statement of financial position at December 31, Year 1,

A. May exceed the amount available for refinancing under the agreement.

B. Depends on the demonstrated ability to consummate the refinancing.

Answer (B) is correct. If an enterprise intends to refinance short-term obligations on a long-term

basis and demonstrates an ability to consummate the refinancing, the

obligations should be excluded from current liabilities and classified as

noncurrent. The ability to consummate the refinancing may be

demonstrated by a post-balance-sheet-date issuance of a long-term

obligation or equity securities, or by entering into a financing agreement

that meets certain criteria. These criteria are that the agreement does not

expire within 1 year, it is noncancelable by the lender, no violation of the

agreement exists at the balance sheet date, and the lender is financially

capable of honoring the agreement.

C. Is reduced by the proportionate change in the working capital ratio.

D. Is zero unless the refinancing has occurred by year end.

Gleim 2015 | Part 1 | Online MCQs | Unit 001

www.facebook.com/CMA.Arabwebsoft Page 7

حصري | لمنتدى عرب ويب سوفت

Question: 17 A statement of financial position is intended to help investors and creditors

A. Assess the amount, timing, and uncertainty of prospective net cash inflows

of a firm.

B. Evaluate economic resources and obligations of a firm.

Answer (B) is correct. The statement of financial position, or balance sheet, provides

information about an entity’s resource structure (assets) and financing

structure (liabilities and equity) at a moment in time. According to the

FASB’s Conceptual Framework, the statement of financial position does

not purport to show the value of a business, but it enables investors,

creditors, and other users to make their own estimates of value. It helps

users to assess liquidity, financial flexibility, profitability, and risk.

C. Evaluate economic performance of a firm.

D. Evaluate changes in the ownership equity of a firm.

Question: 18 A manufacturer receives an advance payment for special-order goods that are to be manufactured and delivered within the next year. The advance payment should be reported in the manufacturer’s current-year statement of financial position as a(n)

A. Current liability.

Answer (A) is correct. The entity has not substantially completed what it must do to be entitled

to the benefits of the advance payment, and the receipt of future

economic benefits is not sufficiently certain to justify income

recognition. Accordingly, the receipt of cash in anticipation of goods to

be delivered or services to be performed must be recognized as a

liability, usually called deferred (or unearned) revenue or deferred (or

unearned) income. Because the manufacturer must deliver the goods

within the next year, this liability is current.

B. Noncurrent liability.

C. Contra asset amount.

D. Accrued revenue.

Question: 19 A cable television entity receives deposits from customers that are refunded when service is terminated. The average customer stays with the entity 8 years. How should these deposits be shown on the financial statements?

A. Operating revenue.

B. Other revenue.

C. Paid-in capital.

D. Liability.

Answer (D) is correct. Liabilities are present obligations arising from past events, the settlement

of which is expected to result in an outflow of resources embodying

economic benefits. Customers’ deposits must be returned or credited to

Gleim 2015 | Part 1 | Online MCQs | Unit 001

www.facebook.com/CMA.Arabwebsoft Page 8

حصري | لمنتدى عرب ويب سوفت

their accounts. The deposits should therefore be recorded as liabilities.

Question: 20 A company has outstanding accounts payable of $30,000 and a short-term construction loan in the amount of $100,000 at year end. The loan was refinanced through issuance of long-term bonds after year end but before issuance of financial statements. How should these liabilities be recorded in the balance sheet?

A. Noncurrent liabilities of $130,000.

B. Current liabilities of $130,000.

C. Current liabilities of $30,000, noncurrent liabilities of $100,000.

Answer (C) is correct. Accounts payable are properly classified as current liabilities because

they are for items entering into the operating cycle. Short-term debt that

is refinanced by a post-balance-sheet-date issuance of long-term debt

should be classified as noncurrent. (The ability to refinance on a long-

term basis has been demonstrated.) Thus, the short-term construction

loan is classified as noncurrent. Accordingly, the entity records current

liabilities of $30,000 and noncurrent liabilities of $100,000.

D. Current liabilities of $130,000, with required footnote disclosure of the

refinancing of the loan.

Question: 21 A statement of financial position provides a basis for all of the following except

A. Computing rates of return.

B. Evaluating capital structure.

C. Assessing liquidity and financial flexibility.

D. Determining profitability and assessing past performance.

Answer (D) is correct. The statement of financial position, also known as the balance sheet,

reports an entity’s financial position at a moment in time. It is therefore

not useful for assessing past performance for a period of time. A balance

sheet can be used to help users assess liquidity, financial flexibility, and

risk.

Question: 22 Noncurrent debt should be included in the current section of the statement of financial position if

A. It is to be converted into common stock before maturity.

B. It matures within the year and will be retired through the use of current

assets.

Answer (B) is correct. Current liabilities include those obligations that are expected to be

satisfied by the (1) payment of cash, (2) use of current assets other than

cash, or (3) creation of new current liabilities within 1 year from the

Gleim 2015 | Part 1 | Online MCQs | Unit 001

www.facebook.com/CMA.Arabwebsoft Page 9

حصري | لمنتدى عرب ويب سوفت

balance sheet date (or operating cycle, if longer).

C. Management plans to refinance it within the year.

D. A bond retirement fund has been set up for use in its scheduled retirement

during the next year.

Question: 23 Dixon Company has the following items recorded on its financial records:

Available-for-sale securities $200,000

Prepaid expenses 400,000

Treasury stock 100,000

The total amount of the above items to be shown as assets on Dixon’s statement of financial position is

A. $400,000

B. $500,000

C. $600,000

Answer (C) is correct. Available-for-sale securities (an investment) and prepaid expenses are

assets, but treasury stock is an equity item. The total of the assets

reported is therefore $600,000 ($200,000 + $400,000).

D. $700,000

Question: 24 A receivable classified as current on the statement of financial position is expected to be collected within

A. The current operating cycle.

B. 1 year.

C. The current operating cycle or 1 year, whichever is longer.

Answer (C) is correct. Current assets are reasonably expected to be realized in cash, sold, or

consumed during the normal operating cycle of the business or within 1

year, whichever is longer. The operating cycle is the time between the

acquisition of materials or services and the final cash realization from the

earning process.

D. The current operating cycle or 1 year, whichever is shorter.

Question: 25A company pays more than the fair value to acquire treasury stock. The difference

between the price paid to acquire the treasury stock and the fair value should be recorded as

A. An asset.

B. A liability.

C. Shareholders’ equity.

Gleim 2015 | Part 1 | Online MCQs | Unit 001

www.facebook.com/CMA.Arabwebsoft Page 11

حصري | لمنتدى عرب ويب سوفت

Answer (C) is correct. Apart from cash paid or received, a firm cannot recognize assets, liabilities, gains, or

losses from transactions in its own stock. Treasury stock is reported on the balance sheet

as a subtraction from equity.

D. An expense.

Question: 26 The purchase of treasury stock is recorded on the statement of financial position as a(n)

A. Increase in assets.

B. Decrease in liabilities.

C. Increase in shareholders’ equity.

D. Decrease in shareholders’ equity.

Answer (D) is correct. The purchase of treasury stock is recorded on the statement of financial

position as a decrease in shareholders’ equity.

Question: 27 Current assets are reasonably expected to be realized in cash or sold or consumed during the normal operating cycle of the business. Current assets most likely include

A. Intangible assets.

B. Purchased goodwill.

C. Organizational costs.

D. Trading securities.

Answer (D) is correct. Current assets include, in descending order of liquidity, cash and cash

equivalents; certain individual trading, available-for-sale, and held-to-

maturity securities; receivables; inventories; and prepaid expenses.

Trading securities are expected to be sold in the near term, so they are

likely to be classified as current.

Question: 28 Rice Co. was incorporated on January 1, Year 6, with $500,000 from the issuance of stock and borrowed funds of $75,000. During the first year of operations, net income was $25,000. On December 15, Rice paid a $2,000 cash dividend. No additional activities affected equity in Year 6. At December 31, Year 6, Rice’s liabilities had increased to $94,000. In Rice’s December 31, Year 6 balance sheet, total assets should be reported at

A. $598,000

B. $600,000

C. $617,000

Answer (C) is correct. Total assets equal the sum of total liabilities and equity. Total liabilities

were $94,000 at year end, and equity amounted to $523,000 ($500,000

from issuance of stock + $25,000 net income – $2,000 cash dividend).

Total assets are therefore $617,000 ($523,000 + $94,000).

Gleim 2015 | Part 1 | Online MCQs | Unit 001

www.facebook.com/CMA.Arabwebsoft Page 11

حصري | لمنتدى عرب ويب سوفت

D. $692,000

Question: 29 Careful reading of an annual report will reveal that off-balance-sheet debt includes

A. Amounts due in future years under operating leases.

Answer (A) is correct. Off-balance-sheet debt includes any type of liability for which the

company is responsible but that does not appear on the balance sheet.

The most common example is the amount due in future years on

operating leases. Under U.S. GAAP, operating leases are not capitalized;

instead, only the periodic payments of rent are reported when actually

paid. Capital leases (those similar to a purchase) must be capitalized and

reported as liabilities.

B. Transfers of accounts receivable without recourse.

C. Current portion of long-term debt.

D. Amounts due in future years under capital leases.

Question: 30 Which one of the following is not a form of off-balance-sheet financing?

A. Sale of receivables.

B. Foreign currency translations.

Answer (B) is correct. Off-balance-sheet financing takes four principal forms: investments in

unconsolidated subsidiaries, special purpose entities, operating leases,

and factoring receivables with recourse.

C. Operating leases.

D. Special purpose entities.

Question: 31 In a multiple-step income statement for a retail company, all of the following are included in the operating section except

A. Sales.

B. Cost of goods sold.

C. Dividend revenue.

Answer (C) is correct. The operating section of a retailer’s income statement includes all

revenues and costs necessary for the operation of the retail establishment,

e.g., sales, cost of goods sold, administrative expenses, and selling

expenses. Dividend revenue, however, is classified under other revenues.

In a statement of cash flows, cash dividends received are considered an

operating cash flow.

D. Administrative and selling expenses.

Gleim 2015 | Part 1 | Online MCQs | Unit 001

www.facebook.com/CMA.Arabwebsoft Page 12

حصري | لمنتدى عرب ويب سوفت

Question: 32 When reporting extraordinary items,

A. Each item (net of tax) is presented on the face of the income statement

separately as a component of net income for the period.

Answer (A) is correct. Extraordinary items are reported net of tax after discontinued operations.

B. Each item is presented exclusive of any related income tax.

C. Each item is presented as an unusual item within income from continuing

operations.

D. All extraordinary gains or losses that occur in a period are summarized as

total gains and total losses, then offset to present the net extraordinary gain

or loss.

Question: 33 Which one of the following items is included in the determination of income from continuing operations?

A. Discontinued operations.

B. Extraordinary loss.

C. Cumulative effect of a change in an accounting principle.

D. Unusual loss from a write-down of inventory.

Answer (D) is correct. Certain items ordinarily are not to be treated as extraordinary gains and

losses. Rather, they are included in the determination of income from

continuing operations. These gains and losses include those from write-

downs of receivables and inventories, translation of foreign currency

amounts, disposal of a business segment, sale of productive assets,

strikes, and accruals on long-term contracts. A write-down of inventory

is therefore included in the computation of income from continuing

operations.

Question: 34 Which one of the following would be shown on a multiple-step income statement but not on a single-step income statement?

A. Loss from discontinued operations.

B. Gross profit.

Answer (B) is correct. A single-step income statement combines all revenues and gains,

combines all expenses and losses, and subtracts the latter from the former

in a “single step” to arrive at net income. Gross profit, being the

difference between sales revenue and cost of goods sold, does not appear

on a single-step income statement.

C. Extraordinary gain.

D. Net income from continuing operations.

Gleim 2015 | Part 1 | Online MCQs | Unit 001

www.facebook.com/CMA.Arabwebsoft Page 13

حصري | لمنتدى عرب ويب سوفت

Question: 35 The major segments of the statement of retained earnings for a period are

A. Dividends declared, prior period adjustments, and changes due to treasury

stock transactions.

B. Before-tax income or loss and dividends paid or declared.

C. Prior-period adjustments, before-tax income or loss, income tax, and

dividends paid.

D. Net income or loss, prior-period adjustments, and dividends paid or

declared.

Answer (D) is correct. The statement of retained earnings is a basic financial statement.

Together with the income statement, the statement of retained earnings is

meant to broadly reflect the results of operations. The statement of

retained earnings consists of beginning retained earnings adjusted for any

prior period adjustment (net of tax), with further adjustments for income

(loss), dividends, and in certain other rare adjustments, e.g., quasi-

reorganizations. The final figure is ending retained earnings.

Question: 36 Because of inexact estimates of the service life and the residual value of a plant asset, a fully depreciated asset was sold in the current year at a material gain. This gain most likely should be reported

A. In the other revenues and gains section of the current income statement.

Answer (A) is correct. Revenues occur in the course of ordinary activities. Gains may or may

not occur in the course of ordinary activities. For example, gains may

occur from the sale of noncurrent assets. Thus, the gain on the sale of a

plant asset is not an operating item and should be classified in an income

statement with separate operating and nonoperating sections in the other

revenues and gains section.

B. As part of sales revenue on the current income statement.

C. In the extraordinary item section of the current income statement.

D. As an adjustment to prior periods’ depreciation on the statement of changes

in equity.

Question: 37 In recording transactions, which of the following best describes the relation between expenses and losses?

A. Losses are extraordinary charges to income, whereas expenses are ordinary

charges to income.

B. Losses are material items, whereas expenses are immaterial items.

C. Losses are expenses that may or may not arise in the course of ordinary

activities.

Answer (C) is correct.

Gleim 2015 | Part 1 | Online MCQs | Unit 001

www.facebook.com/CMA.Arabwebsoft Page 14

حصري | لمنتدى عرب ويب سوفت

Expenses are outflow or other usage of assets or incurrences of liability

(or both) from activities that qualify as ongoing major or central

operations. Losses are similar to expenses but generally do not occur in

ordinary activities. For example, losses may result from the sale of

noncurrent assets or from natural disasters.

D. Expenses can always be prevented, whereas losses can never be prevented.

Question: 38 An entity has a 50% gross margin, general and administrative expenses of $50, interest expense of $20, and net income of $10 for the year just ended. If the corporate tax rate is 50%, the level of sales revenue for the year just ended was

A. $90

B. $135

C. $150

D. $180

Answer (D) is correct. Net income equals sales minus cost of sales, G&A expenses, interest, and

tax. Given a 50% tax rate, income before tax must have been $20 [$10

net income ÷ (1.0 – 0.5 tax rate)]. Accordingly, income before interest

and tax must have been $40 ($20 income before tax + $20 interest), and

the gross margin (sales – cost of sales) must have been $90 ($40 income

before interest and tax + $50 G&A expenses). If the gross margin is 50%

of sales, sales equals $180 ($90 gross margin ÷ 0.5).

Question: 39Assume that employees confessed to a $500,000 inventory theft but are not able to make

restitution. How should this material fraud be shown in the company’s financial statements?

A. Classified as a loss and shown as a separate line item in the income statement.

Answer (A) is correct. Losses may or may not occur in the course of ordinary activities. For example, they may

result from nonreciprocal transactions (e.g., theft), reciprocal transactions (e.g., a sale of

plant assets), or from holding assets or liabilities. Losses are typically displayed separately.

B. Initially classified as an accounts receivable because the employees are responsible for the

goods. Because they cannot pay, the loss would be recognized as a write-off of accounts

receivable.

C. Included in cost of goods sold because the goods are not on hand, losses on inventory

shrinkage are ordinary, and it would cause the least amount of attention.

D. Recorded directly to retained earnings because it is not an income-producing item.

Gleim 2015 | Part 1 | Online MCQs | Unit 001

www.facebook.com/CMA.Arabwebsoft Page 15

حصري | لمنتدى عرب ويب سوفت

Question: 40 An entity had the following opening and closing inventory balances during the current year:

1/1 12/31

Finished goods $ 90,000 $260,000

Raw materials 105,000 130,000

Work-in-progress 220,000 175,000

The following transactions and events occurred during the current year:

$300,000 of raw materials were purchased, of which $20,000 were

returned because of defects.

$600,000 of direct labor costs were incurred.

$750,000 of production overhead costs were incurred.

The cost of goods sold for the current year ended December 31 would be

A. $1,480,000

Answer (A) is correct. Cost of goods sold equals cost of goods manufactured (COGM) adjusted

for the change in finished goods. COGM equals the sum of raw materials

used, direct labor costs, and production overhead, adjusted for the change

in work-in-progress. Raw materials used equals $255,000 ($105,000 BI +

$300,000 purchases – $20,000 returns – $130,000 EI). Thus, COGM

equals $1,650,000 ($255,000 RM + $600,000 DL + $750,000 OH +

$220,000 BWIP – $175,000 EWIP), and COGS equals $1,480,000

($1,650,000 COGM + $90,000 BFG – $260,000 EFG).

B. $1,500,000

C. $1,610,000

D. $1,650,000

Question: 41 The profit and loss statement of Madengrad Mining includes the following information for the current fiscal year:

Sales $160,000

Gross profit 48,000

Year-end finished goods inventory 58,300

Opening finished goods inventory 60,190

The cost of goods manufactured by Madengrad for the current fiscal year is

A. $46,110

B. $49,890

C. $110,110

Answer (C) is correct. Madengrad’s cost of goods manufactured can be calculated as follows:

Gleim 2015 | Part 1 | Online MCQs | Unit 001

www.facebook.com/CMA.Arabwebsoft Page 16

حصري | لمنتدى عرب ويب سوفت

Sales $160,000

Less: gross profit (48,000)

Cost of goods sold $112,000

Add: ending finished goods 58,300

Goods available for sale $170,300

Less: beginning finished goods (60,190)

Cost of goods manufactured $110,110

D. $113,890

Question: 42 If the beginning balance for May of the materials inventory account was $27,500, the ending balance for May is $28,750, and $128,900 of materials were used during the month, the materials purchased during the month cost

A. $101,400

B. $127,650

C. $130,150

Answer (C) is correct. Purchases equals usage adjusted for the inventory change. Hence,

purchases equals $130,150 ($128,900 used – $27,500 BI + $28,750 EI).

D. $157,650

Question: 43 Given the following data for Scurry Company, what is the cost of goods sold?

Beginning inventory of finished goods $100,000

Cost of goods manufactured 700,000

Ending inventory of finished goods 200,000

Beginning work-in-process inventory 300,000

Ending work-in-process inventory 50,000

A. $500,000

B. $600,000

Answer (B) is correct. Scurry’s cost of goods sold can be calculated as follows:

Beginning inventory of finished goods $ 100,000

Gleim 2015 | Part 1 | Online MCQs | Unit 001

www.facebook.com/CMA.Arabwebsoft Page 17

حصري | لمنتدى عرب ويب سوفت

Add: cost of goods manufactured 700,000

Less: ending inventory of finished goods (200,000)

Cost of goods sold $ 600,000

C. $800,000

D. $950,000

Question: 44The following information was taken from last year’s accounting records of a

manufacturing company.

Inventory January 1 December 31

Raw materials $38,000 $ 45,000

Work-in-process 21,000 10,000

Finished goods 78,000 107,000

Other information

Direct labor $236,000

Shipping costs on outgoing orders 6,500

Factory rent 59,000

Factory depreciation 18,700

Advertising expense 24,900

Net purchases of raw materials 115,000

Corporate administrative salaries 178,000

Material handling costs 35,800

On the basis of this information, the company’s cost of goods manufactured and cost of goods sold

are

A. $460,500 and $489,500, respectively.

B. $468,500 and $439,500, respectively.

Answer (B) is correct. This solution requires a series of computations.

Beginning raw materials $ 38,000

Add: net purchases raw materials $115,000

Materials available $153,000

Less: ending materials (45,000)

Materials used in production $108,000

Gleim 2015 | Part 1 | Online MCQs | Unit 001

www.facebook.com/CMA.Arabwebsoft Page 18

حصري | لمنتدى عرب ويب سوفت

Direct labor 236,000

Manufacturing overhead

Factory rent $59,000

Factory depreciation 18,700

Material handling costs 35,800

Total Manufacturing overhead 113,500

Total manufacturing costs $457,500

Add: beginning work-in-process 21,000

Less: ending work-in-process (10,000)

Costs of Goods Manufactured $468,500

Add: beginning finished goods 78,000

Less: ending finished goods (107,000)

Cost of Goods Sold $439,500

C. $468,500 and $470,900, respectively.

D. $646,500 and $617,500, respectively.

Question: 45 Comprehensive income is best defined as

A. Net income excluding extraordinary gains and losses.

B. The change in net assets for the period including contributions by owners

and distributions to owners.

C. Total revenues minus total expenses.

D. The change in net assets for the period excluding owner transactions.

Answer (D) is correct. Comprehensive income includes all changes in equity of a business

entity except those changes resulting from investments by owners and

distributions to owners. Comprehensive income includes two major

categories: net income and other comprehensive income (OCI). Net

income includes the results of operations classified as income from

continuing operations, discontinued operations, and extraordinary items.

Components of comprehensive income not included in the determination

of net income are included in OCI, for example, unrealized gains and

losses on available-for-sale securities (except those that are hedged items

Gleim 2015 | Part 1 | Online MCQs | Unit 001

www.facebook.com/CMA.Arabwebsoft Page 19

حصري | لمنتدى عرب ويب سوفت

in a fair value hedge).

Question: 46 The financial statement that provides a summary of the firm’s operations for a period of time is the

A. Income statement.

Answer (A) is correct. The results of operations for a period of time are reported in the income

statement (statement of earnings) on the accrual basis using an approach

oriented to historical transactions.

B. Statement of financial position.

C. Statement of shareholders’ equity.

D. Statement of retained earnings.

Question: 47 The following information pertains to Maynard Corporation’s income statement for

the 12 months just ended. The company has an effective income tax rate of 40%.

Discontinued operations $(70,000)

Extraordinary loss due to earthquake (90,000)

Income from continuing operations (net of tax) 72,000

Cumulative effect of change in accounting principle 60,000

Maynard’s net income for the year is

A. $36,000

B. $12,000

C. $8,000

D. $(24,000)

Answer (D) is correct. Maynard’s net income for the year is calculated as follows:

Income Times:

Statement Tax As

Item Effect Reported

Income from continuing operations (net of

tax)

$ 72,000

Discontinued operations $(70,000) (1.0 –

.40)

(42,000)

Extraordinary loss due to earthquake (90,000) (1.0 – (54,000)

Gleim 2015 | Part 1 | Online MCQs | Unit 001

www.facebook.com/CMA.Arabwebsoft Page 21

حصري | لمنتدى عرب ويب سوفت

.40)

Net income $(24,000)

Question: 48 Which of the following items is not classified as other comprehensive income (OCI)?

A. Extraordinary gains from extinguishment of debt.

Answer (A) is correct. Comprehensive income is divided into net income and other

comprehensive income (OCI). Under existing accounting standards, OCI

includes (1) unrealized gains and losses on available-for-sale securities

(except those that are hedged items in a fair value hedge); (2) gains and

losses on derivatives designated, qualifying, and effective as cash flow

hedges; (3) certain amounts associated with recognition of the funded

status of postretirement defined benefit plans; and (4) certain foreign

currency items, including foreign currency translations.

B. Foreign currency translation adjustments.

C. Prior service cost adjustment resulting from amendment of a defined

benefit pension plan.

D. Unrealized gains for the year on available-for-sale marketable securities.

Question: 49 Which of the following are acceptable formats for reporting comprehensive income?

I. In one continuous financial statement

II. In a statement of changes in equity

III. In a separate statement of net income

IV. In two separate but consecutive financial statements

A. I and II only.

B. I, II, and III only.

C. III and IV only.

D. I and IV only.

Answer (D) is correct. If an entity that presents a full set of financial statements has items of

other comprehensive income (OCI), it must present comprehensive

income either (1) in a single continuous statement of comprehensive

income or (2) in two separate but consecutive statements (an income

statement and a statement of OCI).

Gleim 2015 | Part 1 | Online MCQs | Unit 001

www.facebook.com/CMA.Arabwebsoft Page 21

حصري | لمنتدى عرب ويب سوفت

Question: 50 A company reports the following information as of December 31:

Sales revenue $800,000

Cost of goods sold 600,000

Operating expenses 90,000

Unrealized holding gain on available-for-sale securities, net of tax 30,000

What amount should the company report as comprehensive income as of December 31?

A. $30,000

B. $110,000

C. $140,000

Answer (C) is correct. Comprehensive income includes net income and other comprehensive

income. Net income equals $110,000 ($800,000 sales revenue –

$600,000 COGS – $90,000 operating expenses). Unrealized holding

gains on available-for-sale securities ($30,000) are included in other

comprehensive income. Thus, comprehensive income is $140,000

($110,000 + $30,000).

D. $200,000

Question: 51 Crawford Company is researching a future change to IFRS. Which one of the following items reported on Crawford’s income statement under U.S. GAAP is required to be changed as a result of adopting IFRS?

A. Crawford values its merchandise inventory using average cost.

B. Crawford uses a multiple-step approach for its income statement.

C. Crawford uses historical cost to value its land, buildings, and intangible

assets even though the value of the land and building are greater than book

value.

D. Crawford’s current-year income statement includes an extraordinary loss.

Answer (D) is correct. Under U.S. GAAP, material transactions that are both unusual in nature

and infrequent in occurrence in the environment in which the company

operates are classified as extraordinary items. Extraordinary items are

reported individually in a separate section in the income statement, net of

tax. Under IFRS, no item is classified as extraordinary, and therefore it

would be recorded in the normal part of the income statement.

Question: 52 All of the following are defined as elements of an income statement except

A. Expenses.

B. Shareholders’ equity.

Answer (B) is correct.

Gleim 2015 | Part 1 | Online MCQs | Unit 001

www.facebook.com/CMA.Arabwebsoft Page 22

حصري | لمنتدى عرب ويب سوفت

Equity of a business entity (or the net assets of a nonbusiness

organization) is a residual amount that reflects the basic accounting

equation: assets minus liabilities equals equity (or net assets). It is

reported on the statement of financial position.

C. Gains and losses.

D. Revenues.

Question: 53 Items reported as prior-period adjustments

A. Do not include the effect of a mistake in the application of accounting

principles, as this is accounted for as a change in accounting principle

rather than as a prior-period adjustment.

B. Do not affect the presentation of prior-period comparative financial

statements.

C. Do not require further disclosure in the body of the financial statements.

D. Are reflected as adjustments of the opening balance of the retained earnings

of the earliest period presented.

Answer (D) is correct. Prior-period adjustments are made for the correction of errors. According

to SFAS 16, Prior Period Adjustments, the effects of errors on prior-

period financial statements are reported as adjustments to beginning

retained earnings for the earliest period presented in the retained earnings

statement. Such errors do not affect the income statement for the current

period.

Question: 54 An appropriation of retained earnings by the board of directors of a corporation for bonded indebtedness will result in

A. The establishment of a sinking fund to retire bonds when they mature.

B. A decrease in cash on the balance sheet with an equal increase in the

investment and funds section of the balance sheet.

C. A decrease in the total amount of retained earnings presented on the

balance sheet.

D. The disclosure that management does not intend to distribute assets, in the

form of dividends, equal to the amount of the appropriation.

Answer (D) is correct. The appropriation of retained earnings is a transfer from one retained

earnings account to another. The only practical effect is to decrease the

amount of retained earnings available for dividends. An appropriation of

retained earnings is purely for disclosure purposes.

Gleim 2015 | Part 1 | Online MCQs | Unit 001

www.facebook.com/CMA.Arabwebsoft Page 23

حصري | لمنتدى عرب ويب سوفت

Question: 55 When treasury stock is accounted for at cost, the cost is reported on the balance sheet as a(n)

A. Asset.

B. Reduction of retained earnings.

C. Reduction of additional paid-in-capital.

D. Unallocated reduction of equity.

Answer (D) is correct. Treasury stock is a corporation’s own stock that has been reacquired but

not retired. In the balance sheet, treasury stock recorded at cost is

subtracted from the total of the capital stock balances, additional paid-in

capital, retained earnings, and accumulated other comprehensive income.

Question: 56 The statement of shareholders’ equity shows a

A. Reconciliation of the beginning and ending balances in shareholders’

equity accounts.

Answer (A) is correct. The statement of shareholders’ equity (changes in equity) presents a

reconciliation in columnar format of the beginning and ending balances

in the various shareholders’ equity accounts. A statement of changes in

equity may include, for example, columns for (1) totals,

(2) comprehensive income, (3) retained earnings, (4) accumulated OCI

(but the components of OCI are presented in another statement), (5)

common stock, and (6) additional paid-in capital.

B. Listing of all shareholders’ equity accounts and their corresponding dollar

amounts.

C. Computation of the number of shares outstanding used for earnings per

share calculations.

D. Reconciliation of net income to net operating cash flow.

Question: 57 Unless the shares are specifically restricted, a holder of common stock with a preemptive right may share proportionately in all of the following except

A. The vote for directors.

B. Corporate assets upon liquidation.

C. Cumulative dividends.

Answer (C) is correct. Common stock does not have the right to accumulate unpaid dividends.

This right is often attached to preferred stock.

D. New issues of stock of the same class.

Gleim 2015 | Part 1 | Online MCQs | Unit 001

www.facebook.com/CMA.Arabwebsoft Page 24

حصري | لمنتدى عرب ويب سوفت

Question: 58 Which one of the following statements is correct regarding the effect preferred stock has on a company?

A. The firm’s after-tax profits are shared equally by common and preferred

shareholders.

B. Control of the firm is now shared by the common and preferred

shareholders, with preferred shareholders having greater control.

C. Preferred shareholders’ claims take precedence over the claims of common

shareholders in the event of liquidation.

Answer (C) is correct. Preferred stockholders have preference over common stockholders with

respect to dividend and liquidation rights, but payment of preferred

dividends, unlike bond interest is not mandatory. In exchange for these

preferences, the preferred stockholders give up the right to vote.

Consequently, preferred stock is a hybrid of debt and equity.

D. Nonpayment of preferred dividends places the firm in default, as does

nonpayment of interest on debt

Question: 59

Zinc Co.’s adjusted trial balance at December 31, Year 6, includes the following account balances:

Common stock, $3 par $600,000

Additional paid-in capital 800,000

Treasury stock, at cost 50,000

Net unrealized holding loss on

available-for-sale securities 20,000

Retained earnings: appropriated for

uninsured earthquake losses 150,000

Retained earnings: unappropriated 200,000

What amount should Zinc report as total equity in its December 31, Year 6, balance sheet?

A. $1,680,000

Answer (A) is correct. Total credits to equity equal $1,750,000 ($600,000 common stock at par

+ $800,000 additional paid-in capital + $350,000 retained earnings). The

treasury stock recorded at cost is subtracted from (debited to) total

equity, and the unrealized holding loss on available-for-sale securities is

debited to other comprehensive income, a component of equity. Because

total debits equal $70,000 ($50,000 cost of treasury stock + $20,000

unrealized loss on available-for-sale securities), total equity equals

Gleim 2015 | Part 1 | Online MCQs | Unit 001

www.facebook.com/CMA.Arabwebsoft Page 25

حصري | لمنتدى عرب ويب سوفت

$1,680,000 ($1,750,000 – $70,000).

B. $1,720,000

C. $1,780,000

D. $1,820,000

Question: 60 A retained earnings appropriation can be used to

A. Absorb a fire loss when a company is self-insured.

B. Provide for a contingent loss that is probable and reasonable.

C. Smooth periodic income.

D. Restrict earnings available for dividends.

Answer (D) is correct. Transfers to and from accounts properly designated as appropriated

retained earnings (such as general purpose contingency reserves or

provisions for replacement costs of fixed assets) are always excluded

from the determination of net income. However, appropriation of

retained earnings is permitted if it is displayed within the equity section

and is clearly identified. The effect of the appropriation is to restrict the

amount of retained earnings available for dividends, not to set aside

assets.

Question: 61 Which one of the following statements regarding treasury stock is correct?

A. It is unretired but no longer outstanding, yet it has all the rights of

outstanding shares.

B. It is an asset representing shares that can be sold in the future or otherwise

issued in stock option plans or in effectuating business combinations.

C. It is unable to participate in the liquidation proceeds of the firm but able to

participate in regular cash dividend distributions as well as stock dividends

and stock splits.

D. It is reflected in shareholders’ equity as a contra account.

Answer (D) is correct. Treasury stock recorded at cost is a reduction of total equity. Treasury

stock recorded at par is a direct reduction of the pertinent contributed

capital balance, e.g., common stock or preferred stock.

Question: 62 Tyler Corporation purchased 10,000 shares of its own $5 par-value common stock for $25 per share. This stock originally sold for $28 per share. Tyler used the cost method to record this transaction. If the par-value method had been used rather than the cost method, which of the following accounts would show a different dollar amount?

A. Treasury stock and total shareholders’ equity.

B. Additional paid-in capital and retained earnings.

C. Paid-in capital from treasury stock and retained earnings.

Gleim 2015 | Part 1 | Online MCQs | Unit 001

www.facebook.com/CMA.Arabwebsoft Page 26

حصري | لمنتدى عرب ويب سوفت

D. Additional paid-in capital and treasury stock.

Answer (D) is correct. Under the cost method, the treasury stock account was debited for the

full market price of the shares; had the par-value method been used,

treasury stock would only have been debited for the par value of the

shares. Under the cost method, the additional paid-in capital account was

not affected; had the par-value method been used, additional paid-in

capital would have been debited for the excess of the market price of the

shares over par.

Question: 63 On December 1, Noble Inc.’s Board of Directors declared a property dividend, payable in stock held in the Multon Company. The dividend was payable on January 5. The investment in Multon stock had an original cost of $100,000 when acquired 2 years ago. The market value of this investment was $150,000 on December 1, $175,000 on December 31, and $160,000 on January 5. The amount to be shown on Noble’s statement of financial position at December 31 as property dividends payable would be

A. $100,000

B. $150,000

Answer (B) is correct. When a property dividend is declared, the property is remeasured at its

fair value as of the declaration date. This amount is then reclassified from

retained earnings to property dividends payable.

C. $160,000

D. $175,000

Question: 64 Garland Corporation, a public company, has declared a property dividend of one share of its investment in Marlowe, Inc., for every 10 shares of its common stock outstanding. The Marlowe shares were originally purchased by Garland for $50 per share; on the date the dividend was declared, the market value was $75 per share. As a result of this declaration, Garland should recognize

A. A loss of $25 per share to be distributed.

B. A gain of $25 per share to be distributed.

Answer (B) is correct. When a property dividend is declared, the property is remeasured at its

fair value as of the declaration date ($75 – $50 = $25).

C. No gain or loss.

D. An appropriate gain or loss based on the market value on the date of

distribution.

Gleim 2015 | Part 1 | Online MCQs | Unit 001

www.facebook.com/CMA.Arabwebsoft Page 27

حصري | لمنتدى عرب ويب سوفت

Question: 65 Grand Corporation has 10,000,000 shares of $10 par-value stock authorized, of which 2,000,000 shares are issued and outstanding. The Board of Directors of Grand declared a 2-for-1 stock split on November 30 to be issued on December 30. The stock was selling for $30 per share on the date of declaration. In addition, the Board has amended the articles of incorporation to allow for a proportional increase in the number of authorized shares. The par-value information appearing in the shareholder’s equity section of Grand’s statement of financial position at December 31 will be

A. $5

Answer (A) is correct. As a result of the 2-for-1 stock split, the par value of Grand’s shares is

halved to $5.

B. $10

C. $15

D. $30

Question: 66 Fox Company has 1,000,000 shares of common stock authorized, of which 100,000 shares are held as treasury shares; the remainder are held by the company shareholders. On November 1, the Board of Directors declared a cash dividend of $.10 per share to be paid on January 2. At the same time, the Board declared a 5% stock dividend to be issued on December 31. On the date of the declaration, the stock was selling for $10 a share, and no fractional shares were to be issued. The total amount of these declarations to be shown as current liabilities on Fox’s statement of financial position as of December 31 is

A. $90,000

Answer (A) is correct. Cash dividends are only paid on outstanding shares. Thus, the dividend

payable at December 31 is $90,000 (900,000 × $.10). Stock dividends

distributable are reported in equity, not current liabilities.

B. $100,000

C. $540,000

D. $600,000

Question: 67 Bertram Company had a balance of $100,000 in retained earnings at the beginning of the year and of $125,000 at the end of the year. Net income for this time period was $40,000. Bertram’s statement of financial position indicated that the dividends payable account had decreased by $5,000 throughout the year, despite the fact that both cash dividends and a stock dividend were declared. The amount of the stock dividend was $8,000. When preparing its statement of cash flows for the year, Bertram should show cash paid for dividends as

A. $20,000

B. $15,000

C. $12,000

Answer (C) is correct. The amount of total dividends declared during the year can be calculated

as follows:

Gleim 2015 | Part 1 | Online MCQs | Unit 001

www.facebook.com/CMA.Arabwebsoft Page 28

حصري | لمنتدى عرب ويب سوفت

Beginning retained earnings $100,000

Net income for the year 40,000

Ending retained earnings (125,000)

Dividends declared during the year $ 15,000

Since $8,000 is the amount of stock dividends declared, the amount of

cash dividends declared this year is $7,000 ($15,000 – $8,000). The

amount of cash dividends paid during the year can be calculated as

follows:

Decrease in the cash dividends payable account during the

period $ 5,000

Cash dividends declared during the year 7,000

Cash paid for dividends during the year $12,000

NOTE: Stock dividends declared does not affect the dividends payable

account.

D. $5,000

Question: 68 How would a stock split affect the par value of the stock and the company’s shareholders’ equity?

Par Value Shareholders’ Equity

A. Decrease Increase

B. Decrease No change

Answer (B) is correct. A stock split reduces the par value of the stock and increases the number

of shares outstanding, making it more attractive to investors. As with a

stock dividend, each shareholder’s proportionate interest in the company

and total book value remain unchanged.

C. Increase Decrease

D. Increase No change

Question: 69 An undistributed stock dividend declared by the Board of Directors should be reported as a(n)

A. Current liability.

B. Long-term liability.

C. Footnote to the financial statements.

D. Item in the shareholders’ equity section.

Gleim 2015 | Part 1 | Online MCQs | Unit 001

www.facebook.com/CMA.Arabwebsoft Page 29

حصري | لمنتدى عرب ويب سوفت

Answer (D) is correct. In accounting for a stock dividend, the fair value of the additional shares

issued is reclassified from retained earnings to capital stock and the

difference to additional paid in capital. Stock dividend distributable is an

item of shareholders’ equity and not a liability.

Question: 70 Which one of the following statements regarding dividends is correct?

A. A stock dividend of 15% of the outstanding common shares results in a

debit to retained earnings at the par value of the stock distributed.

B. At the declaration date of a 30% stock dividend, the carrying value of

retained earnings will be reduced by the fair market value of the stock

distributed.

C. The declaration of a cash dividend will have no effect on book value per

share.

D. The declaration and payment of a 10% stock dividend will result in a

reduction of retained earnings at the fair market value of the stock.

Answer (D) is correct. When a small stock dividend is declared (less than 20% to 25% of the

previously outstanding common shares), retained earnings is debited for

the fair value of the stock.

Question: 71 Which one of the following transactions does not affect the balance of retained earnings?

A. Declaration of a stock dividend.

B. A quasi-reorganization.

C. Declaration of a stock split.

Answer (C) is correct. In a stock split, no journal entry is recorded and no retained earnings are

reclassified.

D. Declaration of a property dividend.

Question: 72 Underhall, Inc.’s common stock is currently selling for $108 per share. Underhall is planning a new stock issue in the near future and would like to stimulate interest in the company. The Board, however, does not want to distribute capital at this time. Therefore, Underhall is considering whether to offer a 2-for-1 common stock split or a 100% stock dividend on its common stock. The best reason for opting for the stock split is that

A. It will not decrease shareholders’ equity.

B. It will not impair the company’s ability to pay dividends in the future.

Answer (B) is correct. A 2-for-1 stock split doubles the number of shares outstanding; retained

earnings is not affected. Under a stock dividend, however, a portion of

retained earnings is reclassified as common stock. Since dividends are

Gleim 2015 | Part 1 | Online MCQs | Unit 001

www.facebook.com/CMA.Arabwebsoft Page 31

حصري | لمنتدى عرب ويب سوفت

restricted by the amount of available retained earnings, a stock dividend,

but not a stock split, will impair the firm’s ability to pay dividends in the

future.

C. The impact on earnings per share will not be as great.

D. The par value per share will remain unchanged.

Question: 73 When preparing the statement of cash flows, companies are required to report separately as operating cash flows all of the following except

A. Interest received on investments in bonds.

B. Interest paid on the company’s bonds.

C. Cash collected from customers.

D. Cash dividends paid on the company’s stock.

Answer (D) is correct. In general, the cash flows from transactions and other events that enter

into the determination of income are to be classified as operating. Cash

receipts from sales of goods and services, from interest on loans, and

from dividends on equity securities are from operating activities. Cash

payments to suppliers for inventory; to employees for wages; to other

suppliers and employees for other goods and services; to governments

for taxes, duties, fines, and fees; and to lenders for interest are also from

operating activities. However, distributions to owners (cash dividends on

a company’s own stock) are cash flows from financing, not operating,

activities.

Question: 74 A statement of cash flows is intended to help users of financial statements

A. Evaluate a firm’s liquidity, solvency, and financial flexibility.

Answer (A) is correct. The primary purpose of a statement of cash flows is to provide

information about the cash receipts and payments of an entity during a

period. If used with information in the other financial statements, the

statement of cash flows should help users to assess the entity’s ability to

generate positive future net cash flows (liquidity), its ability to meet

obligations (solvency) and pay dividends, the need for external financing,

the reasons for differences between income and cash receipts and

payments, and the cash and noncash aspects of the investing and

financing activities.

B. Evaluate a firm’s economic resources and obligations.

C. Determine a firm’s components of income from operations.

D. Determine whether insiders have sold or purchased the firm’s stock.

Gleim 2015 | Part 1 | Online MCQs | Unit 001

www.facebook.com/CMA.Arabwebsoft Page 31

حصري | لمنتدى عرب ويب سوفت

Question: 75 Which of the following items is specifically included in the body of a statement of cash flows?

A. Operating and nonoperating cash flow information.

Answer (A) is correct. All noncash transactions are excluded from the body of the statement of

cash flows to avoid undue complexity and detraction from the objective

of providing information about cash flows. Information about all noncash

financing and investing activities affecting recognized assets and

liabilities shall be reported in related disclosures.

B. Conversion of debt to equity.

C. Acquiring an asset through a capital lease.

D. Purchasing a building by giving a mortgage to the seller.

Question: 76 With respect to the content and form of the statement of cash flows, the

A. Pronouncements covering the cash flow statement encourage the use of the

indirect method.

B. Indirect method adjusts ending retained earnings to reconcile it to net cash

flows from operations.

C. Direct method of reporting cash flows from operating activities includes

disclosing the major classes of gross cash receipts and gross cash

payments.

Answer (C) is correct. The FASB encourages use of the direct method of reporting major

classes of operating cash receipts and payments, but the indirect method

may be used. The minimum disclosures of operating cash flows under

the direct method are cash collected from customers, interest and

dividends received, other operating cash receipts, cash paid to employees

and other suppliers of goods or services, interest paid, income taxes paid,

and other operating cash payments.

D. Reconciliation of the net income to net operating cash flow need not be

presented when using the direct method.

Question: 77 Depreciation expense is added to net income under the indirect method of preparing a statement of cash flows in order to

A. Report all assets at gross carrying amount.

B. Ensure depreciation has been properly reported.

C. Reverse noncash charges deducted from net income.

Answer (C) is correct. The indirect method begins with net income and then removes the effects

of past deferrals of operating cash receipts and payments, accruals of

expected future operating cash receipts and payments, and net income

items not affecting operating cash flows (e.g., depreciation).

Gleim 2015 | Part 1 | Online MCQs | Unit 001

www.facebook.com/CMA.Arabwebsoft Page 32

حصري | لمنتدى عرب ويب سوفت

D. Calculate net carrying amount.

Question: 78 All of the following should be classified under the operating section in a statement of cash flows except a

A. Decrease in inventory.

B. Depreciation expense.

C. Decrease in prepaid insurance.

D. Purchase of land and building in exchange for a long-term note.

Answer (D) is correct. Operating activities include all transactions and other events not

classified as investing and financing activities. Operating activities

include producing and delivering goods and providing services. Cash

flows from such activities are usually included in the determination of

net income. However, the purchase of land and a building in exchange

for a long-term note is an investing activity. Because this transaction

does not affect cash, it is reported in related disclosures of noncash

investing and financing activities.

Question: 79 Which one of the following transactions should be classified as a financing activity in a statement of cash flows?

A. Purchase of equipment.

B. Purchase of treasury stock.

Answer (B) is correct. Financing activities are defined to include the issuance of stock, the

payment of dividends, the receipt of donor-restricted resources to be used

for long-term purposes, treasury stock transactions (purchases or sales),

the issuance of debt, the repayment of amounts borrowed, obtaining and

paying for other resources obtained from creditors on long-term credit.

C. Sale of trademarks.

D. Payment of interest on a mortgage note.

Question: 80 Kelli Company acquired land by assuming a mortgage for the full acquisition cost. This transaction should be disclosed on Kelli’s statement of cash flows as a(n)

A. Financing activity.

B. Investing activity.

C. Operating activity.

D. Noncash financing and investing activity.

Answer (D) is correct. The exchange of debt for a long-lived asset does not involve a cash flow.

It is therefore classified as a noncash financing and investing activity.

Gleim 2015 | Part 1 | Online MCQs | Unit 001

www.facebook.com/CMA.Arabwebsoft Page 33

حصري | لمنتدى عرب ويب سوفت

Question: 81 Which one of the following transactions should not be classified as a financing activity in the statement of cash flows?

A. Issuance of common stock.

B. Purchase of treasury stock.

C. Payment of dividends.

D. Income tax refund.

Answer (D) is correct. Financing activities include obtaining resources from owners and

providing them with a return on, and a return of, their investment. Cash

inflows from financing activities include proceeds from issuing equity

instruments. Cash outflows include outlays to reacquire the enterprise’s

equity instruments, and outlays to pay dividends. However, an income

tax refund is an operating activity.

Question: 82 All of the following should be classified as investing activities in the statement of cash flows except

A. Cash outflows to purchase manufacturing equipment.

B. Cash inflows from the sale of bonds of other entities.

C. Cash outflows to lenders for interest.

Answer (C) is correct. Investing activities include the lending of money and the collecting of

those loans; the acquisition, sale, or other disposal of debt or equity

instruments; and the acquisition, sale, or other disposition of assets

(excluding inventory) that are held for or used in the production of goods

or services. Investing activities do not include acquiring and disposing of

certain loans or other debt or equity instruments that are acquired

specifically for resale. Cash outflows to lenders for interest are cash from

an operating, not an investing, activity.

D. Cash inflows from the sale of a manufacturing plant.

Question: 83 All of the following should be included in the reconciliation of net income to net operating cash flow in the statement of cash flows except a(n)

A. Decrease in inventory.

B. Decrease in prepaid insurance.

C. Purchase of land and building in exchange for a long-term note.

Answer (C) is correct. The purchase of land and a building in exchange for a long-term note is a

noncash investing activity that does not affect net income. Thus, it is

reported in the related disclosures section of the cash flow statement but

is not a reconciling item.

D. Increase in income tax payable.

Gleim 2015 | Part 1 | Online MCQs | Unit 001

www.facebook.com/CMA.Arabwebsoft Page 34

حصري | لمنتدى عرب ويب سوفت

Question: 84 In preparing a statement of cash flows, an item included in determining net cash flow from operating activities is the

A. Amortization of a bond premium.

Answer (A) is correct. The debtor (issuer) on a bond sold at a premium debits or reduces the

bond premium for the excess of cash interest paid over interest expense

recognized under the effective interest method. The lender (buyer)

likewise reduces the bond premium (by a credit) for the excess of cash

interest received over interest income recognized. Interest paid (received)

is a cash outflow (inflow) from an operating activity. In a reconciliation

of net income to net cash flow from operating activities, both the issuer

of the bond and the purchaser must make an adjustment for the

difference between the cash flow and the effect on net income. Because

the issuer’s cash outflow exceeded interest expense, it must deduct the

difference (premium amortization) from net income in performing the

reconciliation. The purchaser’s cash inflow is greater than interest

income, so it must add the difference (premium amortization) to net

income to arrive at net cash flow from operating activities.

B. Proceeds from the sale of equipment for cash.

C. Cash dividends paid.

D. Purchase of treasury stock.

Question: 85 The information reported in the statement of cash flows should help investors, creditors, and others to assess all of the following except the

A. Amount, timing, and uncertainty of prospective net cash inflows of a firm.

B. Company’s ability to pay dividends and meet obligations.

C. Company’s ability to generate future cash flows.

D. Management of the firm with respect to the efficient and profitable use of

its resources.

Answer (D) is correct. The statement of cash flows is not designed to provide information with

respect to the efficient and profitable use of the firm’s resources.

Financial reporting provides information about an enterprise’s

performance during a period when it was under the direction of a

particular management but does not directly provide information about

that management’s performance. Financial reporting does not try to

separate the impact of a particular management’s performance from the

effects of prior management actions, general economic conditions, the

supply and demand for an enterprise’s inputs and outputs, price changes,

and other events.

Gleim 2015 | Part 1 | Online MCQs | Unit 001

www.facebook.com/CMA.Arabwebsoft Page 35

حصري | لمنتدى عرب ويب سوفت

Question: 86 To calculate cash flows using the indirect method, which one of the following items must be added back to net income?

A. Revenue.

B. Marketing expense.

C. Depreciation expense.

Answer (C) is correct. The indirect method begins with accrual-basis net income or the change

in net assets and removes items that did not affect operating cash flow.

Depreciation is a non-cash item and thus does not affect the cash flows.

This amount must be added back to net income because it decreased net

income even though it had no cash effect.

D. Interest income.

Question: 87 The net income for Cypress, Inc., was $3,000,000 for the year ended December 31. Additional information is as follows:

Depreciation on fixed assets $1,500,000

Gain from cash sale of land 200,000

Increase in accounts payable 300,000

Dividends paid on preferred stock 400,000

The net cash provided by operating activities in the statement of cash flows for the year ended December 31 should be

A. $4,200,000

B. $4,500,000

C. $4,600,000

Answer (C) is correct. Net operating cash flow may be determined by adjusting net income.

Depreciation is an expense not directly affecting cash flows that should

be added back to net income. The increase in accounts payable is added

to net income because it indicates that an expense has been recorded but

not paid. The gain on the sale of land is an accrual-basis item affecting

net income and thus should be subtracted. The dividends paid on

preferred stock are cash outflows from financing, not operating, activities

and do not require an adjustment. Thus, net cash flow from operations is

$4,600,000 ($3,000,000 + $1,500,000 – $200,000 + $300,000).

D. $4,800,000

Gleim 2015 | Part 1 | Online MCQs | Unit 001

www.facebook.com/CMA.Arabwebsoft Page 36

حصري | لمنتدى عرب ويب سوفت

Fact Pattern: Royce Company had the following transactions during the fiscal year ended December 31, Year 2:

Accounts receivable decreased from $115,000 on December

31, Year 1, to $100,000 on December 31, Year 2.

Royce’s board of directors declared dividends on December

31, Year 2, of $.05 per share on the 2.8 million shares

outstanding, payable to shareholders of record on January 31,

Year 3. The company did not declare or pay dividends for

fiscal Year 1.

Sold a truck with a net

carrying amount of $7,000

for $5,000 cash, reporting a

loss of $2,000.

Paid interest to bondholders

of $780,000.

The cash balance was

$106,000 on December 31,

Year 1, and $284,000 on

December 31,

Question: 88 Royce Company uses the direct method to prepare its statement of cash flows at December 31, Year 2. The interest paid to bondholders is reported in the

A. Financing section, as a use or outflow of cash.

B. Operating section, as a use or outflow of cash.

Answer (B) is correct. Payment of interest on debt is considered a cash outflow from an

operating activity, although repayment of debt principal is a financing

activity.

C. Investing section, as a use or outflow of cash.

D. Debt section, as a use or outflow of cash.

Fact Pattern: Royce Company had the following transactions during the fiscal year ended December 31, Year 2:

Accounts receivable decreased from $115,000 on December

31, Year 1, to $100,000 on December 31, Year 2.

Royce’s board of directors declared dividends on December

31, Year 2, of $.05 per share on the 2.8 million shares

outstanding, payable to shareholders of record on January

31, Year 3. The company did not declare or pay dividends for

fiscal Year 1.

Sold a truck with a net

carrying amount of $7,000

for $5,000 cash, reporting a

loss of $2,000.

Paid interest to bondholders

of $780,000.

The cash balance was

$106,000 on December 31,

Year 1, and $284,000 on

December 31, Year 2.

Question: 89 Royce Company uses the indirect method to prepare its Year 2 statement of cash flows. It reports a(n)

A. Source or inflow of funds of $5,000 from the sale of the truck in the

financing section.

B. Use or outflow of funds of $140,000 in the financing section, representing

dividends.

C. Deduction of $15,000 in the operating section, representing the decrease in

year-end accounts receivable.

D. Addition of $2,000 in the operating section for the $2,000 loss on the sale

Gleim 2015 | Part 1 | Online MCQs | Unit 001

www.facebook.com/CMA.Arabwebsoft Page 37

حصري | لمنتدى عرب ويب سوفت

of the truck.

Answer (D) is correct. The indirect method determines net operating cash flow by adjusting net

income. Under the indirect method, the $5,000 cash inflow from the sale

of the truck is shown in the investing section. A $2,000 loss was

recognized and properly deducted to determine net income. This loss,

however, did not require the use of cash and should be added to net

income in the operating section.

Fact Pattern: Royce Company had the following transactions during the fiscal year ended December 31, Year 2:

Accounts receivable decreased from $115,000 on December

31, Year 1, to $100,000 on December 31, Year 2.

Royce’s board of directors declared dividends on December

31, Year 2, of $.05 per share on the 2.8 million shares

outstanding, payable to shareholders of record on January

31, Year 3. The company did not declare or pay dividends for

fiscal Year 1.

Sold a truck with a net

carrying amount of $7,000

for $5,000 cash, reporting a

loss of $2,000.

Paid interest to bondholders

of $780,000.

The cash balance was

$106,000 on December 31,

Year 1, and $284,000 on

December 31, Year 2.

Question: 90 The total of cash provided (used) by operating activities plus cash provided (used) by investing activities plus cash provided (used) by financing activities is

A. Cash provided of $284,000.

B. Cash provided of $178,000.

Answer (B) is correct. The total of cash provided (used) by the three activities (operating,

investing, and financing) should equal the increase or decrease in cash

for the year. During Year 2, the cash balance increased from $106,000 to

$284,000. Thus, the sources of cash must have exceeded the uses by

$178,000.

C. Cash used of $582,000.

D. Equal to net income reported for fiscal year ended December 31, Year 2.

Question: 91 The following information was taken from the accounting records of Oak Corporation for the year ended December 31:

Proceeds from issuance of preferred stock F $4,000,000

Dividends paid on preferred stock F 400,000

Bonds payable converted to common stock 2,000,000

Payment for purchase of machinery 500,000

Proceeds from sale of plant building 1,200,000

2% stock dividend on common stock 300,000

Gain on sale of plant building 200,000

Gleim 2015 | Part 1 | Online MCQs | Unit 001

www.facebook.com/CMA.Arabwebsoft Page 38

حصري | لمنتدى عرب ويب سوفت

The net cash flows from investing and financing activities that should be presented on Oak’s statement of cash flows for the year ended December 31 are, respectively,

A. $700,000 and $3,600,000.

Answer (A) is correct. The relevant calculations are as follows:

Proceeds from sale of plant building $1,200,000

Payment for purchase of machinery (500,000)

Net cash provided by investing activities $ 700,000

Proceeds from issuance of preferred stock $4,000,000

Dividends paid on preferred stock (400,000)

Net cash provided by financing activities $3,600,000

B. $700,000 and $3,900,000.

C. $900,000 and $3,900,000.

D. $900,000 and $3,600,000.

Question: 92 Zip Company entered into the following transactions during the year:

Purchased stock for $200,000

Purchased electronic equipment for use on the manufacturing floor for $300,000

Paid dividends to shareholders of Zip Company in the amount of $800,000

The amount to be reported in the investing activities section of Zip’s statement of cash flows

would be

A. $200,000

B. $500,000

Answer (B) is correct. The statement of cash flows classifies an enterprise’s cash flows into

three categories. Investing activities typically include the purchase and

sale of securities of other entities and the purchase and sale of property,

plant, and equipment. Thus, the amount to be reported in the investing

activities section of Zip’s statement of cash flows is $500,000 ($200,000

+ $300,000).

C. $800,000

D. $1,300,000

Question: 93 When using the statement of cash flows to evaluate a company’s continuing solvency, the most important factor to consider is the cash

A. Balance at the end of the period.

B. Flows from (used for) operating activities.

Answer (B) is correct. Solvency is the ability of an entity to pay its noncurrent debts as they

Gleim 2015 | Part 1 | Online MCQs | Unit 001

www.facebook.com/CMA.Arabwebsoft Page 39

حصري | لمنتدى عرب ويب سوفت

become due. A statement of cash flows provides information about,

among other things, an entity’s activities in generating cash through

operations (operating activities) to (1) repay debt, (2) distribute

dividends, or (3) reinvest to maintain or expand operating capacity. Thus,

cash flows from operating activities (net operating cash inflows), which

are generated by an entity’s ongoing major or central activities, are the

best indicator of its ability to remain solvent over the long term.

C. Flows from (used for) investing activities.

D. Flows from (used for) financing activities.

Question: 94 Dividends paid to shareholders are shown on the statement of cash flows as

A. Operating cash inflows.

B. Operating cash outflows.

C. Cash flows from investing activities.

D. Cash flows from financing activities.

Answer (D) is correct. The payment of dividends is a cash outflow from a financing activity.

The receipt of dividends, however, is generally considered a cash inflow

from an operating activity.

Question: 95 All of the following are classifications on the statement of cash flows except

A. Operating activities.

B. Equity activities.

Answer (B) is correct. The three classifications used on the statement of cash flows are

operating activities, investing activities, and financing activities.

C. Investing activities.

D. Financing activities.

Question: 96The sale of available-for-sale securities should be accounted for on the statement of cash

flows as a(n)

A. Operating activity.

B. Investing activity.

Answer (B) is correct. Investing activities include acquiring and disposing of debt or equity instruments.

C. Financing activity.

D. Noncash investing and financing activity.

Question: 97 Metro, Inc., reported net income of $150,000 for the current year. Changes occurred in

Gleim 2015 | Part 1 | Online MCQs | Unit 001

www.facebook.com/CMA.Arabwebsoft Page 41

حصري | لمنتدى عرب ويب سوفت

several balance sheet accounts during the current year as follows:

Investment in Videogold, Inc., stock, all of which was acquired in the

previous year, carried on the equity basis $5,500 increase

Accumulated depreciation, caused by major repair to projection

equipment 2,100 decrease

Premium on bonds payable 1,400 decrease

Deferred income tax liability (long-term) 1,800 increase In Metro’s current year cash flow statement, the reported net cash provided by operating activities should be

A. $150,400

B. $148,300

C. $144,900

Answer (C) is correct. The increase in the equity-based investment reflects the investor’s share

of the investee’s net income after adjustment for dividends received.

Hence, it is a noncash revenue and should be subtracted in the

reconciliation of net income to net operating cash inflow. A major repair

provides benefits to more than one period and therefore should not be

expensed. One method of accounting for a major repair is to charge

accumulated depreciation if the useful life of the asset has been extended,

with the offsetting credit to cash, a payable, etc. However, the cash

outflow, if any, is from an investing activity. The item has no effect on

net income and no adjustment is necessary. Amortization of bond

premium is a noncash income statement item that reduces accrual-basis

expenses and therefore must be subtracted from net income to arrive at

net cash flow from operating activities. The increase in the deferred tax

liability is a noncash item that reduces net income and should be added in

the reconciliation. Accordingly, net cash provided by operations is

$144,900 ($150,000 – $5,500 – $1,400 + $1,800).

D. $142,800

Question: 98 Hauschka Company reported net income for the year of $1,050,000. During the year, accounts receivable decreased $300,000, prepaid expenses increased $150,000, accounts payable for merchandise decreased $150,000, and liabilities for other expenses increased $100,000. Administrative expenses include depreciation expense of $50,000, and the company reported a loss on the sale of obsolete equipment of $10,000. Calculate Hauschka’s net cash flows from operating activities during the year.

A. $1,790,000

B. $1,690,000

C. $1,210,000

Answer (C) is correct. Net operating cash flow may be determined by adjusting net income. The

depreciation expense, decrease in accounts receivable, increase in

liabilities, and loss on the sale of obsolete equipment must be added

back. The increase in prepaid expense and decrease in accounts payable

must be subtracted from net income. Thus, net cash flow from operations

Gleim 2015 | Part 1 | Online MCQs | Unit 001

www.facebook.com/CMA.Arabwebsoft Page 41

حصري | لمنتدى عرب ويب سوفت

is $1,210,000 ($1,050,000 net income + $50,000 depreciation + $300,000

accounts receivable + $100,000 liabilities + $10,000 loss – $150,000

prepaid expenses – $150,000 accounts payable).

D. $1,110,000

Question: 99 Garnett Company’s year-end income statement shows the following:

Revenues $5,000,000

Selling and general expenses (including depreciation expense of

$200,000) 3,800,000

Interest expense 50,000

Gain on sale of equipment 40,000

Income tax expense (including deferred tax expense of $30,000) 320,000

Net income $ 870,000

During the year, Garnett’s noncash current assets rose by $100,000, and current liabilities increased by $150,000. On its statement of cash flows, Garnett would report cash provided by operating activities of

A. $1,080,000

B. $1,110,000

Answer (B) is correct. Net operating cash flow may be determined by adjusting net income. Net

income of $870,000 is decreased by the increase in current assets of

$100,000, increased by the increase in current liabilities of $150,000,

increased by depreciation expense of $200,000, decreased by the gain on

sale of equipment of $40,000, and increased by the deferred tax liability.

Thus, cash provided by operating activities would be $1,110,000.

C. $1,160,000

D. $1,190,000

Question: 100 An accountant with Nasbo Enterprises, Inc., has gathered the following information to prepare the statement of cash flows for the current year. Net income of $456,900 includes a deduction of $45,600 for depreciation expense. The company issued $300,000 of dividends this year and purchased one new building for $275,000. The balance sheets from the current period and prior period included the following balances:

Prior Year Current Year

Accounts receivable, net $ 56,860 $ 45,300

Accounts payable 12,900 10,745

Inventory 186,700 194,320

Using the indirect method, what is the amount of cash provided by operating activities?

A. $202,500

B. $405,205

Gleim 2015 | Part 1 | Online MCQs | Unit 001

www.facebook.com/CMA.Arabwebsoft Page 42

حصري | لمنتدى عرب ويب سوفت

C. $504,285

Answer (C) is correct. Net operating cash flow may be determined by adjusting net income.

Depreciation is an expense not directly affecting cash flows that should

be added back to net income. The decrease in accounts payable is

subtracted from net income because it indicates that an expense has been

paid, while the decrease in accounts receivable should be added to net

income. The increase in inventory should be subtracted from net income

because cash was used to purchase the inventory. The dividends paid on

preferred stock are cash outflows from financing, not operating, activities

and do not require an adjustment. Thus, net cash flow from operations is

$504,285 ($456,900 + $45,600 + $11,560 – $2,155 – $7,620).

D. $521,405

Question: 100 An accountant with Nasbo Enterprises, Inc., has gathered the following information to prepare the statement of cash flows for the current year. Net income of $456,900 includes a deduction of $45,600 for depreciation expense. The company issued $300,000 of dividends this year and purchased one new building for $275,000. The balance sheets from the current period and prior period included the following balances:

Prior Year Current Year

Accounts receivable, net $ 56,860 $ 45,300

Accounts payable 12,900 10,745

Inventory 186,700 194,320

Using the indirect method, what is the amount of cash provided by operating activities?

A. $202,500

B. $405,205

C. $504,285

Answer (C) is correct. Net operating cash flow may be determined by adjusting net income.

Depreciation is an expense not directly affecting cash flows that should

be added back to net income. The decrease in accounts payable is

subtracted from net income because it indicates that an expense has been

paid, while the decrease in accounts receivable should be added to net

income. The increase in inventory should be subtracted from net income

because cash was used to purchase the inventory. The dividends paid on

preferred stock are cash outflows from financing, not operating, activities

and do not require an adjustment. Thus, net cash flow from operations is

$504,285 ($456,900 + $45,600 + $11,560 – $2,155 – $7,620).

D. $521,405

Question: 101 Which one of the following would result in a decrease in cash flow measured under the indirect method of preparing a statement of cash flows?

A. Amortization expense.

Gleim 2015 | Part 1 | Online MCQs | Unit 001

www.facebook.com/CMA.Arabwebsoft Page 43

حصري | لمنتدى عرب ويب سوفت

B. Decrease in income taxes payable.

Answer (B) is correct. The indirect method reconciles accrual-basis net income to net operating

cash flow. A decrease in income taxes payable implies an operating cash

outflow not reflected in net income. Thus, the reconciling adjustment is a

subtraction from net income. The result is a lower measure of net

operating cash flow.

C. Proceeds from the issuance of common stock.

D. Decrease in inventories.

Question: 102 A statement of cash flows prepared using the indirect method would have cash activities listed in which one of the following orders?

A. Financing, investing, operating.

B. Investing, financing, operating.

C. Operating, financing, investing.

D. Operating, investing, financing.

Answer (D) is correct. A statement of cash flows prepared using either the direct or the indirect

method lists the categories of cash flows in the following order:

operating, investing, and financing.

Question: 103 Which one of the following should be classified as a cash flow from an operating activity on the statement of cash flows?

A. A decrease in accounts payable during the year.

Answer (A) is correct. Operating activities are all transactions and other events that are not

financing or investing activities. In general, operating activities involve

the production and delivery of goods and the provision of services. Their

effects normally are reported in earnings. A decrease in accounts payable

indicates a cash outflow to the entity’s suppliers in payment for goods or

services.

B. An increase in cash resulting from the issuance of previously authorized

common stock.

C. The payment of cash for the purchase of additional equipment needed for

current production.

D. The payment of a cash dividend from money arising from current

operations.

Gleim 2015 | Part 1 | Online MCQs | Unit 001

www.facebook.com/CMA.Arabwebsoft Page 44

حصري | لمنتدى عرب ويب سوفت

Question: 104 The most commonly used method for calculating and reporting a company’s net cash flow from operating activities on its statement of cash flows is the

A. Direct method.

B. Indirect method.

Answer (B) is correct. The FASB has expressed a preference for the direct method. However, if

the direct method is used, a separate reconciliation based on the indirect

method must be provided in a separate schedule. For this reason, most

entities use the indirect method. The same net operating cash flow is

reported under both methods.

C. Single-step method.

D. Multiple-step method.

Question: 105 The presentation of the major classes of operating cash receipts (such as receipts from customers) minus the major classes of operating cash disbursements (such as cash paid for merchandise) is best described as the

A. Direct method of calculating net cash provided or used by operating

activities.

Answer (A) is correct. The direct method converts the accrual-basis amounts in the income

statement to the cash basis. It then reports the separate categories of gross

cash receipts and disbursements. Net cash flow from operating activities

is the difference between total cash receipts and total cash disbursements.

B. Cash method of determining income in conformity with generally accepted

accounting principles.

C. Format of the statement of cash flows.

D. Indirect method of calculating net cash provided or used by operating

activities.

Question: 106 Larry Mitchell, Bailey Company’s controller, is gathering data for the statement of cash flows for the most recent year end. Mitchell is planning to use the direct method to prepare this statement and has made the following list of cash inflows for the period:

Collections of $100,000 for goods sold to customers

Securities purchased for investment purposes with an original cost of $100,000 sold

for $125,000

Proceeds from the issuance of additional company stock totaling $10,000

The correct amount to be shown as cash inflows from operating activities is

A. $100,000

Answer (A) is correct. Cash flows from operating activities are those generated by the firm’s

major and ongoing activities. They include cash flows from all activities

not classified as investing or financing. Only the $100,000 of collections

Gleim 2015 | Part 1 | Online MCQs | Unit 001

www.facebook.com/CMA.Arabwebsoft Page 45

حصري | لمنتدى عرب ويب سوفت

on sales to customers qualifies.

B. $135,000

C. $225,000

D. $235,000

Question: 107 During the year, Deltech, Inc., acquired a long-term productive asset for $5,000 and also borrowed $10,000 from a local bank. These transactions should be reported on Deltech’s statement of cash flows as

A. Outflows for investing activities, $5,000; inflows from financing activities,

$10,000.

Answer (A) is correct. The acquisition and disposal of property, plant, equipment, and other

productive assets are investing activities. Borrowing money is a

financing activity. Deltech’s transactions should therefore be reported on

its statement of cash flows as a $5,000 outflow for investing activities

and a $10,000 inflow from financing activities.

B. Inflows from investing activities, $10,000; outflows for financing activities,

$5,000.

C. Outflows for operating activities, $5,000; inflows from financing activities,

$10,000.

D. Outflows for financing activities, $5,000; inflows from investing activities,

$10,000.

Question: 108 Atwater Company has recorded the following payments for the current period:

Purchase Trillium stock $300,000

Dividends paid to Atwater shareholders 200,000

Repurchase of Atwater Company stock 400,000

The amount to be shown in the investing activities section of Atwater’s statement of cash flows should be

A. $300,000

Answer (A) is correct. Financing activities include paying dividends and treasury stock

transactions. Investing activities include acquiring and disposing of debt

and equity instruments. Thus, the amount to be shown in the investing

activities section of Atwater’s statement of cash flows is $300,000.

B. $500,000

C. $700,000

D. $900,000

Gleim 2015 | Part 1 | Online MCQs | Unit 001

www.facebook.com/CMA.Arabwebsoft Page 46

حصري | لمنتدى عرب ويب سوفت

Question: 109 Carlson Company has the following payments recorded for the current period:

Dividends paid to Carlson shareholders $150,000

Interest paid on bank loan 250,000

Purchase of equipment 350,000

The total amount of the above items to be shown in the operating activities section of Carlson’s statement of cash flows should be

A. $150,000

B. $250,000

Answer (B) is correct. Cash flows from operating activities include cash flows from all

activities not classified as investing or financing. Their effects normally

are reported in earnings. Operating cash flows include the payment and

collection of interest, dividends paid are a financing cash outflow, and

the purchase of equipment is an investing activity. Thus, the total amount

to be reported in the operating activities section of the statement of cash

flows is $250,000.

C. $350,000

D. $750,000

Question: 110 Barber Company has recorded the following payments for the current period:

Interest paid on bank loan $300,000

Dividends paid to Barber shareholders 200,000

Repurchase of Barber stock 400,000

The amount to be shown in the financing activities section of Barber’s statement of cash flows should be

A. $300,000

B. $500,000

C. $600,000

Answer (C) is correct. The payment and collection of interest are treated as cash flows from

operating activities. Financing activities include paying dividends and

treasury stock transactions. Thus, the amount to be reported in the

financing activities section of the statement of cash flows is $600,000

($200,000 + $400,000).

D. $900,000

Gleim 2015 | Part 1 | Online MCQs | Unit 001

www.facebook.com/CMA.Arabwebsoft Page 47

حصري | لمنتدى عرب ويب سوفت

Fact Pattern: Selected financial information for Kristina Company for the year just ended is shown below.

Net income $2,000,000

Increase in net accounts receivable 300,000

Decrease in inventory 100,000

Increase in accounts payable 200,000

Depreciation expense 400,000

Gain on the sale of available-for-sale securities 700,000

Cash receivable from the issue of common stock 800,000

Cash paid for dividends 80,000

Cash paid for the acquisition of land 1,500,000

Cash received from the sale of available-for-sale securities 2,800,000

Question: 111 Kristina’s cash flow from financing activities for the year is

A. $(80,000)

Answer (A) is correct. Cash flows from financing activities for the year consist of the $80,000

outflow for dividends paid. The issue of common stock is a financing

activity, but the $800,000 of proceeds have not yet been received.

B. $720,000

C. $800,000

D. $3,520,000

Fact Pattern: Selected financial information for Kristina Company for the year just ended is shown below.

Net income $2,000,000

Increase in net accounts receivable 300,000

Decrease in inventory 100,000

Increase in accounts payable 200,000

Depreciation expense 400,000

Gain on the sale of available-for-sale securities 700,000

Cash receivable from the issue of common stock 800,000

Cash paid for dividends 80,000

Cash paid for the acquisition of land 1,500,000

Gleim 2015 | Part 1 | Online MCQs | Unit 001

www.facebook.com/CMA.Arabwebsoft Page 48

حصري | لمنتدى عرب ويب سوفت

Cash received from the sale of available-for-sale securities 2,800,000

Question: 112 Kristina’s cash flow from investing activities for the year is

A. $(1,500,000)

B. $1,220,000

C. $1,300,000

Answer (C) is correct. Cash flows from investing activities for the year include the $2,800,000

inflow from the sale of available-for-sale securities and the $1,500,000

cash outflow for the purchase of land ($2,800,000 − $1,500,000 =

$1,300,000 net cash inflow).

D. $2,800,000

Question: 113 For the fiscal year just ended, Doran Electronics had the following results:

Net income $920,000

Depreciation expense 110,000

Increase in accounts payable 45,000

Increase in net accounts receivable 73,000

Increase in deferred income tax liability 16,000

Doran’s net cash flow from operating activities is

A. $928,000

B. $986,000

C. $1,018,000

Answer (C) is correct. The following is the net cash flow from operating activities calculated

using the indirect method:

Net income $ 920,000

Add: increase in accounts payable 45,000

Add: increase in deferred tax liability 16,000

Add: depreciation expense 110,000

Minus: increase in net accounts receivable (73,000)

Net cash provided by operating activities $1,018,000

Gleim 2015 | Part 1 | Online MCQs | Unit 001

www.facebook.com/CMA.Arabwebsoft Page 49

حصري | لمنتدى عرب ويب سوفت

The adjustment from cost of goods sold (an accrual accounting amount

used to calculate net income) to cash paid to suppliers requires two steps:

(1) from cost of goods sold to purchases and (2) from purchases to cash

paid to suppliers. An increase in inventory is subtracted from net income.

It indicates that purchases were greater than cost of goods sold. A

decrease in inventory is added to net income. It indicates that purchases

were less than cost of goods sold. However, the change in inventory is

not given, so it is assumed to be zero. The increase in accounts payable is

added to net income. It indicates that cash paid to suppliers was $45,000

less than purchases. Thus, the net effect of the changes in inventory and

accounts payable is that cash paid to suppliers was $45,000 ($0 +

$45,000) less than the accrual basis cost of goods sold. The increase in a

deferred income tax liability (debit income tax expense, credit deferred

liability) is a noncash item. The adjustment is a $16,000 addition to net

income. Depreciation ($110,000) also is a noncash item that is added to

net income. The net accounts receivable balance increased by $73,000,

implying that cash collections were less than sales. If sales, collections,

write-offs, and recognition of bad debt expense were the only relevant

transactions, $73,000 should be subtracted from net income. Use of the

change in net accounts receivable as a reconciliation adjustment is a

short-cut method. It yields the same net adjustment to net income as

separately including the effects of the change in gross accounts

receivable, bad debt expense (a noncash item resulting in an addition),

and bad debt write-offs (reflecting that write-offs did not result in

collections).

D. $1,074,000

Question: 114 Three years ago, Jameson Company purchased stock in Zebra, Inc., at a cost of $100,000. This stock was sold for $150,000 during the current fiscal year. The result of this transaction should be shown in the investing activities section of Jameson’s statement of cash flows as

A. Zero.

B. $50,000

C. $100,000

D. $150,000

Answer (D) is correct. The statement of cash flows reports the cash effects of transactions. The

accrual-basis gain on the stock is not relevant.

Gleim 2015 | Part 1 | Online MCQs | Unit 001

www.facebook.com/CMA.Arabwebsoft Page 51

حصري | لمنتدى عرب ويب سوفت

Question: 115 Madden Corporation’s controller has gathered the following information as a basis for preparing the statement of cash flows. Net income for the current year was $82,000. During the year, old equipment with a cost of $60,000 and a net carrying amount of $53,000 was sold for cash at a gain of $10,000. New equipment was purchased for $100,000. Shown below are selected closing balances for last year and the current year.

Last Year Current Year

Cash $ 39,000 $ 85,000

Accounts receivable net 43,000 37,000

Inventories 93,000 105,000

Equipment 360,000 400,000

Accumulated depreciation -- equipment 70,000 83,000

Accounts payable 22,000 19,000

Notes payable 100,000 100,000

Common stock 250,000 250,000

Retained earnings 93,000 175,000 Madden’s net cash flow from operating activities for the current year is

A. $63,000

B. $73,000

C. $83,000

Answer (C) is correct. The net operating cash flow may be determined by reconciling it with net

income.

Net income $ 82,000

Add: decrease in receivables 6,000

Add: depreciation expense 20,000

Minus: increase in inventories (12,000)

Minus: decrease in payables (3,000)

Minus: gain on sale of equipment (10,000)

Net cash provided by operating activities $ 83,000

The net accounts receivable balance declined by $6,000 ($43,000 –

$37,000), implying that cash collections exceeded sales. Assuming that

sales, collections, write-offs, and recognition of bad debt expense were

the only relevant transactions, $6,000 should be added to net income.

Use of the change in net accounts receivable as a reconciliation

adjustment is a short-cut method. It yields the same net adjustment to net

Gleim 2015 | Part 1 | Online MCQs | Unit 001

www.facebook.com/CMA.Arabwebsoft Page 51

حصري | لمنتدى عرب ويب سوفت

income as separately including the effects of the change in gross

accounts receivable, bad debt expense (a noncash item), and bad debt

write-offs.

Equipment costing $60,000 and having a carrying amount of $53,000

was sold on January 1, Year 6, for $63,000 in cash. Thus, the debit to

accumulated depreciation must have been $7,000 ($60,000 – $53,000).

During the year, Madden must have recognized $20,000 of depreciation

[$83,000 acc. dep. year end – ($70,000 acc. dep. begin. of yr. – $7,000

acc. dep. on sold equip.)]. The depreciation should be added to net

income because it is included in net income but had no cash effect.

The adjustment from cost of goods sold (an accrual accounting amount

used to calculate net income) to cash paid to suppliers requires two steps:

(1) from cost of goods sold to purchases and (2) from purchases to cash

paid to suppliers. The $12,000 ($105,000 – $93,000) increase in

inventory is subtracted from net income. It indicates that purchases were

$12,000 greater than cost of goods sold. The decrease in accounts

payable is subtracted from net income. It indicates that cash paid to

suppliers was $3,000 greater than purchases. Thus, the net effect of the

changes in inventory and accounts payable is that cash paid to suppliers

was $15,000 ($12,000 + $3,000) less than the accrual basis cost of goods

sold. The $10,000 gain on the sale of equipment is subtracted from net

income because it is a cash inflow from an investing, not an operating,

activity.

D. $93,000

Fact Pattern: Selected financial information for Kristina Company for the year just ended is shown below.

Net income $2,000,000

Increase in net accounts receivable 300,000

Decrease in inventory 100,000

Increase in accounts payable 200,000

Depreciation expense 400,000

Gain on the sale of available-for-sale securities 700,000

Cash receivable from the issue of common stock 800,000

Cash paid for dividends 80,000

Cash paid for the acquisition of land 1,500,000

Cash received from the sale of available-for-sale securities 2,800,000

Gleim 2015 | Part 1 | Online MCQs | Unit 001

www.facebook.com/CMA.Arabwebsoft Page 52

حصري | لمنتدى عرب ويب سوفت

Question: 116 Assuming the indirect method is used, Kristina’s cash flow from operating activities for the year is

A. $1,700,000

Answer (A) is correct. The following is the net cash flow from operating activities calculated

using the indirect method:

Net income $2,000,000

Add: decrease in inventory 100,000

Add: increase in accounts payable 200,000

Add: depreciation expense 400,000

Minus: increase in net accounts receivable (300,000)

Minus: gain on sale of securities (700,000)

Net cash provided by operating activities $1,700,000

The adjustment from cost of goods sold (an accrual accounting amount

used to calculate net income) to cash paid to suppliers requires two steps:

(1) from cost of goods sold to purchases and (2) from purchases to cash

paid to suppliers. The $100,000 decrease in inventory is added to net

income. It indicates that purchases were $100,000 less than cost of goods

sold. The $200,000 increase in accounts payable is added to net income.

It indicates that cash paid to suppliers was $200,000 less than purchases.

Thus, the net effect of the changes in inventory and accounts payable is

that cash paid to suppliers was $300,000 ($100,000 + $200,000) less than

the accrual basis cost of goods sold. Depreciation expense ($300,000) is

a noncash item included in net income. Hence, it is subtracted from net

income. The net accounts receivable balance increased by $300,000,

implying that cash collections were less than sales. If sales, collections,

write-offs, and recognition of bad debt expense were the only relevant

transactions, $300,000 should be subtracted from net income. Use of the

change in net accounts receivable as a reconciliation adjustment is a

short-cut method. It yields the same net adjustment to net income as

separately including the effects of the change in gross accounts

receivable, bad debt expense (a noncash item resulting in an addition),

and bad debt write-offs (a subtraction to reflect that write-offs did not

result in collections). The sale of securities is an investing activity. It also

is subtracted from net income.

B. $2,000,000

C. $2,400,000

D. $3,100,000

Gleim 2015 | Part 1 | Online MCQs | Unit 001

www.facebook.com/CMA.Arabwebsoft Page 53

حصري | لمنتدى عرب ويب سوفت

Question: 117 ABC operates a catering service that specializes in business luncheons for large corporations. ABC requires customers to place their orders 2 weeks in advance of the scheduled events. ABC bills its customers on the 10th day of the month following the date of service and requires that payment be made within 30 days of the billing date. Conceptually, ABC should recognize revenue from its catering services at the date when a

A. Customer places an order.

B. Luncheon is served.

Answer (B) is correct. Revenues should be recognized when (1) realized or realizable and (2)

earned. The most common time at which these two conditions are met is

when goods are delivered or services are rendered.

C. Billing is mailed.

D. Customer’s payment is received.

Question: 118 A company provides fertilization, insect control, and disease control services for a variety of trees, plants, and shrubs on a contract basis. For $50 per month, the company will visit the subscriber’s premises and apply appropriate mixtures. If the subscriber has any problems between the regularly scheduled application dates, the company’s personnel will promptly make additional service calls to correct the situation. Some subscribers elect to pay for an entire year because the company offers an annual price of $540 if paid in advance. For a subscriber who pays the annual fee in advance, the company should recognize the related revenue

A. When the cash is collected.

B. Evenly over the year as the services are performed.

Answer (B) is correct. Revenues should be recognized when (1) realized or realizable and (2)

earned. The most common time at which these two conditions are met is

when goods are delivered or services are rendered. In the situation

presented, the performance of the service (monthly spraying) is so

significant to creating a sufficient probability of a flow of future

economic benefits that it should be the triggering event for revenue

recognition.

C. At the end of the contract year after all of the services have been

performed.

D. At the end of the fiscal year.

Question: 119 On February 1, Year 1, a computer software firm agrees to program a software package. Twelve payments of $10,000 on the first of each month are to be made, with the first payment March 1, Year 1. The software is accepted by the client June 1, Year 2. How much Year 1 revenue should be recognized?

A. $0

Answer (A) is correct. Revenues should be recognized when (1) realized or realizable and (2)

earned. Because the software firm has not substantially fulfilled its

obligation, the earning process has not been substantially completed in

Gleim 2015 | Part 1 | Online MCQs | Unit 001

www.facebook.com/CMA.Arabwebsoft Page 54

حصري | لمنتدى عرب ويب سوفت

Year 1. Accordingly, a liability should be recognized because the entity

has a current obligation arising from a past event that will require an

outflow of economic benefits, that is, to deliver the software or to refund

the customer’s money. Thus, a liability for $100,000 and revenue of $0

should be recognized for Year 1.

B. $100,000

C. $110,000

D. $120,000

Question: 120 An airline should recognize revenue from airline tickets in the period when

A. Passenger reservations are booked.

B. Passenger reservations are confirmed.

C. Tickets are issued.

D. Related flights occur.

Answer (D) is correct. Revenues should be recognized when (1) realized or realizable and (2)

earned. Although the benefits of the service rendered are reliably

measurable on the date the reservations are booked, the earning process

is not substantially completed until the airline has fulfilled its obligation,

that is, when the related flights occur.

Question: 121 A department store sells gift certificates that may be redeemed for merchandise. Each certificate expires 3 years after issuance. The revenue from the gift certificates should be recognized

A. Evenly over 3 years from the date of issuance.

B. In the period the certificates are sold.

C. In the period the certificates expire.

D. In the period the certificates are redeemed or in the period they expire if

they are allowed to lapse.

Answer (D) is correct. Revenues should be recognized when (1) realized or realizable and (2)

earned. These criteria are met when the certificates are redeemed or

expire.

Question: 122 To comply with the matching principle, the cost of labor services of an employee who participates in the manufacturing of a product normally should be charged to the income statement in the period in which the

A. Work is performed.

B. Employee is paid.

C. Product is completed.

D. Product is sold.

Gleim 2015 | Part 1 | Online MCQs | Unit 001

www.facebook.com/CMA.Arabwebsoft Page 55

حصري | لمنتدى عرب ويب سوفت

Answer (D) is correct. The matching principle states that expenses should be recognized in the

same period as the revenues that those expenses helped produce.

Revenues related to the employee’s labor are not recognized until the

goods are sold.

Question: 123 Revenues of an entity are usually measured by the exchange values of the assets or liabilities involved. Recognition of revenue does not occur until

A. The revenue is realizable.

B. The revenue is realized and earned.

Answer (B) is correct. According to the FASB’s conceptual framework, revenues should be

recognized when they are realized or realizable and earned. Revenues are

realized when products, merchandise, or other assets are exchanged for

cash or claims to cash. Revenues are realizable when related assets

received or held are readily convertible to known amounts of cash or

claims to cash. Revenues are earned when the entity has substantially

accomplished what it must do to be entitled to the benefits represented by

the revenues.

C. Products or services are exchanged for cash or claims to cash.

D. The entity has substantially accomplished what it agreed to do.

Question: 124 Robin Gavaskar, who recently founded a company that produces baseball bats and balls, wants to determine her company’s policy for revenue recognition. According to the revenue recognition principle, the mostappropriate time to recognize revenue would be when

A. The sale occurs.

Answer (A) is correct. Revenues are normally recognized when they are realized or realizable

and earned. Revenues are realized (or realizable) when goods or services

have been exchanged for cash or claims to cash (assets readily

convertible to cash). Revenues are earned when the earning process is

substantially complete, and the entity is entitled to the resulting benefits

or revenues. The revenue recognition criteria are ordinarily met at the

point of sale (time of delivery of goods or services).

B. Cash is received.

C. Production is completed.

D. Quarterly financial statements are prepared.

Question: 125 For financial statement purposes, the installment method of accounting may be used if the

A. Collection period extends over more than 12 months.

B. Installments are due in different years.

C. Ultimate amount collectible is indeterminate.

Gleim 2015 | Part 1 | Online MCQs | Unit 001

www.facebook.com/CMA.Arabwebsoft Page 56

حصري | لمنتدى عرب ويب سوفت

Answer (C) is correct. Profits from sales in the ordinary course of business usually should be

recognized at the time of sale unless collection of the sales price is not

reasonably assured. When receivables are collected over an extended

period and, because of the terms of the transaction or other conditions, no

reasonable basis exists for estimating the degree of collectibility, the

installment method or the cost-recovery method of accounting may be

used.

D. Percentage-of-completion method is inappropriate.

Question: 126 It is proper to recognize revenue prior to the sale of merchandise when

I. The revenue will be reported as an installment sale.

II. The revenue will be reported under the cost-recovery method.

A. I only.

B. II only.

C. Both I and II.

D. Neither I nor II.

Answer (D) is correct. The installment method recognizes income on a sale as the related

receivable is collected. Under the cost-recovery method, profit is

recognized only after collections exceed the cost of the item sold.

Question: 127 Several of Fox, Inc.’s customers are having cash flow problems. Information pertaining to these customers for the years ended March 31, Year 7 and Year 8 follows:

3/31/Yr 7 3/31/Yr 8

Sales $10,000 $15,000

Cost of sales 8,000 9,000

Cash collections

on Year 7 sales 7,000 3,000

on Year 8 sales -- 12,000

If the cost-recovery method is used, what amount would Fox report as gross profit from sales to these customers for the year ended March 31, Year 8?

A. $2,000

B. $3,000

C. $5,000

Answer (C) is correct. The cost-recovery method recognizes profit only after collections exceed

Gleim 2015 | Part 1 | Online MCQs | Unit 001

www.facebook.com/CMA.Arabwebsoft Page 57

حصري | لمنتدى عرب ويب سوفت

the cost of the item sold, that is, when the full cost has been recovered.

Subsequent amounts collected are treated entirely as revenue (debit cash

and deferred gross profit, credit the receivable and realized gross profit).

The sum of collections in excess of costs to be recognized as gross profit

is $5,000 {[$3,000 Year 8 collections on Year 7 sales – ($8,000 cost –

$7,000 Year 7 collections on Year 7 sales)] + ($12,000 collections on

Year 8 sales – $9,000 cost)}.

D. $15,000

Question: 128 Paulson Company uses the percentage-of-completion method to account for long-term construction contracts. The following information relates to a contract that was awarded at a price of $700,000. The estimated costs were $500,000, and the contract duration was 3 years.

Year 1 Year 2 Year 3

Cumulative cost to date $300,000 $390,000 $530,000

Costs to complete at year end 250,000 130,000 --

Progress billings 325,000 220,000 155,000

Collections on account 300,000 200,000 200,000 Assuming that $65,000 was recognized as gross profit in Year 1, the amount of gross profit Paulson recognized in Year 2 was

A. $35,000

B. $70,000

Answer (B) is correct. Determining the annual recognized gross profit requires calculation of

the estimated total gross profit.

Year 1 Year 2

Contract price $700,000

$700,000

Minus: estimated total costs

Costs to date $300,000 $390,000

Estimated costs to complete 250,000

130,000

Estimated total costs $550,000

$520,000

Estimated total gross profit $150,000

$180,000

The completion percentage for Year 2 is the ratio of costs incurred to

date to estimated total costs ($390,000 ÷ $520,000 = 75%). The

cumulative gross profit recognized at the end of Year 2 is therefore

$135,000 ($180,000 × 75%). Because $65,000 was recognized in Year 1,

the amount recognized in Year 2 is $70,000 ($135,000 – $65,000).

Gleim 2015 | Part 1 | Online MCQs | Unit 001

www.facebook.com/CMA.Arabwebsoft Page 58

حصري | لمنتدى عرب ويب سوفت

C. $135,000

D. $170,000

Question: 129 The calculation of the income recognized in the third year of a 5-year construction contract accounted for using the percentage-of-completion method includes the ratio of

A. Costs incurred in Year 3 to total billings.

B. Costs incurred in Year 3 to total estimated costs.

C. Total costs incurred to date to total billings.

D. Total costs incurred to date to total estimated costs.

Answer (D) is correct. The percentage-of-completion method recognizes gross profit or revenue

based on the ratio of costs to date to estimated total costs. (This

relationship is the recommended but not the only basis for determining

progress.)

Question: 130 A company appropriately uses the completed-contract method to account for a long-term construction contract. Revenue is recognized when progress billings are

Recorded Collected

A. No Yes

B. Yes Yes

C. Yes No

D. No No

Answer (D) is correct. GAAP require that revenue be recognized when it is realized or

realizable and earned. Under the completed-contract method, revenue

recognition is appropriate only at the completion of the contract. Neither

the recording nor the collection of progress billings affects this

recognition.

Question: 131 A building contractor has a fixed-price contract to construct a large building. It is estimated that the building will take 2 years to complete. Progress billings will be sent to the customer at quarterly intervals. Which of the following describes the preferable point for revenue recognition for this contract if the outcome of the contract can be estimated reliably?

A. After the contract is signed.

B. As progress is made toward completion of the contract.

Answer (B) is correct. Under the percentage-of-completion method, revenues and expenses are

recognized based on the stage of completion at the balance sheet date if

the outcome of the contract can be estimated reliably. For a fixed-price

contract, the outcome can be estimated reliably if (1) total revenue can be

measured reliably, (2) it is probable that the economic benefits of the

Gleim 2015 | Part 1 | Online MCQs | Unit 001

www.facebook.com/CMA.Arabwebsoft Page 59

حصري | لمنتدى عرب ويب سوفت

contract will flow to the enterprise, (3) contract costs to complete and

stage of completion can be measured reliably, and (4) contract costs can

be clearly identified and measured reliably so that actual and estimated

costs can be compared.

C. As cash is received.

D. When the contract is completed.

Question: 132 How should the balances of progress billings and construction in progress be shown at reporting dates prior to the completion of a long-term contract?

A. Progress billings as deferred income, construction in progress as a deferred

expense.

B. Progress billings as income, construction in progress as inventory.

C. Net, as a current asset if debit balance and current liability if credit balance.

Answer (C) is correct. The difference between construction in progress (costs and recognized

gross profit) and progress billings to date must be reported as a current

asset if construction in progress exceeds total billings, and as a current

liability if billings exceed construction in progress. Separate recognition

is required for each project.

D. Net, as gross profit from construction if credit balance, and loss from

construction if debit balance.

Question: 133 During Year 1, Tidal Co. began construction on a project scheduled for completion in Year 3. At December 31, Year 1, an overall loss was anticipated at contract completion. What would be the effect of the project on Year 1 operating income under the percentage-of-completion method and the completed-contract method?

Percentage-of- Completion Completed-Contract

A. No effect No effect

B. No effect Decrease

C. Decrease No effect

D. Decrease Decrease

Answer (D) is correct. When the current estimate of total contract costs indicates a loss, an

immediate provision for the entire loss should be made regardless of

method. Thus, under either method, Year 1 operating income is

decreased by the projected loss.

Gleim 2015 | Part 1 | Online MCQs | Unit 001

www.facebook.com/CMA.Arabwebsoft Page 61

حصري | لمنتدى عرب ويب سوفت

Question: 134 A company began work on a long-term construction contract in Year 1. The contract price was $3,000,000. Year-end information related to the contract is as follows:

Year 1 Year 2 Year 3

Estimated total cost $2,000,000 $2,000,000 $2,000,000

Cost incurred 700,000 900,000 400,000

Billings 800,000 1,200,000 1,000,000

Collections 600,000 1,200,000 1,200,000

Under the percentage-of-completion method, the gross profit to be recognized in Year 1 is

A. $(100,000)

B. $100,000

C. $200,000

D. $350,000

Answer (D) is correct. The percentage-of-completion method recognizes revenue based on the

stage of completion of the contract. One typical method for estimating

the stage of completion is the calculation of ratio of the contract costs

incurred to date to the estimated total costs. The percentage-of-

completion at year-end on the cost-to-cost basis is 35% ($700,000 ÷

$2,000,000). The gross profit for Year 1 is the anticipated gross profit on

the contract times the completion percentage. Thus, profit for Year 1 is

$350,000 [($3,000,000 – $2,000,000) × 35%].

أدعمنا بـ عرب ويب سوفت

الموقع الرائد في الشهادات المهنيه

نحو غد أفضل

www.facebook.com/CMA.arabwebsoft

www.arabwebsoft.com/forums

Gleim 2015 | Part 1 | Online MCQs | Unit 002

www.facebook.com/CMA.Arabwebsoft Page 1

حصري | لمنتدى عرب ويب سوفت

Question: 135 The measurement basis most often used to report a long-term payable representing a commitment to pay money at a determinable future date is

A. Historical cost.

B. Current cost.

C. Net realizable value.

D. Present value of future cash flows.

Answer (D) is correct. The measurement basis most commonly adopted by entities in preparing

their financial statements is historical cost. However, it is usually

combined with other measurement bases (attributes). The attribute used

to measure a long-term receivable or payable is the present or discounted

value of its future cash flows.

Question: 136 Statements of financial position on December 31, Year 1, and December 31, Year 2, are presented below:

Dec. 31,

Year 1

Dec. 31,

Year 2

Assets:

Cash $ 50,000 $ 60,000

Accounts receivable 95,000 89,000

Allowance for uncollectible accounts (4,000) (3,000)

Inventory 120,000 140,000

Property, plant, and equipment 295,000 340,000

Accumulated depreciation (102,000) (119,000)

Total Assets $ 454,000 $ 507,000

Liabilities and equity:

Trade accounts payable $ 62,000 $ 49,000

Interest payable 8,000 11,000

Bonds payable 200,000 200,000

Unamortized bond discount (15,000) (10,000)

Equity 199,000 257,000

Total liabilities and equity $ 454,000 $ 507,000

Additional information for Year 2:

1. Sales revenue was $338,000.

2. $3,000 of accounts receivable was written off.

Cash collections from customers in Year 2 were

A. $341,000

Gleim 2015 | Part 1 | Online MCQs | Unit 002

www.facebook.com/CMA.Arabwebsoft Page 2

حصري | لمنتدى عرب ويب سوفت

Answer (A) is correct. Cash collections from customers equals beginning accounts receivable,

plus sales revenue, minus accounts written off, minus ending accounts

receivable. In Year 2, cash collections from customers were $341,000

($95,000 + $338,000 – $3,000 – $89,000).

B. $338,000

C. $344,000

D. $335,000

Question: 137 An analysis of an entity’s $150,000 accounts receivable at year end resulted in a $5,000 ending balance for its allowance for uncollectible accounts and a bad debt expense of $2,000. During the past year, recoveries on bad debts previously written off were correctly recorded at $500. If the beginning balance in the allowance for uncollectible accounts was $4,700, what was the amount of accounts receivable written off as uncollectible during the year?

A. $1,200

B. $1,800

C. $2,200

Answer (C) is correct. Under the allowance method, uncollectible accounts are written off by a

debit to the allowance and a credit to accounts receivable. The $500 of

recovered bad debts is accounted for by a debit to accounts receivable

and a credit to the allowance. The $2,000 bad debt expense is also

credited to the allowance. The amount of accounts receivable written off

can be calculated as follows:

Beginning allowance $4,700

Bad debt expense 2,000

Recoveries 500

Ending allowance (5,000)

A/R written off $2,200

D. $2,800

Question: 138 The following information applies to Nichola Manufacturing Company, which has a 6-month operating cycle:

Cash sales $100,000

Credit sales during the sixth month with net 30 days terms 150,000

Credit sale during the fifth month with special terms of net 9 months 10,000

Gleim 2015 | Part 1 | Online MCQs | Unit 002

www.facebook.com/CMA.Arabwebsoft Page 3

حصري | لمنتدى عرب ويب سوفت

Interest earned and accrued on an investment that matures during month 3

of the next cycle 2,000

The total of Nichola’s trade accounts receivable at the end of the current cycle is

A. $152,000

B. $160,000

Answer (B) is correct. A receivable classified as current on the statement of financial position is

expected to be collected within the current operating cycle or 1 year,

whichever is longer. The total of the trade accounts receivable at the end

of the current cycle is therefore $160,000 ($150,000 + $10,000).

C. $260,000

D. $262,000

Question: 139 Johnson Company uses the allowance method to account for uncollectible accounts receivable. After recording the estimate of uncollectible accounts expense for the current year, Johnson decided to write off in the current year the $10,000 account of a customer who had filed for bankruptcy. What effect does this write-off have on the company’s current net income and total current assets, respectively?

Net Income

Total Current Assets

A. Decrease Decrease

B. No effect Decrease

C. Decrease No effect

D. No effect No effect

Answer (D) is correct. Johnson uses the allowance method. Thus, when a specific amount is

written off, the journal entry is

Allowance for doubtful accounts $10,000

Accounts receivable $10,000

The write-off of a bad debt has no effect on expenses, net income, and

total current assets.

Question: 140 Based on the industry average, Davis Corporation estimates that its bad debts should average 3% of credit sales. The balance in the allowance for uncollectible accounts at the beginning of Year 3 was $140,000. During Year 3, credit sales totaled $10,000,000, accounts of $100,000 were deemed to be uncollectible, and payment was received on a $20,000 account that had previously been written off as uncollectible. The entry to record bad debt expense at the end of Year 3 would include a credit to the allowance for uncollectible accounts of

A. $300,000

Gleim 2015 | Part 1 | Online MCQs | Unit 002

www.facebook.com/CMA.Arabwebsoft Page 4

حصري | لمنتدى عرب ويب سوفت

Answer (A) is correct. Bad debt expense is based on the income statement approach. It treats

bad debt expense as a function of sales on account. Thus, it is projected

to be $300,000 ($10,000,000 × 3%). The entry to record bad debt

expense is

Bad debt expense $300,000

Allowance for doubtful accounts $300,000

B. $260,000

C. $240,000

D. $160,000

Question: 141 The following information has been compiled by Able Manufacturing Company:

Sale of company products for the period to customers with net 30-day terms

amounting to $150,000.

Sale of company products for the period to a customer, supported by a note for

$25,000, with special terms of net 180 days.

Balance of trade receivables at the end of the last period was $300,000.

Collections of open trade receivables during the period was $200,000.

Rental income for the period, both earned and accrued but not yet collected, from

the Able Employees’ Credit Union for use of company facilities was $2,000.

The open trade receivables balance to be shown on the statement of financial position

for the period is

A. $250,000

Answer (A) is correct. The open trade receivables balance is calculated as follows:

Previous ending balance $300,000

Add: sales to customers (terms net 30) 150,000

Minus: collections during period (200,000)

Open trade receivables reported $250,000

B. $252,000

C. $275,000

D. $277,000

Gleim 2015 | Part 1 | Online MCQs | Unit 002

www.facebook.com/CMA.Arabwebsoft Page 5

حصري | لمنتدى عرب ويب سوفت

Question: 142 The following information relates to Jay Co.’s accounts receivable for the year just ended:

Accounts receivable, 1/1 $ 650,000

Credit sales for the year 2,700,000

Sales returns for the year 75,000

Accounts written off during the year 40,000

Collections from customers during the year 2,150,000

Estimated future sales returns at 12/31 50,000

Estimated uncollectible accounts at 12/31 110,000

What amount should Jay report for accounts receivable, before allowances for sales returns and uncollectible accounts, at December 31?

A. $1,200,000

B. $1,125,000

C. $1,085,000

Answer (C) is correct. The ending balance in accounts receivable consists of the $650,000

beginning debit balance, plus debits for $2,700,000 of credit sales, minus

credits for $2,150,000 of collections, $40,000 of accounts written off,

and $75,000 of sales returns.

Accounts Receivable (in 000s)

1/1 $ 650 $ 75 Sales returns

Credit sales 2,700 2,150 Collections

40 Write-offs

12/31 $1,085

The $110,000 of estimated uncollectible receivables and the $50,000 of

estimated sales returns are not relevant because they affect the allowance

accounts but not gross accounts receivable.

D. $925,000

Question: 143 A shoe retailer allows customers to return shoes within 90 days of purchase. The company estimates that 5% of sales will be returned within the 90-day period. During the month, the company has sales of $200,000 and returns of sales made in prior months of $5,000. What amount should the company record as net sales revenue for new sales made during the month?

A. $185,000

B. $190,000

Gleim 2015 | Part 1 | Online MCQs | Unit 002

www.facebook.com/CMA.Arabwebsoft Page 6

حصري | لمنتدى عرب ويب سوفت

Answer (B) is correct. The company has $200,000 of sales and estimates that 5% of sales will

be returned. Thus, the company will recognize $10,000 ($200,000 × 5%)

for sales returns (contra revenue) and for a corresponding allowance for

sales returns (contra asset). This amount is subtracted from total sales to

find net sales revenue of $190,000 ($200,000 – $10,000).

C. $195,000

D. $200,000

Question: 144 An internal auditor is deriving cash flow data based on an incomplete set of facts. Bad debt expense was $2,000. Additional data for this period follows:

Credit sales $100,000

Gross accounts receivable -- beginning balance 5,000

Allowance for bad debts -- beginning balance (500)

Accounts receivable written off 1,000

Increase in net accounts receivable (after subtraction of allowance for bad

debts) 30,000

How much cash was collected this period on credit sales?

A. $64,000

B. $68,000

Answer (B) is correct. The beginning balance of gross accounts receivable (A/R) was $5,000

(debit). Thus, net beginning A/R was $4,500 ($5,000 – $500 credit in the

allowance for bad debts). The allowance was credited for the $2,000 bad

debt expense. Accordingly, the ending allowance (credit) was $1,500

($500 – $1,000 write-off + $2,000). Given a $30,000 increase in net A/R,

ending net A/R must have been $34,500 ($4,500 beginning net A/R +

$30,000), with ending gross A/R of $36,000 ($34,500 + $1,500).

Collections were therefore $68,000 ($5,000 beginning gross A/R –

$1,000 write-off + $100,000 credit sales – $36,000 ending gross A/R).

Gross A/R

$ 5,000 Beg. Bal. $ 1,000 Write-off

100,000 Cr. Sales 68,000 Collections

$ 36,000

End. Bal.

C. $68,500

D. $70,000

Gleim 2015 | Part 1 | Online MCQs | Unit 002

www.facebook.com/CMA.Arabwebsoft Page 7

حصري | لمنتدى عرب ويب سوفت

Question: 145 Marr Co. had the following sales and accounts receivable balances, prior to any adjustments at year end:

Credit sales $10,000,000

Accounts receivable 3,000,000

Allowance for uncollectible accounts 50,000

Marr uses 3% of accounts receivable to determine its allowance for uncollectible accounts at year end. By what amount should Marr adjust its allowance for uncollectible accounts at year end?

A. $0

B. $40,000

Answer (B) is correct. The entity uses the percentage of accounts receivable method to estimate

the allowance. The year-end balance should be $90,000 ($3,000,000 A/R

× 3%). Hence, the year-end adjustment is $40,000 ($90,000 – $50,000)

unadjusted balance.

C. $90,000

D. $140,000

Question: 146 When the allowance method of recognizing uncollectible accounts is used, the entry to record the write-off of a specific account

A. Decreases both accounts receivable and the allowance for uncollectible

accounts.

Answer (A) is correct. When an account receivable is written off, both accounts receivable and

the allowance for uncollectible accounts are decreased. If an account

previously written off is collected, the account must be reinstated by

increasing both accounts receivable and the allowance. The account

receivable is then decreased by the amount of cash collected.

B. Decreases accounts receivable and increases the allowance for

uncollectible accounts.

C. Increases the allowance for uncollectible accounts and decreases net

income.

D. Decreases both accounts receivable and net income.

Question: 147 Wren Company had the following account balances at December 31:

Accounts receivable $ 900,000

Allowance for uncollectible accounts

(before any provision for the year

Gleim 2015 | Part 1 | Online MCQs | Unit 002

www.facebook.com/CMA.Arabwebsoft Page 8

حصري | لمنتدى عرب ويب سوفت

uncollectible accounts expense) 16,000

Credit sales for the year 1,750,000

Wren is considering the following methods of estimating uncollectible accounts expense for the year:

Based on credit sales at 2%

Based on accounts receivable at 5%

What amount should Wren charge to uncollectible accounts expense under each method?

Percentage of Percentage of Credit Sales Accounts Receivable

A. $51,000 $45,000

B. $51,000 $29,000

C. $35,000 $45,000

D. $35,000 $29,000

Answer (D) is correct. Uncollectible accounts expense is estimated in two ways. One

emphasizes asset valuation, while the other emphasizes income

measurement. The first is based on an aging of the receivables to

determine the balance in the allowance for uncollectible accounts. Bad

debt expense is the amount necessary to adjust the allowance account to

this estimated balance. The second recognizes bad debt expense as a

percentage of sales. The corresponding credit is to the allowance for

uncollectible accounts. Under the first method, if uncollectible accounts

are estimated to be 5% of gross accounts receivable, the allowance

account should have a balance of $45,000 ($900,000 × 5%), and the

entry is to debit uncollectible accounts expense and credit the allowance

for $29,000 ($45,000 – $16,000 existing balance). Under the second

method, bad debt expense is $35,000 ($1,750,000 × 2%).

Question: 148 On March 31, Vale Co. had an unadjusted credit balance of $1,000 in its allowance for uncollectible accounts. An analysis of Vale’s trade accounts receivable at that date revealed the following:

Estimated

Age Amount Uncollectible

0-30 days $60,000 5%

31-60 days 4,000 10%

Over 60 days 2,000 70%

What amount should Vale report as allowance for uncollectible accounts in its March 31 balance sheet?

A. $4,800

Gleim 2015 | Part 1 | Online MCQs | Unit 002

www.facebook.com/CMA.Arabwebsoft Page 9

حصري | لمنتدى عرب ويب سوفت

Answer (A) is correct. The aging schedule determines the balance in the allowance for

uncollectible accounts. Of the accounts that are no more than 30 days

old, the amount uncollectible is $3,000 ($60,000 × 5%). Accounts that

are 31-60 days old and over 60 days old have estimated uncollectible

balances of $400 ($4,000 × 10%) and $1,400 ($2,000 × 70%),

respectively. Hence, the amount recorded in the allowance for

uncollectible accounts is $4,800 ($3,000 + $400 + $1,400). The $1,000

balance already in the account is disregarded because the aging schedule

determines the balance that should be in the account.

B. $4,000

C. $3,800

D. $3,000

Question: 149 Which method of recording uncollectible accounts expense is consistent with accrual accounting?

Allowance Direct Write-Off

A. Yes Yes

B. Yes No

Answer (B) is correct. The allowance method attempts both to match the expense with the

related revenue and to determine the NRV of the accounts receivable.

This method is acceptable under GAAP. The direct write-off method

debits expense and credits accounts receivable at the time uncollectibility

is established. This method does not match revenue and expense or state

receivables at NRV. It is not acceptable under GAAP.

C. No Yes

D. No No

Question: 150 Under the allowance method of recognizing uncollectible accounts, the entry to write-off an uncollectible account

A. Increases the allowance for uncollectible accounts.

B. Has no effect on the allowance for uncollectible accounts.

C. Has no effect on net income.

Answer (C) is correct. The entry to record bad debt expense under the allowance method is to

debit bad debt expense and credit the allowance account. When a specific

account is then written off, the allowance is debited and accounts

receivable credited. Net income is affected when bad debt expense is

recognized, not at the time of the write-off. Because accounts receivable

and the allowance account are decreased by the same amount, a write-off

of an account also has no effect on the net amount of accounts

receivable.

Gleim 2015 | Part 1 | Online MCQs | Unit 002

www.facebook.com/CMA.Arabwebsoft Page 11

حصري | لمنتدى عرب ويب سوفت

D. Decreases net income.

Question: 151 The following accounts were abstracted from Roxy Co.’s unadjusted trial balance at December 31:

Debit Credit

Accounts receivable $1,000,000

Allowance for uncollectible accounts 8,000

Net credit sales $3,000,000

Roxy estimates that 3% of the gross accounts receivable will become uncollectible. After adjustment at December 31, the allowance for uncollectible accounts should have a credit balance of

A. $90,000

B. $82,000

C. $38,000

D. $30,000

Answer (D) is correct. The allowance for uncollectible accounts at year end should have a credit

balance of $30,000. This amount is equal to the $1 million of accounts

receivable multiplied by the 3% that are estimated to become

uncollectible.

Question: 152 Mill Co.’s allowance for uncollectible accounts was $100,000 at the end of Year 2 and $90,000 at the end of Year 1. For the year ended December 31, Year 2, Mill reported bad debt expense of $16,000 in its income statement. What amount did Mill debit to the appropriate account in Year 2 to write off actual bad debts?

A. $6,000

Answer (A) is correct. When uncollectible accounts are written off, a debit is made to the

allowance and a credit to accounts receivable. The beginning balance in

the allowance account is $90,000, the ending balance is $100,000, and

the bad debt expense is $16,000. Because write-offs equal the beginning

balance, plus the bad debt expense, minus the ending balance, $6,000 of

accounts must have been written off.

Allowance

Write-offs $6,000 $ 90,000 12/31/Yr 1

16,000 Bad debt expense

$100,000 12/31/Yr 2

B. $10,000

C. $16,000

Gleim 2015 | Part 1 | Online MCQs | Unit 002

www.facebook.com/CMA.Arabwebsoft Page 11

حصري | لمنتدى عرب ويب سوفت

D. $26,000

Question: 153 DEF is the consignee for 1,000 units of product X for ABC Company. ABC should recognize the revenue from these 1,000 units when

A. The agreement between DEF and ABC is signed.

B. ABC ships the goods to DEF.

C. DEF receives the goods from ABC.

D. DEF sells the goods and informs ABC of the sale.

Answer (D) is correct. Under a consignment arrangement, the consignor ships goods to the

consignee, who acts as sales agent for the consignor. The goods are in the

physical possession of the consignee but remain the property of the

consignor and are included in the consignor’s inventory. Revenue and the

related cost of goods sold from consigned goods are recognized by the

consignor only when the merchandise is sold and delivered to the final

customer. Accordingly, recognition occurs when notification is received

that the consignee has sold the goods.

Question: 154 An entity had the following account balances in the pre-closing trial balance:

Opening inventory $100,000

Closing inventory 150,000

Purchases 400,000

Transportation-in 6,000

Purchase discounts 40,000

Purchase allowances 15,000

Returned purchases 5,000

The entity had net purchases for the period of

A. $340,000

B. $346,000

Answer (B) is correct. Purchase discounts, allowances, and returns are subtractions from

purchases because they are reductions of cost. Transportation-in is an

addition because it increases cost. Thus, net purchases equals $346,000

($400,000 + $6,000 – $40,000 – $15,000 – $5,000).

C. $370,000

D. $376,000

Gleim 2015 | Part 1 | Online MCQs | Unit 002

www.facebook.com/CMA.Arabwebsoft Page 12

حصري | لمنتدى عرب ويب سوفت

Question: 155 A physical inventory count showed an entity had inventory costing $1,000,000 on hand at December 31, Year 1. Excluded from this amount were the following:

Goods costing $82,000, shipped to a customer free on board (FOB) shipping point

on December 28, Year 1. They were expected to be received by the customer on

January 4, Year 2.

Goods costing $122,000, shipped to a customer free on board (FOB) destination

December 30, Year 1. They were expected to be received by the customer on

January 5, Year 2.

Compute the correct ending inventory to be reported on the shipper’s statement of financial position at December 31, Year 1.

A. $1,000,000

B. $1,082,000

C. $1,122,000

Answer (C) is correct. The goods shipped FOB shipping point should be counted in the buyer’s,

not the seller’s, inventory because title and risk of loss pass at the time

and place of shipment. These goods were properly excluded from ending

inventory. The goods shipped FOB destination were improperly excluded

from the seller’s ending inventory. The title and risk of loss did not pass

until the time and place where the goods reached their destination and

were duly tendered. Thus, the correct ending inventory is $1,122,000

($1,000,000 beginning balance + $122,000 goods shipped FOB

destination).

D. $1,204,000

Question: 156 The following selected data from statements of financial position on December 31, Year 1, and December 31, Year 2, are presented below:

12/31/Year 1 12/31/Year 2

Inventory $120,000 $140,000

Trade accounts payable 62,000 49,000

Additional information for Year 2:

1. Cash payments to suppliers of merchandise were $180,000.

Cost of goods sold in Year 2 was

A. $147,000

Answer (A) is correct. Cost of goods sold equals beginning inventory, plus purchases, minus

ending inventory. To determine cost of goods sold, purchases must be

calculated. Purchases equal $167,000 ($49,000 ending accounts payable

+ $180,000 payments to suppliers – $62,000 beginning accounts

Gleim 2015 | Part 1 | Online MCQs | Unit 002

www.facebook.com/CMA.Arabwebsoft Page 13

حصري | لمنتدى عرب ويب سوفت

payable). Thus, cost of goods sold equals $147,000 ($120,000 beginning

inventory + $167,000 purchases – $140,000 ending inventory).

B. $160,000

C. $167,000

D. $180,000

Question: 157 An entity with a December 31 year end purchased $2,000 of inventory on account. The

seller was responsible for delivery to the shipping point, with freight of $50 paid at

destination by the buyer. The invoice date was December 27, Year 1, and the goods arrived

on January 3, Year 2.

Now assume the terms required the seller to deliver to the destination instead of the shipping point. What is the correct amount of inventory and freight-in relating to this purchase on the Year 1 financial statements?

Inventory Freight-In

A. $0 $0

Answer (A) is correct. Title and risk of loss passed to the buyer at the destination, and the seller

incurred the expense of delivery to that point. The goods did not arrive

until after year end, so they should not be included in Year 1 inventory.

Freight-in should also not be recorded until Year 2.

B. $2,050 $0

C. $0 $50

D. $2,000 $50

Question: 158 A retail entity maintains a markup of 25% based on cost. The entity has the following information for the current year:

Purchases of merchandise $690,000

Freight-in on purchases 25,000

Sales 900,000

Ending inventory 80,000

Beginning inventory was

A. $40,000

B. $85,000

Answer (B) is correct. Cost of goods sold for a period equals beginning inventory, plus

purchases, plus freight-in, minus ending inventory. Given that sales

reflect 125% of cost, cost of goods sold must equal $720,000 ($900,000

Gleim 2015 | Part 1 | Online MCQs | Unit 002

www.facebook.com/CMA.Arabwebsoft Page 14

حصري | لمنتدى عرب ويب سوفت

sales ÷ 1.25). Consequently, the beginning inventory must have been

$85,000 ($720,000 COGS + $80,000 EI – $690,000 purchases – $25,000

freight-in).

C. $110,000

D. $265,000

Fact Pattern: An entity had the following opening and closing inventory balances during the current year:

1/1 12/31

Finished goods $ 90,000 $260,000

Raw materials 105,000 130,000

Work-in-progress 220,000 175,000

The following transactions and events occurred during the current year: $300,000 of raw materials were purchased, of which $20,000 were returned because of defects.

$600,000 of direct labor costs were incurred.

$750,000 of production overhead costs were incurred

Question: 159 If the entity’s raw materials inventory as of December 31 of the current year (ending inventory) was miscounted and the true figure was higher than $130,000, one effect on the year-end financial statements would be that

A. Profit is overstated.

B. Cost of goods sold is overstated.

Answer (B) is correct. If the ending inventory of raw materials is understated, raw materials

used is overstated, cost of goods produced is overstated, and cost of

goods sold is overstated.

C. Working capital is overstated.

D. Cost of goods produced is understated.

Question: 160 The following information is available for an entity for the quarter ended March 31, of the current year:

Merchandise inventory, as of

January 1 of the current year $ 30,000

Sales 200,000

Purchases 190,000

The gross profit margin is normally 20% of sales. What is the estimated cost of the merchandise inventory at March 31, of the current year?

Gleim 2015 | Part 1 | Online MCQs | Unit 002

www.facebook.com/CMA.Arabwebsoft Page 15

حصري | لمنتدى عرب ويب سوفت

A. $20,000

B. $40,000

C. $60,000

Answer (C) is correct. Using the gross profit method, cost of goods sold for the quarter is

estimated to be $160,000 [$200,000 sales × (1.0 – 0.2)]. Goods available

for sale was $220,000 ($30,000 beginning inventory + $190,000

purchases). Estimated ending inventory is therefore $60,000 ($220,000

goods available for sale – $160,000 estimated cost of goods sold).

D. $180,000

Question: 161 An internal auditor performs an analytical procedure to compare the gross margins of various divisional operations with those of other divisions and with the individual division’s performance in previous years. The internal auditor notes a significant increase in the gross margin at one division. The internal auditor does some preliminary investigation and also notes that there were no changes in products, production methods, or divisional management during the year. The most likely cause of the increase in gross margin is

A. An increase in the number of competitors selling similar products.

B. A decrease in the number of suppliers of the material used in

manufacturing the product.

C. An overstatement of year-end inventory.

Answer (C) is correct. An overstatement of year-end inventory results in an understatement of

cost of goods sold, which overstates gross margin.

D. An understatement of year-end accounts receivable.

Question: 162 If certain goods owned by an entity were not recorded as a purchase and were not counted in ending inventory, in error, then

A. Cost of goods sold for the period will be understated.

B. Cost of goods sold for the period will be overstated.

C. Net income for the period will be understated.

D. There will be no effect on cost of goods sold or profit for the period.

Answer (D) is correct. The effects of the errors on cost of goods sold are offsetting. Purchases,

which increase cost of goods sold, and ending inventory, which

decreases cost of goods sold, are understated by the same amount.

Neither cost of goods sold nor net income is affected.

Gleim 2015 | Part 1 | Online MCQs | Unit 002

www.facebook.com/CMA.Arabwebsoft Page 16

حصري | لمنتدى عرب ويب سوفت

Question: 163 What is the cost of ending inventory given the following factors?

Beginning inventory $ 5,000

Total production costs 60,000

Cost of goods sold 55,000

Direct labor 40,000

A. $5,000

B. $10,000

Answer (B) is correct. Beginning inventory, plus purchases (or other inventory additions),

minus cost of goods sold, equals ending inventory. Thus, ending

inventory equals $10,000 ($5,000 + $60,000 – $55,000). Direct labor is

included in total production costs.

C. $45,000

D. $50,000

Question: 164 Holly Company’s inventory is overstated at December 31 of this year. The result will be

A. Understated income this year.

B. Understated retained earnings this year.

C. Understated retained earnings next year.

D. Understated income next year.

Answer (D) is correct. Cost of goods sold equals beginning finished goods, plus cost of goods

manufactured for a manufacturer or purchases for a retailer, minus

ending finished goods. Overstated ending inventory therefore results in

understated cost of goods sold, overstated net income, and overstated

retained earnings in the period of the error. When these errors reverse in

the following period, beginning inventory and cost of goods sold will be

overstated, and net income will be understated. Retained earnings will be

correct.

Question: 165 Which one of the following errors will result in the overstatement of net income?

A. Overstatement of beginning inventory.

B. Overstatement of ending inventory.

Answer (B) is correct. Cost of goods sold equals beginning finished goods, plus cost of goods

manufactured for a manufacturer or purchases for a retailer, minus

ending finished goods. Overstated ending inventory therefore results in

understated cost of goods sold, overstated net income, and overstated

Gleim 2015 | Part 1 | Online MCQs | Unit 002

www.facebook.com/CMA.Arabwebsoft Page 17

حصري | لمنتدى عرب ويب سوفت

retained earnings in the period of the error.

C. Overstatement of goodwill amortization.

D. Overstatement of bad debt expense.

Question: 166 The following information applies to the income statement of Addison Company:

Gross sales $1,000,000

Net sales 900,000

Freight-in 10,000

Ending inventory 200,000

Gross profit margin 40%

Addison’s cost of goods available for sale is

A. $550,000

B. $560,000

C. $740,000

Answer (C) is correct. The gross profit (gross margin) method calculates ending inventory at a

given time by subtracting an estimated cost of goods sold from the sum

of beginning inventory and purchases (or cost of goods manufactured).

The estimated cost of goods sold equals sales minus the gross profit. The

gross profit equals sales multiplied by the gross profit percentage, an

amount ordinarily determined on a historical basis. Given that the gross

margin percentage is 40% of net sales, cost of goods sold must be 60% of

net sales, or $540,000 ($900,000 × 60%). Goods available for sale equals

cost of goods sold plus ending inventory ($540,000 + $200,000 =

$740,000).

D. $800,000

Question: 167 An entity started in Year 1 with 200 scented candles on hand at a cost of $3.50 each. These candles sell for $7.00 each. The following schedule represents the purchases and sales of candles during Year 1:

Transaction Quantity Unit Quantity

Number Purchased Cost Sold

1 --- --- 150

2 250 $3.30 ---

3 --- --- 100

Gleim 2015 | Part 1 | Online MCQs | Unit 002

www.facebook.com/CMA.Arabwebsoft Page 18

حصري | لمنتدى عرب ويب سوفت

4 200 3.10 ---

5 --- --- 200

6 350 3.00 ---

7 --- --- 300

If the entity uses periodic FIFO inventory pricing, the gross profit for Year 1 would be

A. $2,755

B. $2,805

Answer (B) is correct. The FIFO method assumes that the first goods purchased are the first

goods sold and that ending inventory consists of the latest purchases.

Moreover, whether the inventory system is periodic or perpetual does not

affect FIFO measurement. The cost of goods sold is $2,445 {beginning

inventory (200 units × $3.50) + purchases [(250 units × $3.30) + (200

units × $3.10) + (350 units × $3.00)] – ending inventory (250 units ×

$3.00)}. Thus, the gross profit for Year 1 using FIFO is $2,805 [sales

(750 units × $7.00) – cost of goods sold of $2,445].

C. $2,854

D. $2,920

Question: 168 The cost of materials has risen steadily over the year. Which of the following methods of estimating the ending balance of the materials inventory account will result in the highest profit, assuming all other variables remain constant?

A. Last-in, first-out (LIFO).

B. First-in, first-out (FIFO).

Answer (B) is correct. Profit will be higher when cost of goods sold is lower, other factors held

constant. Cost of goods sold equals beginning inventory, plus purchases,

minus ending inventory. Accordingly, cost of goods sold will be lowest

when the ending inventory is highest. In an inflationary environment,

ending inventory is highest under FIFO because the older, less expensive

items are deemed to have been sold, leaving the more expensive items in

the ending inventory.

C. Weighted average.

D. Specific identification.

Question: 169 When a right of return exists, an entity may recognize revenue from a sale of goods at the time of sale only if

A. The amount of future returns can be reliably estimated.

Gleim 2015 | Part 1 | Online MCQs | Unit 002

www.facebook.com/CMA.Arabwebsoft Page 19

حصري | لمنتدى عرب ويب سوفت

Answer (A) is correct. One condition for recognition of revenue from the sale of goods is the

transfer of the significant risks and rewards of ownership. Retention of

significant risk may occur when, for example, the buyer may rescind the

purchase for a reason stipulated in the contract, and the buyer is uncertain

about the probability of return. However, if the entity can reliably

estimate future returns and recognizes a liability for returns based on

experience and other pertinent information, revenue may be recognized

at the time of sale if the other conditions for revenue recognition also are

met.

B. The seller retains the risks and rewards of ownership.

C. The buyer resells the goods.

D. The seller believes returns will not be material.

Fact Pattern: Illustrated below is a perpetual inventory card for the current year. Date Units Purchased Units Sold Units Balance

January 1 0

January 12 1,000 @ $2.00 1,000

March 15 300 700

May 5 500 @ $2.20 1,200

July 8 500 700

November 24 1,000 @ $1.65 1,700

Additional information: The entity had no opening inventory.

The items sold on March 15 were purchased on January 12.

The items sold on July 8 were purchased on May 5.

Question: 170 The ending inventory balance under the first-in, first-out (FIFO) method of inventory valuation is

A. $3,050

B. $3,150

Answer (B) is correct. Under the FIFO method, the 1,700 units of ending inventory are valued

at the most recent prices. Ending inventory is assumed to include 1,000

units purchased November 24, 500 units purchased May 5, and 200 units

purchased January 12. Hence, the ending inventory is $3,150 [(1,000 ×

$1.65) + (500 × $2.20) + (200 × $2.00)].

C. $3,230

D. $3,430

Gleim 2015 | Part 1 | Online MCQs | Unit 002

www.facebook.com/CMA.Arabwebsoft Page 21

حصري | لمنتدى عرب ويب سوفت

Fact Pattern: Illustrated below is a perpetual inventory card for the current year. Date Units Purchased Units Sold Units Balance

January 1 0

January 12 1,000 @ $2.00 1,000

March 15 300 700

May 5 500 @ $2.20 1,200

July 8 500 700

November 24 1,000 @ $1.65 1,700

Additional information: The entity had no opening inventory.

The items sold on March 15 were purchased on January 12.

The items sold on July 8 were purchased on May 5.

Question: 171 The cost of goods sold under the specific identification method of inventory valuation is

A. $1,320

B. $1,520

C. $1,600

D. $1,700

Answer (D) is correct. Of the 800 units sold during the period, the 300 units sold on March 15

were purchased on January 12 at a cost of $2.00 per unit. The remaining

500 units were purchased on May 5 at a cost of $2.20 per unit. The cost

of goods sold under the specific identification method is therefore $1,700

[(300 units × $2.00) + (500 units × $2.20)].

Fact Pattern: Illustrated below is a perpetual inventory card for the current year. Date Units Purchased Units Sold Units Balance

January 1 0

January 12 1,000 @ $2.00 1,000

March 15 300 700

May 5 500 @ $2.20 1,200

July 8 500 700

November 24 1,000 @ $1.65 1,700

Additional information: The entity had no opening inventory.

The items sold on March 15 were purchased on January 12.

The items sold on July 8 were purchased on May 5.

Gleim 2015 | Part 1 | Online MCQs | Unit 002

www.facebook.com/CMA.Arabwebsoft Page 21

حصري | لمنتدى عرب ويب سوفت

Question: 172 A merchandising company had the following inventory related transactions in its first year of operations:

Purchases Sales in Balance

Date in Units Units in Units

Jan. 1 10,000 @ $5 10,000

March 1 6,000 @ $6 16,000

May 1 3,000 13,000

July 1 8,000 @ $6.25 21,000

Sept. 1 12,000 9,000

Nov. 1 5,000 @ $7 14,000

Dec. 1 2,000 12,000

If the company uses the first-in-first-out (FIFO) method of inventory valuation, its ending inventory balance (rounded) will be

A. $62,000

B. $70,759

C. $78,750

Answer (C) is correct. The first-in-first-out (FIFO) method treats the oldest units as being sold

first and the newest units remain in inventory. Because the company has

12,000 units remaining, ending inventory equals $78,750 [(5,000 × $7) +

(7,000 × $6.25)].

D. $84,000

Question: 173 An entity has 8,000 units in inventory on January 1, valued at 10 per unit. During the year, the entity sold 25,000 units and purchased inventory as follows:

Quantity

Date Purchased Unit Price

April 1 15,000 units $ 8

July 1 10,000 units 9

October 1 12,500 units 10

If the entity uses the weighted-average method of inventory valuation, cost of goods sold for the period will be

A. $186,978

B. $197,000

Gleim 2015 | Part 1 | Online MCQs | Unit 002

www.facebook.com/CMA.Arabwebsoft Page 22

حصري | لمنتدى عرب ويب سوفت

C. $228,023

Answer (C) is correct. Under the weighted-average method, the weighted-average cost per unit

is multiplied by the number of units sold to determine the cost of goods

sold for the period. The total units available for sale equaled 45,500

(8,000 + 15,000 + 10,000 + 12,500). The total cost of all units available

for sale was $415,000 [(8,000 × $10) + (15,000 × $8) + (10,000 × $9) +

(12,500 × $10)]. Thus, the weighted-average cost per unit of inventory

was $9.1209 ($415,000 ÷ 45,500), and cost of goods sold was $228,023

(25,000 × $9.1209).

D. $235,000

Question: 174 On January 1, a company has no opening inventory balance. The following purchases are made during the year:

Units Unit

Purchased Cost

January 1 5,000 $10.00

April 1 5,000 9.00

July 1 5,000 8.00

October 1 5,000 7.50

There are 10,000 units in inventory on December 31.

If the company uses the first-in, first-out (FIFO) method of inventory valuation, the ending

inventory balance will be

A. $77,500

Answer (A) is correct. Under first-in, first-out (FIFO) inventory valuation, the 10,000 units in

ending inventory are assumed to have been the most recent items

purchased. The cost of the most recent 10,000 units purchased is: (5,000

units × $7.50) + (5,000 units × $8) = $77,500.

B. $85,000

C. $86,250

D. $95,000

Gleim 2015 | Part 1 | Online MCQs | Unit 002

www.facebook.com/CMA.Arabwebsoft Page 23

حصري | لمنتدى عرب ويب سوفت

Question: 175 Which inventory pricing method generally approximates current cost for each of the following?

Ending Cost of Inventory Goods Sold

A. FIFO FIFO

B. LIFO FIFO

C. FIFO LIFO

Answer (C) is correct. FIFO assigns the most recent acquisition costs to ending inventory and

the earliest acquisition costs to cost of goods sold. LIFO assigns the

earliest acquisition costs to ending inventory (it is permitted by

U.S. GAAP but not by IFRS). Thus, FIFO approximates current cost for

ending inventory, and LIFO approximates current cost of goods sold.

D. LIFO LIFO

Question: 176 Which of the following changes in accounting policies resulting from a significant change in the expected pattern of economic benefit will increase profit?

A. A change from FIFO to LIFO inventory valuation when costs are rising.

B. A change from FIFO to weighted-average inventory valuation when costs

are falling.

Answer (B) is correct. In a period of falling costs, FIFO results in higher cost of goods sold than

the weighted-average method. FIFO includes the higher, earlier costs in

cost of goods sold, and the weighted-average method averages the later,

lower costs with the higher, earlier costs. Thus, a change from FIFO to

weighted-average costing reduces cost of goods sold and increases

reported profit.

C. A change from accelerated to straight-line depreciation in the later years of

the depreciable lives of the assets.

D. A change from straight-line to accelerated depreciation in the early years of

the depreciable lives of the assets.

Question: 177 On January 1, a company has no opening inventory balance. The following purchases are made during the year:

Units Unit

Purchased Cost

January 1 5,000 $10.00

April 1 5,000 9.00

July 1 5,000 8.00

October 1 5,000 7.50

There are 10,000 units in inventory on December 31.

Gleim 2015 | Part 1 | Online MCQs | Unit 002

www.facebook.com/CMA.Arabwebsoft Page 24

حصري | لمنتدى عرب ويب سوفت

If the company uses the last-in, first-out (LIFO) method of inventory valuation, cost of goods sold for the year will be:

A. $77,500

Answer (A) is correct. A total of 20,000 units was available for sale (0 beginning inventory +

20,000 purchased during year). Since 10,000 remain in ending inventory,

10,000 were sold (20,000 available – 10,000 remaining). Under the LIFO

method, the units sold were those purchased in July and October. Cost of

goods sold for the year thus equaled $77,500 [(5,000 units x $8.00 July)

+ (5,000 units x $7.50 October)].

B. $86,250

C. $87,500

D. $95,000

Question: 178 The advantage of the last-in, first-out inventory method is based on the assumption that

A. The most recently incurred costs should be allocated to the cost of goods

sold.

Answer (A) is correct. Under the LIFO method, the most recent costs of acquiring or producing

inventory are expensed as part of cost of goods sold. Given inflation, this

method results in the highest cost of goods.

B. Costs should be charged to revenue in the order in which they are incurred.

C. Costs should be charged to cost of goods sold at average cost.

D. Current costs should be based on representative or normal conditions of

efficiency and volume of operations.

Question: 179 Which inventory cost flow method is prohibited according to IFRS?

A. First-in, first-out (FIFO) method.

B. Specific identification method.

C. Weighted average cost method.

D. Last-in, first-out (LIFO) method.

Answer (D) is correct. The last-in, first-out (LIFO) method is prohibited by IFRS. This method

is based on the assumption that the newest items are sold first. Its effect

is to include current prices in cost of goods sold. But the LIFO

assumption ordinarily does not match actual inventory use.

Question: 180 The inventory method yielding the same inventory measurement and cost of goods sold whether a perpetual or periodic system is used is

A. Average cost.

B. First-in, first-out.

Gleim 2015 | Part 1 | Online MCQs | Unit 002

www.facebook.com/CMA.Arabwebsoft Page 25

حصري | لمنتدى عرب ويب سوفت

Answer (B) is correct. A perpetual inventory system will result in the same dollar amount of

ending inventory as a periodic inventory system assuming a FIFO cost

flow. Under both perpetual and periodic systems, the same units are

deemed to be in ending inventory.

C. Last-in, first-out.

D. Either first-in, first-out or last-in, first-out.

Question: 181 In a period of rising prices, which one of the following inventory methods usually provides the best matching of expenses against revenues?

A. Weighted average.

B. First-in, first-out.

C. Last-in, first-out.

Answer (C) is correct. A significant advantage of the LIFO method is its matching of current

revenues with the most recent product costs. When prices are rising

(which is most of the time), the most recent costs are the highest costs,

resulting in higher cost of goods sold and lower net income. The lower

net income means lower taxes.

D. Specific identification.

Question: 182 Which one of the following actions would result in a decrease in income?

A. Liquidating last-in, first-out layers of inventory when prices have been

increasing.

B. Changing from first-in, first-out to last-in, first-out inventory method when

prices are decreasing.

C. Accelerating purchases at the end of the year when using last-in, first-out

inventory method in times of rising prices.

Answer (C) is correct. Under the LIFO method, the most recent costs of acquiring or producing

inventory are expensed as part of cost of goods sold. In a time of rising

prices, charging newer, higher-priced goods against current revenues

decreases income.

D. Changing the number of last-in, first-out pools.

Question: 183 In periods of rising costs, which one of the following inventory cash flow assumptions will result in higher cost of sales?

A. First-in, first-out.

B. Last-in, first-out.

Gleim 2015 | Part 1 | Online MCQs | Unit 002

www.facebook.com/CMA.Arabwebsoft Page 26

حصري | لمنتدى عرب ويب سوفت

Answer (B) is correct. A significant advantage of the LIFO method is its matching of current

revenues with the most recent product costs. When prices are rising

(which is most of the time), the most recent costs are the highest costs,

resulting in higher cost of goods sold and lower net income. The lower

net income means lower taxes.

C. Weighted average.

D. Moving average.

Fact Pattern: During January, Metro Co., which maintains a perpetual inventory system, recorded the following information pertaining to its inventory:

Units

Unit Total On

Units

Cost

Cost

Hand

Balance on 1/1 1,000 $1 $1,000 1,000

Purchased on 1/7 600 3 1,800 1,600

Sold on 1/20 900 700

Purchased on 1/25 400 5 2,000 1,100

Question: 184 Under the moving-average method, what amount should Metro report as inventory at January 31?

A. $2,640

B. $3,225

Answer (B) is correct. The moving-average system is only applicable to perpetual inventories. It

requires that a new weighted average be computed after every purchase.

This moving average is based on remaining inventory held and the new

inventory purchased. Based on the calculations below, the moving-

average cost per unit for the 1/20 sale is $1.75, and the cost of goods sold

(COGS) for January is $1,575 (900 units sold × $1.75). Thus, ending

inventory is $3,225 ($1,000 beginning balance + $1,800 purchase on 1/7

– $1,575 COGS on 1/20 + $2,000 purchase on 1/25).

Moving-Average

Units Cost/Unit Total Cost

Balance 1/1 1,000 $1.00 $1,000

Purchase 1/7 600 3.00 1,800

Gleim 2015 | Part 1 | Online MCQs | Unit 002

www.facebook.com/CMA.Arabwebsoft Page 27

حصري | لمنتدى عرب ويب سوفت

1,600 $1.75 $2,800

C. $3,300

D. $3,900

Question: 185 The weighted average for the year inventory cost flow method is applicable to which of the following inventory systems?

Periodic Perpetual

A. Yes Yes

B. Yes No

Answer (B) is correct. The weighted-average method determines an average cost only once (at

the end of the period) and is therefore applicable only to a periodic

system. In contrast, the moving-average method requires determination

of a new weighted-average cost after each purchase and thus applies only

to a perpetual system.

C. No Yes

D. No No

Question: 186 In accounting for inventories, generally accepted accounting principles require departure from the historical cost principle when the utility of inventory has fallen below cost. This rule is known as the “lower-of-cost-or-market” rule. The term “market” as defined here means

A. Original cost minus allowance for obsolescence.

B. Original cost plus normal profit margin.

C. Replacement cost of the inventory.

Answer (C) is correct. Market is the replacement cost of the inventory as determined in the

market in which the entity buys its inventory, not the market in which it

sells to customers. Market is limited to a ceiling amount equal to net

realizable value and a floor amount equal to net realizable value minus a

normal profit margin.

D. Original cost minus cost to dispose.

Fact Pattern: The data below concern items in Stockholm Co.’s inventory.

Per Unit Gear Stuff Wickets

Historical cost $190.00 $106.00 $53.00

Selling price 217.00 145.00 73.75

Cost to complete and sell 19.00 8.00 2.50

Gleim 2015 | Part 1 | Online MCQs | Unit 002

www.facebook.com/CMA.Arabwebsoft Page 28

حصري | لمنتدى عرب ويب سوفت

Current replacement cost 203.00 105.00 51.00

Normal profit margin 32.00 29.00 21.25

Question: 187 Under IFRS, what amount should Stockholm Co. compare with historical cost to measure the amount at which the wickets should be measured?

A. $51.00

B. $50.00

C. $71.25

Answer (C) is correct. Inventory is recorded at cost. However, inventory must be written down

if its utility is no longer as great as its cost. To make this determination

under IFRS, historical cost is compared with the inventory’s net

realizable value (NRV). NRV is the estimated selling price in the

ordinary course of business minus estimated costs of completion and

sale. NRV for the wickets is $71.25 ($73.75 selling price – $2.50

estimated costs of completion and sale).

D. $73.75

Question: 188 An entity that prepares its financial statements using IFRS reported the following selected per-unit data relating to work-in-process:

Selling price $100

Completion costs 10

Historical cost 91

Replacement cost 108

Normal gross profit 20

Selling cost 5

In comparison with historical cost, what will be the per-unit effect on gross profit of measuring ending inventory?

A. No effect.

B. Reduction of $6.

Answer (B) is correct. Under IFRS, inventories are measured subsequent to acquisition at the

lower of cost or net realizable value (NRV). NRV equals selling price

minus estimated completion and selling costs. Given that historical cost

is $91 and NRV is $85 ($100 selling price – $10 completion cost – $5

selling cost), the effect on per-unit gross profit is a reduction of $6. This

amount is expensed.

C. Reduction of $26.

Gleim 2015 | Part 1 | Online MCQs | Unit 002

www.facebook.com/CMA.Arabwebsoft Page 29

حصري | لمنتدى عرب ويب سوفت

D. Increase of $5.

Question: 189 The following data apply to a unit of inventory:

Selling price $22

Selling cost 2

Normal profit margin 5

Replacement cost 10

Using the lower of cost or market (LCM) method of measuring inventory, what is the market amount for this unit of inventory?

A. $10.00

B. $15.00

Answer (B) is correct. Under the LCM method, market is current replacement cost subject to a

maximum (ceiling) equal to net realizable value and a minimum (floor)

equal to net realizable value minus a normal profit margin. NRV equals

selling price minus costs of completion and disposal. Thus, the maximum

market amount is the $20 NRV ($22 selling price – $2 selling cost), and

the minimum is $15 ($20 NRV – $5 normal profit margin). Because the

minimum exceeds the $10 replacement cost, it is the market amount.

C. $17.50

D. $20.00

Question: 190 Based on a physical inventory taken on December 31, Chewy Co. determined its chocolate inventory on a FIFO basis at $26,000 with a replacement cost of $20,000. Chewy estimated that, after further processing costs of $12,000, the chocolate could be sold as finished candy bars for $40,000. Chewy’s normal profit margin is 10% of sales. Under the lower-of-cost-or-market rule, what amount should Chewy report as chocolate inventory in its December 31 balance sheet?

A. $28,000

B. $26,000

C. $24,000

Answer (C) is correct. Market equals current replacement cost subject to maximum and

minimum values. The maximum is NRV, and the minimum is NRV

minus normal profit. When replacement cost is within this range, it is

used as market. Cost is given as $26,000. NRV is $28,000 ($40,000

selling price – $12,000 additional processing costs), and NRV minus a

normal profit equals $24,000 [$28,000 – ($40,000 × 10%)]. Because the

lowest amount in the range ($24,000) exceeds replacement cost

($20,000), it is used as market. Because market value ($24,000) is less

Gleim 2015 | Part 1 | Online MCQs | Unit 002

www.facebook.com/CMA.Arabwebsoft Page 31

حصري | لمنتدى عرب ويب سوفت

than cost ($26,000), it is also the inventory amount.

D. $20,000

Question: 191 The lower-of-cost-or-market rule for inventories may be applied to total inventory, to groups of similar items, or to each item. Which application generally results in the lowest inventory amount?

A. All applications result in the same amount.

B. Total inventory.

C. Groups of similar items.

D. Separately to each item.

Answer (D) is correct. Applying the LCM rule to each item of inventory produces the lowest

amount for each item and therefore the lowest and most conservative

measurement for the total inventory. The reason is that aggregating items

results in the inclusion of some items at amounts greater than LCM. For

example, if item A (cost $2, market $1) and item B (cost $3, market $4)

are aggregated for LCM purposes, the inventory measurement is $5. If

the rule is applied separately to A and B, the LCM measurement is $4.

Question: 192 Under the lower-of-cost-or-market method, the replacement cost of an inventory item would be used as the designated market value

A. When it is below the net realizable value less the normal profit margin.

B. When it is below the net realizable value and above the net realizable value

less the normal profit margin.

Answer (B) is correct. Market is current replacement cost subject to maximum and minimum

values. The maximum is net realizable value, and the minimum is net

realizable value less normal profit. When replacement cost is within this

range, it is used as the market amount.

C. When it is above the net realizable value.

D. Regardless of net realizable value.

Question: 193 Ward Distribution Company has determined its December 31 inventory on a FIFO basis at $200,000. Information pertaining to that inventory follows:

Estimated selling price $204,000

Estimated cost of disposal 10,000

Normal profit margin 30,000

Current replacement cost 180,000

Ward records losses that result from applying the lower-of-cost-or-market rule. At December

Gleim 2015 | Part 1 | Online MCQs | Unit 002

www.facebook.com/CMA.Arabwebsoft Page 31

حصري | لمنتدى عرب ويب سوفت

31, the loss that Ward should recognize is

A. $0

B. $6,000

C. $14,000

D. $20,000

Answer (D) is correct. As indicated below, the $180,000 replacement cost falls between the

$194,000 ceiling and the $164,000 floor. Hence, it will be used as market

in the LCM determination. Because the $180,000 market value is

$20,000 lower than the $200,000 historical cost, the inventory should be

valued at $180,000 and a $20,000 loss recognized.

NRV ($204,000 – $10,000) $194,000

Replacement cost $180,000

NRV – Normal profit ($194,000 – $30,000) $164,000

Question: 194 The replacement cost of an inventory item is below the net realizable value and above the net realizable value less the normal profit margin. The original cost of the inventory item is below the net realizable value less the normal profit margin. Under the lower-of-cost-or-market (LCM) method, the inventory item should be valued at

A. Net realizable value.

B. Net realizable value less the normal profit margin.

C. Original cost.

Answer (C) is correct. When replacement cost is below the NRV and above the NRV less the

normal profit margin, market equals replacement cost. Given that the

original cost of the inventory item is below market, the original cost

should be used to measure the inventory item under the LCM method.

D. Replacement cost.

Question: 195 Lorraine Co. has determined its fiscal year-end inventory on a FIFO basis to be $400,000. Information pertaining to that inventory follows:

Estimated selling price $408,000

Estimated cost of disposal 20,000

Normal profit margin 60,000

Current replacement cost 360,000

Lorraine records losses that result from applying the lower-of-cost-or-market (LCM) rule. At its year end, what should be the net carrying value of Lorraine’s inventory?

A. $400,000

Gleim 2015 | Part 1 | Online MCQs | Unit 002

www.facebook.com/CMA.Arabwebsoft Page 32

حصري | لمنتدى عرب ويب سوفت

B. $388,000

C. $360,000

Answer (C) is correct. Under the LCM method, market is current replacement cost subject to a

maximum (ceiling) equal to net realizable value and a minimum (floor)

equal to net realizable value minus a normal profit. NRV equals selling

price minus costs of completion and disposal. Here, original cost is

$400,000 and replacement cost is $360,000. The LCM method uses the

lower of the two, $360,000, to measure inventory. However, the

inventory measure cannot exceed the NRV of $388,000 ($408,000

selling price – $20,000 cost of disposal). Furthermore, the inventory

carrying amount cannot be lower than NRV minus normal profit, or

$328,000 ($388,000 NRV – $60,000 normal profit). Because the lower

of cost or market ($360,000) is between $388,000 (ceiling) and $328,000

(floor), the net carrying amount is $360,000.

D. $328,000

Question: 196 Which of the following is true regarding inventory adjustments under IFRS?

A. IFRS do not require inventory adjustments.

B. Reversals of adjustments are allowed in a subsequent period.

Answer (B) is correct. Both IFRS and U.S. GAAP require the cost of inventory to be written

down if the utility of the goods is impaired. Under IFRS, inventories are

measured subsequent to initial recognition at the lower of cost and net

realizable value (NRV), with NRV assessed each period. Moreover,

unlike U.S. GAAP, IFRS permit inventory to be written up to the lower

of cost and NRV if previously written down. The reversal is permissible

only to the extent of the prior write-down.

C. A reversal of a previous write-down may be higher than the previous write-

down.

D. Adjustments may not be reversed in a subsequent period.

Question: 197 A company determined the following information for its inventory at the end of an interim period on June 30, Year 2:

Historical cost $80,000

Net realizable value (NRV) 77,000

Current replacement cost 76,000

Normal profit margin 2,000

The company expects that on December 31, Year 2, the inventory’s NRV reduced by a normal profit margin will be at least $81,000. What amount of inventory should the company report in its interim financial statements under IFRS and under U.S. GAAP on June 30, Year

Gleim 2015 | Part 1 | Online MCQs | Unit 002

www.facebook.com/CMA.Arabwebsoft Page 33

حصري | لمنتدى عرب ويب سوفت

2?

IFRS

U.S. GAAP

A. $77,000 $80,000

Answer (A) is correct. Under U.S. GAAP, inventory is reported at its historical cost of $80,000

because no write-down of inventory is reasonably anticipated for the

year. Under IFRS, the inventory is measured at the lower of cost

($80,000) and NRV ($77,000) for each interim reporting period. Whether

a market decline is expected to be reversed by the end of the annual

period is not considered. Thus, the inventory is not reported at its NRV

of $77,000.

B. $77,000 $76,000

C. $80,000 $80,000

D. $80,000 $81,000

Question: 198 Investments classified as held-to-maturity are measured at

A. Fair value, with unrealized gains and losses reported in net income.

B. Fair value, with unrealized gains and losses reported in other

comprehensive income (OCI).

C. Replacement cost, with no unrealized gains or losses reported.

D. Amortized cost, with no unrealized gains or losses reported.

Answer (D) is correct. Assuming the fair value option has not been elected, held-to-maturity

securities are reported at amortized cost, with no unrealized gains or

losses reported.

Question: 199 King Company has the following investment portfolio:

Cost

Fair Value

Trading securities:

Quill Company common stock $140,000 $150,000

Barton, Inc., common stock 125,000 110,000

Delta, Inc., 8% bonds 225,000 240,000

Securities to be held to maturity:

Armand, Inc., 9% bonds 80,000 84,000

Port City municipal bonds 180,000 210,000

Gleim 2015 | Part 1 | Online MCQs | Unit 002

www.facebook.com/CMA.Arabwebsoft Page 34

حصري | لمنتدى عرب ويب سوفت

Available-for-sale securities:

Knox Co. common stock 45,000 51,000

Vernon, Inc., preferred stock 97,000 109,000

The total amount of these investments to be reported on King’s statement of financial position is

A. $892,000

B. $902,000

C. $920,000

Answer (C) is correct. Trading securities and available-for-sale securities are reported at their

fair values at each balance sheet date. Held-to-maturity securities are

reported at amortized cost. The total amount of these investments to be

reported on King’s statement of financial position is therefore calculated

as follows:

Trading securities:

Quill Company common stock $150,000

Barton, Inc., common stock 110,000

Delta, Inc., 8% bonds 240,000

Securities to be held to maturity:

Armand, Inc., 9% bonds 80,000

Port City municipal bonds 180,000

Available-for-sale securities:

Knox Co. common stock 51,000

Vernon, Inc., preferred stock 109,000

Total $920,000

D. $954,000

Question: 200 Which one of the following statements with regard to marketable securities is incorrect?

A. In the trading portfolio of marketable equity securities, unrealized gains

and losses are recorded on the income statement.

B. In the available-for-sale portfolio of marketable equity securities,

unrealized gains and losses are recorded on the income statement.

Gleim 2015 | Part 1 | Online MCQs | Unit 002

www.facebook.com/CMA.Arabwebsoft Page 35

حصري | لمنتدى عرب ويب سوفت

Answer (B) is correct. Assuming the fair value option has not been elected, unrealized holding

gains and losses on available-for-sale securities are reported in other

comprehensive income. (But all or part of unrealized holding gains and

losses on available-for-sale securities designated and qualifying as

hedged items in a fair value hedge are recognized in earnings.)

C. The held-to-maturity portfolio consists only of debt securities.

D. Securities may be transferred from the held-to-maturity to the available-for-

sale portfolio.

Question: 201 Securities held primarily for sale in the near term to generate income on short-term price differences are known as

A. Available-for-sale securities.

B. Equity securities.

C. Held-to-maturity securities.

D. Trading securities.

Answer (D) is correct. Trading securities are bought and held primarily for sale in the near term.

They are purchased and sold frequently. They are initially recorded at

cost but are remeasured at fair value at each balance sheet date, with the

unrealized holding gains or losses recognized in earnings.

Question: 202 Unrealized gains and losses on trading securities should be presented in the

A. Statement of financial position.

B. Income statement.

Answer (B) is correct. Unrealized holding gains and losses on trading securities are included in

earnings and are therefore reported in the income statement.

C. Notes to the financial statements.

D. Statement of retained earnings.

Question: 203 Vanity Corporation holds investments in equity securities. These investments were acquired last year and have been properly classified as available-for-sale (AFS) securities. During the current year, the company sold some of the AFS securities at a loss. At year end, the remaining portfolio of AFS securities had appreciated in total value compared with the value at the end of last year. Based on these facts, which one of the following should Vanity report in its financial statements at the end of the current year?

Income Statement

Balance sheet

A. Unrealized loss on sale of

AFS securities

Unrealized holding gain on appreciation

of AFS securities

B. Realized loss on sale of Unrealized holding gain on appreciation

Gleim 2015 | Part 1 | Online MCQs | Unit 002

www.facebook.com/CMA.Arabwebsoft Page 36

حصري | لمنتدى عرب ويب سوفت

AFS securities of AFS securities

Answer (B) is correct. Realized losses on the sale of available-for-sale securities are included in

the calculation of current period earnings. However, unrealized holding

gains and losses on available-for-sale securities, including those

classified as current assets, are excluded from earnings and are reported

as components of other comprehensive income.

C. Unrealized holding gain on

appreciation of AFS securities

Unrealized loss on sale of

AFS securities.

D. Realized loss on sale of AFS securities and

unrealized holding gain on appreciation of

AFS securities

Unrealized holding

gains/losses not

reported he

Question: 204 Kale Co. purchased bonds at a discount on the open market as an investment and has the intent and ability to hold these bonds to maturity. Absent an election of the fair value option, Kale should account for these bonds at

A. Cost.

B. Amortized cost.

Answer (B) is correct. Absent an election of the fair value option, investments in debt securities

must be classified asheld-to-maturity and measured at amortized cost in

the balance sheet if the reporting entity has the positive intent and ability

to hold them to maturity.

C. Fair value.

D. Lower of cost or market.

Question: 205 At year end, Rim Co. held several investments with the intent of selling them in the near term. The investments consisted of $100,000, 8%, 5-year bonds, purchased for $92,000, and equity securities purchased for $35,000. At year end, the bonds were selling on the open market for $105,000, and the equity securities had a market value of $50,000. What amount should Rim report as trading securities in its year-end balance sheet?

A. $50,000

B. $127,000

C. $142,000

D. $155,000

Answer (D) is correct. Trading securities are debt securities not classified as held to maturity

and equity securities with readily determinable fair values that are bought

and held primarily for sale in the near term. Hence, the bonds and the

equity securities are trading securities. They are initially recorded at cost

but are subsequently measured at fair value at each balance sheet date.

Quoted market prices in active markets are the best evidence of fair

Gleim 2015 | Part 1 | Online MCQs | Unit 002

www.facebook.com/CMA.Arabwebsoft Page 37

حصري | لمنتدى عرب ويب سوفت

value. Based on market quotes at year end, the bonds had a fair value of

$105,000, and the equity securities had a fair value of $50,000. The total

is $155,000.

Question: 206 An entity should report the marketable equity securities that it has classified as trading at

A. Lower of cost or market, with holding gains and losses included in

earnings.

B. Lower of cost or market, with holding gains included in earnings only to

the extent of previously recognized holding losses.

C. Fair value, with holding gains included in earnings only to the extent of

previously recognized holding losses.

D. Fair value, with holding gains and losses included in earnings.

Answer (D) is correct. Trading securities are those held principally for sale in the near term.

They are classified as current and consist of debt securities and equity

securities with readily determinable fair values. Unrealized holding gains

and losses on trading securities are reported in earnings. On a statement

of financial position, these securities are reported at fair value.

Question: 207 On December 31, Ott Co. had investments in trading securities as follows:

Fair

Cost Value

Man Co. $10,000 $ 8,000

Kemo, Inc. 9,000 11,000

Fenn Corp. 11,000 9,000

$30,000 $28,000

Ott’s December 31 balance sheet should report the trading securities as

A. $26,000

B. $28,000

Answer (B) is correct. Trading securities are reported at fair value, and unrealized holding gains

and losses are included in earnings. Consequently, the securities should

be reported as $28,000.

C. $29,000

D. $30,000

Gleim 2015 | Part 1 | Online MCQs | Unit 002

www.facebook.com/CMA.Arabwebsoft Page 38

حصري | لمنتدى عرب ويب سوفت

Question: 208 Nola Co. has a portfolio of marketable equity securities that it does not intend to sell in the near term. How should Nola classify these securities, and how should it report unrealized gains and losses from these securities?

Classify as Report in

A. Trading securities A component of income from continuing operations

B. Available-for-sale securities Other comprehensive income (OCI)

Answer (B) is correct. Marketable equity securities may be classified as either trading or

available-for-sale (assuming no election of the fair value option). Equity

securities that are not expected to be sold in the near term should be

classified as available-for-sale. These securities should be reported at fair

value, with unrealized holding gains and losses (except those on

securities designated as being hedged in a fair value hedge) excluded

from earnings and reported in OCI.

C. Trading securities Other comprehensive income (OCI)

D. Available-for-sale

securities

A component of income from continuing

operations

Question: 209 The following information pertains to Lark Corp.’s available-for-sale securities:

December 31

Year 2 Year 3

Cost $100,000 $100,000

Fair value 90,000 120,000

Differences between cost and fair values are considered to be temporary. The decline in fair value was properly accounted for at December 31, Year 2. Ignoring tax effects, by what amount should other comprehensive income (OCI) be credited at December 31, Year 3?

A. $0

B. $10,000

C. $20,000

D. $30,000

Answer (D) is correct. Unrealized holding gains and losses on available-for-sale securities,

including those classified as current assets, are not included in earnings

but ordinarily are reported in OCI, net of tax effects (ignored in this

question). At December 31, Year 2 (assuming the securities are not

designated as being hedged in a fair value hedge), OCI should have been

debited for $10,000 for the excess of cost over fair value to reflect an

unrealized holding loss. At December 31, Year 3, OCI should be credited

to reflect a $30,000 unrealized holding gain ($120,000 fair value at

Gleim 2015 | Part 1 | Online MCQs | Unit 002

www.facebook.com/CMA.Arabwebsoft Page 39

حصري | لمنتدى عرب ويب سوفت

12/31/Year 3 – $90,000 fair value at 12/31/Year 2).

Question: 210 The following information was extracted from Gil Co.’s December 31 balance sheet:

Debit Balance

Noncurrent assets:

Available-for-sale securities (carried at fair value) $96,450

Equity:

Accumulated other comprehensive income (OCI)

Unrealized gains and losses on available-for-sale securities 19,800 Historical cost of the available-for-sale securities was

A. $63,595

B. $76,650

C. $96,450

D. $116,250

Answer (D) is correct. The existence of an equity account with a debit balance signifies that the

available-for-sale securities are reported at fair value that is less than

historical cost. The difference is the net unrealized loss balance. Hence,

historical cost must have been $116,250 ($96,450 available-for-sale

securities at fair value + $19,800 net unrealized loss).

Question: 211 When the fair value of an investment in debt securities exceeds its amortized cost, how should each of the following debt securities be reported at the end of the year, given no election of the fair value option?

Debt Securities Classified As

Held-to-Maturity Available-for-Sale

A. Amortized cost Amortized cost

B. Amortized cost Fair value

Answer (B) is correct. Investments in debt securities must be classified as held-to-maturity and

measured at amortized cost in the balance sheet if the reporting entity has

the positive intent and ability to hold them to maturity. Investments in

equity securities are classified as either trading or available-for-sale.

Equity securities that are not expected to be sold in the near term should

be classified as available-for-sale. These securities should be reported at

fair value, with unrealized holding gains and losses (except those on

securities designated as being hedged in a fair value hedge) excluded

from earnings and reported in OCI.

C. Fair value Fair value

Gleim 2015 | Part 1 | Online MCQs | Unit 002

www.facebook.com/CMA.Arabwebsoft Page 41

حصري | لمنتدى عرب ويب سوفت

D. Fair value Amortized cost

Question: 212 Reed, Inc., began operations on January 1. The following information pertains to Reed’s December 31 securities:

Trading Available-for-Sale

Cost $360,000 $550,000

Fair value 320,000 450,000

Lower of cost or fair value

applied to each security 304,000 420,000

If the declines are judged to be temporary, what amounts should Reed report for its trading and available-for-sale securities in the assets section of its December 31 balance sheet?

Trading Available-for-Sale

A. $360,000 $550,000

B. $360,000 $450,000

C. $320,000 $450,000

Answer (C) is correct. Fair value accounting applies to both trading and available-for-sale

securities. The difference in treatment is that the unrealized holding gains

and losses are included in earnings for trading securities and in other

comprehensive income for available-for-sale securities, assuming the

latter are not designated as being hedged in a fair value hedge. Thus,

these securities should be reported in the assets section of the balance

sheet at their fair values of $320,000 and $450,000, respectively.

D. $304,000 $420,000

Question: 213 Beach Co. determined that the decline in the fair value (FV) of an investment was below the amortized cost and permanent in nature. The investment was classified as available-for-sale on Beach’s books. The controller would properly record the decrease in FV by including it in which of the following?

A. Other comprehensive income section of the income statement only.

B. Earnings section of the income statement and writing down the cost basis

to FV.

Answer (B) is correct. The amortized cost basis is used to calculate the amount of any

impairment. The amortized cost basis should be distinguished from fair

value, which equals the cost basis plus or minus the net unrealized

holding gain or loss. If a decline in fair value of an individual available-

for-sale security below its amortized cost basis is other than temporary,

Gleim 2015 | Part 1 | Online MCQs | Unit 002

www.facebook.com/CMA.Arabwebsoft Page 41

حصري | لمنتدى عرب ويب سوفت

the amortized cost basis is written down to fair value as a new cost basis.

The write-down is deemed to be a realized loss and is included in

earnings.

C. Extraordinary items section of the income statement, net of tax, and writing

down the cost basis to FV.

D. Other comprehensive income section of the income statement, and writing

down the cost basis to FV.

Question: 214 On January 2, Year 1, Adam Co. purchased as a long-term investment 10,000 shares of Mill Corp.’s common stock for $40 a share. These securities were properly classified as available-for-sale. On December 31, Year 1, the market price of Mill’s stock was $35 a share, reflecting a temporary decline in market price. On January 28, Year 2, Adam sold 8,000 shares of Mill stock for $30 a share. For the year ended December 31, Year 2, Adam should report a realized loss on disposal of a long-term investment of

A. $100,000

B. $80,000

Answer (B) is correct. A realized loss or gain is recognized when an individual security is sold

or otherwise disposed of. The investment was acquired for $40 per share.

Because the shares were purchased as a long-term investment, they

should be classified as available-for-sale securities. Thus, the temporary

decline in fair value at 12/31/Year 1 was debited to other comprehensive

income and was not included in earnings. Accordingly, the realized loss

included in earnings at 12/31/Year 2 was $80,000 [8,000 shares × ($40 –

$30)].

C. $60,000

D. $40,000

Question: 215 A corporation acquires a 30% voting interest in another corporation. In this situation, the long-term investment is generally accounted for on the investor corporation’s books using which of the following reporting methods?

A. Lower-of-cost-or-market.

B. Cost.

C. Consolidated.

D. Equity.

Answer (D) is correct. If an investor can exercise significant influence over an investee, the

investment should be accounted for by the equity method. When a

corporation owns 20% or more of the voting power of the investee, the

ability to exercise significant influence is presumed.

Gleim 2015 | Part 1 | Online MCQs | Unit 002

www.facebook.com/CMA.Arabwebsoft Page 42

حصري | لمنتدى عرب ويب سوفت

Question: 216 Johnstone Company owns 10,000 shares of Breva Corporation’s stock; Breva currently has 40,000 shares outstanding. During the year, Breva had net income of $200,000 and paid $160,000 in dividends. At the beginning of the year, there was a balance of $150,000 in Johnstone’s equity method investment in Breva Corporation account. At the end of the year, the balance in this account should be

A. $110,000

B. $150,000

C. $160,000

Answer (C) is correct. Johnstone holds 25% (10,000 ÷ 40,000) of Breva’s voting common

stock. Under the equity method, (1) an investor recognizes its share of

the investee’s net income as an increase in the investment account:

Investment in Breva ($200,000 × 25%) $50,000

Income -- equity-method investee $50,000

(2) a dividend from the investee is treated as a return of an investment:

Cash ($160,000 × 25%) $40,000

Investment in Breva $40,000

Thus, at the end of the year, the balance in the investment in Breva

account is $160,000 ($150,000 + $50,000 – $40,000).

D. $240,000

Question: 217 An investor uses the equity method to account for an investment in common stock. The investor’s equity in the earnings of the investee is affected by

A Change in Fair

Cash Dividends Value of the Investee’s from Investee Common Stock

A. No Yes

B. No No

Answer (B) is correct. Under the equity method, an investor debits an investment and credits

revenue for its share of the investee’s earnings. The receipt of a cash

dividend from the investee is treated as a return of an investment. Thus, it

is credited to the investment but does not affect equity-based earnings. A

change in fair value has no effect on an investment in securities

accounted for under the equity method.

C. Yes No

D. Yes Yes

Question: 218 An investor uses the equity method to account for an investment in common stock. After the date of acquisition, the investment account of the investor is

A. Not affected by its share of the earnings or losses of the investee.

Gleim 2015 | Part 1 | Online MCQs | Unit 002

www.facebook.com/CMA.Arabwebsoft Page 43

حصري | لمنتدى عرب ويب سوفت

B. Not affected by its share of the earnings of the investee, but is decreased by

its share of the losses of the investee.

C. Increased by its share of the earnings of the investee, but is not affected by

its share of the losses of the investee.

D. Increased by its share of the earnings of the investee, and is decreased by

its share of the losses of the investee.

Answer (D) is correct. After the date of acquisition, an equity-based investment in common

stock account of an investor is increased by its share of the earnings of

the investee, decreased by its share of the losses of the investee, and

decreased by its share of cash dividends received from the investee.

Question: 219 On January 1, Dyer Co. acquired as a long-term investment a 20% common stock interest in Eason Co. Dyer paid $700,000 for this investment when the fair value and carrying amount of Eason’s net assets was $3.5 million. Dyer can exercise significant influence over Eason’s operating and financial policies. For the year ended December 31, Eason reported net income of $400,000 and declared and paid cash dividends of $160,000. How much revenue from this investment should Dyer report for the year?

A. $32,000

B. $48,000

C. $80,000

Answer (C) is correct. Because the investor can exercise significant influence over the

investee’s operating and financial policies, the investment should be

accounted for using the equity method. The $700,000 paid for the

investment is equal to 20% of the $3.5 million fair value. Moreover, the

carrying amount and fair value of the net assets were the same. Thus, no

goodwill impairment or other acquisition differential that might require

adjustment of Dyer’s share of the investee’s net income is associated

with this investment. Under these circumstances, revenue from the

investment is 20% of the reported net income of $400,000, or $80,000.

The cash dividend does not affect the amount of income to be reported.

D. $112,000

Question: 220 Peel Co. received a cash dividend from a common stock investment. Should Peel report an increase in the investment account if it uses the fair value method or the equity method of accounting?

Fair Value Equity

A. No No

Answer (A) is correct. Under the fair value method and the cost method, (used only if the fair

value method and the equity method are not applicable), dividends from

Gleim 2015 | Part 1 | Online MCQs | Unit 002

www.facebook.com/CMA.Arabwebsoft Page 44

حصري | لمنتدى عرب ويب سوفت

an investee should be accounted for by the investor as dividend income

unless a liquidating dividend is received. Thus, assuming that the

dividend is not liquidating, it has no effect on the investment account

under the fair value method. Under the equity method, the investor

recognizes its equity in the undistributed earnings of the investee.

Consequently, cash dividends decrease the investment account because

the dividend is considered to be a return of investment.

B. Yes Yes

C. Yes No

D. No Yes

Question: 221 When an investor uses the equity method to account for investments in common stock, the investment account will be increased when the investor recognizes

A. A proportionate interest in the net income of the investee.

Answer (A) is correct. Under the equity method, the investor’s share of the investee’s net

income is accounted for as an addition to the carrying amount of the

investment on the investor’s books. Losses and dividends are reflected as

reductions of the carrying amount.

B. A cash dividend received from the investee.

C. Periodic amortization of the goodwill related to the purchase.

D. Depreciation related to the excess of fair value over the carrying amount of

the investee’s depreciable assets at the date of purchase by the investor.

Question: 222 Green Corp. owns 30% of the outstanding common stock and 100% of the outstanding noncumulative nonvoting preferred stock of Axel Corp. In Year 1, Axel declared dividends of $100,000 on its common stock and $60,000 on its preferred stock. Green exercises significant influence over Axel’s operations and uses the equity method to account for the investment in the common stock. What amount of dividend revenue should Green report in its income statement for the year ended December 31, Year 1?

A. $0

B. $30,000

C. $60,000

Answer (C) is correct. Under the equity method, the receipt of a cash dividend from the investee

should be credited to the investment account. It is a return of, not a return

on, the investment. However, the equity method is not applicable to

preferred stock. Thus, Green should report $60,000 of revenue when the

preferred dividends are declared.

D. $90,000

Gleim 2015 | Part 1 | Online MCQs | Unit 002

www.facebook.com/CMA.Arabwebsoft Page 45

حصري | لمنتدى عرب ويب سوفت

Question: 223 On July 1, Year 1, Denver Corp. purchased 3,000 shares of Eagle Co.’s 10,000 outstanding shares of common stock for $20 per share but did not elect the fair value option. On December 15, Year 1, Eagle paid $40,000 in dividends to its common shareholders. Eagle’s net income for the year ended December 31, Year 1, was $120,000, earned evenly throughout the year. In its Year 1 income statement, what amount of income from this investment should Denver report?

A. $36,000

B. $18,000

Answer (B) is correct. Denver Corp.’s purchase of 30% of Eagle presumably allows it to

exercise significant influence. Hence, it should apply the equity method.

The investor’s share of the investee’s income is a function of the

percentage of ownership and the length of time the investment was held.

The income from this investment was therefore $18,000 [$120,000 ×

30% × (6 months ÷ 12 months)].

C. $12,000

D. $6,000

Question: 224 Entity A acquires all of the voting shares of Entity B for $1,000,000. At the time of the acquisition, the net fair value of the identifiable assets acquired and liabilities assumed had a carrying amount of $900,000 and a fair value of $800,000. The amount of goodwill Entity A will record on the acquisition date is

A. $0

B. $100,000

C. $200,000

Answer (C) is correct. Given no prior equity interest or noncontrolling interest, goodwill equals

the excess of the fair value of the consideration transferred over the fair

value of the net of the identifiable assets acquired and liabilities assumed.

Consequently, goodwill is $200,000 ($1,000,000 – $800,000).

D. $300,000

Question: 225 A conglomerate entity acquired 100% of the net assets of a target entity for $900 cash. The target entity’s statement of financial position just prior to the acquisition is presented below.

Target Entity (as of acquisition date)

Carrying Fair

Amount Value

Cash $ 100 $100

Receivables 200 200

Inventory 150 200

Property, plant,

Gleim 2015 | Part 1 | Online MCQs | Unit 002

www.facebook.com/CMA.Arabwebsoft Page 46

حصري | لمنتدى عرب ويب سوفت

and equipment (net) 600 400

Total assets $1,050 $900

Current liabilities $ 200 $200

Share capital 200

Retained earnings 650

Total liabilities and equity $1,050

The amount of goodwill to be recorded by the conglomerate entity related to its purchase of the target entity is

A. $(200)

B. $50

C. $200

Answer (C) is correct. Given no prior equity interest and no noncontrolling interest, goodwill is

the excess of the fair value of the consideration transferred over the net

of the fair values of the identifiable net assets acquired. This net fair

value equals the sum of cash, receivables, inventory, and PPE, minus

liabilities. Hence, the net fair value acquired is $700, and goodwill is

$200 ($900 fair value of the consideration transferred – $700).

D. None of the answers are correct.

Question: 226 Entity X owns 90% of Entity Y. Early in the year, X lent Y $1,000,000. No payments have been made on the debt by year end. Proper accounting at year end in the consolidated financial statements would

A. Eliminate 100% of the receivable, the payable, and the related interest.

Answer (A) is correct. In a consolidated statement of financial position, reciprocal balances,

such as receivables and payables, between a parent and a consolidated

subsidiary should be eliminated in their entirety regardless of the portion

of the subsidiary’s shares held by the parent. Thus, all effects of the

$1,000,000 loan should be eliminated in the preparation of the year-end

consolidated statement of financial position.

B. Eliminate 100% of the receivable and the payable but not any related

interest.

C. Eliminate 90% of the receivable, the payable, and the related interest.

D. Eliminate 90% of the receivable and the payable but not any related

interest.

Gleim 2015 | Part 1 | Online MCQs | Unit 002

www.facebook.com/CMA.Arabwebsoft Page 47

حصري | لمنتدى عرب ويب سوفت

Question: 227 Costs incurred in completing a business combination are listed below.

General administrative costs $240,000

Consulting fees 120,000

Direct cost to register and issue equity securities 80,000

The amount charged to the expenses of the business combination is

A. $80,000

B. $120,000

C. $240,000

D. $360,000

Answer (D) is correct. Acquisition-related costs, such as finder’s fees, professional and

consulting fees, and general administrative costs, are expensed as

incurred. But direct issue costs of equity (underwriting, legal, accounting,

tax, registration, etc.) are debited to additional paid-in capital.

Accordingly, the amount expensed is $360,000 ($240,000 + $120,000).

Question: 228 Alton Corporation purchased 100% of the shares of Jones Corporation for $600,000. Financial information for Jones Corporation is provided below.

Jones Corporation ($000)

Carrying

Amount

Fair

Value

Cash $ 50 $ 50

Accounts receivable 100 100

Inventory 150 100

Total current assets 300 250

Property, plant, and equipment (net) 500 600

Total assets $800 $850

Current liabilities $150 $150

Long-term debt 200 200

Total liabilities 350 350

Common stock 150 150

Paid-in capital 80 80

Gleim 2015 | Part 1 | Online MCQs | Unit 002

www.facebook.com/CMA.Arabwebsoft Page 48

حصري | لمنتدى عرب ويب سوفت

Retained earnings 220

Total shareholders’ equity 450

Total liabilities and shareholders’ equity $800

The amount of goodwill resulting from this purchase, if any, would be

A. $200,000

B. $150,000

C. $100,000

Answer (C) is correct. Goodwill is the excess of (1) the sum of the acquisition-date fair values

of (a) the consideration transferred ($600,000), (b) any noncontrolling

interest in the acquiree ($0), and (c) the acquirer’s previously held equity

interest in the acquiree ($0) over (2) the net of the acquisition-date fair

values of the identifiable assets acquired ($850,000) and liabilities

assumed ($350,000). The amount of goodwill is calculated as follows:

Consideration transferred $600,000

Acquisition-date fair value of net assets acquired ($850,000 –

$350,000) (500,000)

Goodwill $100,000

D. $0

Question: 229 How should the acquirer recognize a bargain purchase in a business acquisition?

A. As negative goodwill in the statement of financial position.

B. As goodwill in the statement of financial position.

C. As a gain in earnings at the acquisition date.

Answer (C) is correct. A bargain purchase is recognized in the consolidated financial statements

as an ordinary gain at the acquisition date. A bargain purchase occurs

when the net of the acquisition-date fair values of identifiable assets

acquired and liabilities assumed exceeds the sum of the acquisition-date

fair values of the consideration transferred, any noncontrolling interest

recognized, and any previously held equity interest in the acquiree.

D. As a deferred gain that is amortized into earnings over the estimated future

periods benefited.

Gleim 2015 | Part 1 | Online MCQs | Unit 002

www.facebook.com/CMA.Arabwebsoft Page 49

حصري | لمنتدى عرب ويب سوفت

Question: 230 Par Corp. owns 60% of Sub Corp.’s outstanding capital stock. On May 1, Par advanced Sub $70,000 in cash, which was still outstanding at December 31. What portion of this advance should be eliminated in the preparation of the December 31 consolidated balance sheet?

A. $70,000

Answer (A) is correct. In a consolidated balance sheet, reciprocal balances, such as receivables

and payables, between a parent and a consolidated subsidiary should be

eliminated in their entirety regardless of the portion of the subsidiary’s

stock held by the parent. Thus, the entire $70,000 advance should be

eliminated in the preparation of the year-end consolidated balance sheet.

B. $42,000

C. $28,000

D. $0

Question: 231 Shep Co. has a receivable from its parent, Pep Co. Should this receivable be separately reported in Shep’s balance sheet and in Pep’s consolidated balance sheet.

Shep’s Pep’s Consolidated Balance Sheet Balance Sheet

A. Yes No

Answer (A) is correct. In a consolidated balance sheet, reciprocal balances, such as receivables

and payables, between a parent and a consolidated subsidiary are

eliminated in their entirety, regardless of the portion of the subsidiary’s

stock held by the parent. However, intraentity transactions should not be

eliminated from the separate financial statements of the entities.

B. Yes Yes

C. No No

D. No Yes

Question: 232 Water Co. owns 80% of the outstanding common stock of Fire Co. On December 31, Year 3, Fire sold equipment to Water at a price in excess of Fire’s carrying amount but less than its original cost. On a consolidated balance sheet at December 31, Year 3, the carrying amount of the equipment should be reported at

A. Water’s original cost.

B. Fire’s original cost.

C. Water’s original cost minus Fire’s recorded gain.

Answer (C) is correct. In consolidated financial statements, the effects of intraentity transactions

should be eliminated. The original amount recorded for the acquisition

by Water Co. of the equipment from Fire Co. was the carrying amount on

Fire’s balance sheet plus the gain on the sale. In the consolidated

Gleim 2015 | Part 1 | Online MCQs | Unit 002

www.facebook.com/CMA.Arabwebsoft Page 51

حصري | لمنتدى عرب ويب سوفت

financial statements, the equipment should be reported at the amount

previously recorded on Fire’s balance sheet. This amount is the original

cost recorded by Water minus the gain recognized by Fire when the

transaction took place.

D. Water’s original cost minus 80% of Fire’s recorded gain.

Question: 233 Vadis Co. sells appliances that include a 3-year warranty. Service calls under the warranty are performed by an independent mechanic under a contract with Vadis. Based on experience, warranty costs are estimated at $30 for each machine sold. When should Vadis recognize these warranty costs?

A. Evenly over the life of the warranty.

B. When the service calls are performed.

C. When payments are made to the mechanic.

D. When the machines are sold.

Answer (D) is correct. Under the accrual method, a provision for warranty costs is made when

the related revenue is recognized.

Question: 234 East Corp. manufactures stereo systems that carry a 2-year warranty against defects. Based on past experience, warranty costs are estimated at 4% of sales for the warranty period. During the year, stereo system sales totaled $3 million, and warranty costs of $67,500 were incurred. In its income statement for the year ended December 31, East should report warranty expense of

A. $52,500

B. $60,000

C. $67,500

D. $120,000

Answer (D) is correct. Warranty expense equals 4% of sales for the period, or $120,000

($3,000,000 × 4%).

Question: 235 During Year 3, Rex Co. introduced a new product carrying a 2-year warranty against defects. The estimated warranty costs related to dollar sales are 2% within 12 months following sale and 4% in the second 12 months following sale. Sales and actual warranty expenditures for the years ended December 31, Year 3 and Year 4, are as follows:

Actual Warranty

Sales Expenditures

Year 3 $ 600,000 $ 9,000

Year 4 1,000,000 30,000

Gleim 2015 | Part 1 | Online MCQs | Unit 002

www.facebook.com/CMA.Arabwebsoft Page 51

حصري | لمنتدى عرب ويب سوفت

$1,600,000 $39,000

At December 31, Year 4, Rex should report an estimated warranty liability of

A. $0

B. $39,000

C. $57,000

Answer (C) is correct. Because this product is new, the beginning balance in the estimated

warranty liability account at the beginning of Year 3 is $0. For Year 3,

the estimated warranty costs related to dollar sales are 6% (2% + 4%) of

sales or $36,000 ($600,000 × 6%). For Year 4, the estimated warranty

costs are $60,000 ($1,000,000 sales × 6%). These amounts are charged to

warranty expense and credited to the estimated warranty liability

account. This liability account is debited for expenditures of $9,000 and

$30,000 in Year 3 and Year 4, respectively. Hence, the estimated

warranty liability at 12/31/Yr 4 is $57,000.

Estimated Warranty Liability

$ 0 1/1/Yr 3

Year 3 expenditures $ 9,000 36,000 Year 3 expense

Year 4 expenditures 30,000 60,000 Year 4 expense

$57,000

12/31/Yr 4

D. $96,000

Question: 236 On April 1, Ash Corp. began offering a new product for sale under a 1-year warranty. Of the 5,000 units in inventory at April 1, 3,000 had been sold by June 30. Based on its experience with similar products, Ash estimated that the average warranty cost per unit sold would be $8. Actual warranty costs incurred from April 1 through June 30 were $7,000. At June 30, what amount should Ash report as estimated warranty liability?

A. $9,000

B. $16,000

C. $17,000

Answer (C) is correct. If 3,000 units were sold at an estimated $8 per unit warranty cost, the

total credits to the liability account equaled $24,000 (3,000 × $8). Given

that actual warranty costs of $7,000 were debited to the account, the

ending balance must have been $17,000 ($24,000 – $7,000).

D. $33,000

Gleim 2015 | Part 1 | Online MCQs | Unit 002

www.facebook.com/CMA.Arabwebsoft Page 52

حصري | لمنتدى عرب ويب سوفت

Question: 237 A corporation entered into a purchase commitment to buy inventory. At the end of the accounting period, the current market value of the inventory was less than the fixed purchase price by a material amount. Which of the following accounting treatments is most appropriate?

A. Describe the nature of the contract in a note to the financial statements,

recognize a loss in the income statement, and recognize a liability for the

accrued loss.

Answer (A) is correct. A commitment to acquire goods in the future is not recorded at the time

of the agreement, e.g., by debiting an asset and crediting a liability. But

recognition in earnings of a loss on goods subject to a firm purchase

commitment is required if the market price of these goods declines below

the commitment price. The reason for current loss recognition is the

same as that for inventory on hand. A decrease (not an increase) in the

future benefits of the commitment should be recognized when it occurs.

Thus, the lower of cost or market rule is followed. If material losses are

expected to arise from firm, noncancelable, and unhedged commitments

for the future purchase of inventory, they should be measured in the same

way as inventory losses, and, if material, recognized and separately

disclosed in the income statement. The entry is to debit unrealized

holding loss-earnings and to credit liability-purchase commitment.

Furthermore, certain disclosures are required for unconditional purchase

obligations that are unrecorded. They include the nature and term of the

obligation.

B. Describe the nature of the contract and the estimated amount of the loss in

a note to the financial statements, but do not recognize a loss in the income

statement.

C. Describe the nature of the contract in a note to the financial statements,

recognize a loss in the income statement, and recognize a reduction in

inventory equal to the amount of the loss by use of a valuation account.

D. Neither describe the purchase obligation nor recognize a loss on the income

statement or balance sheet.

Question: 238 Net losses on firm purchase commitments to acquire goods for inventory result from a contract price that exceeds the current market price. If a firm expects that losses will occur when the purchase occurs, expected losses, if material,

A. Should be recognized in the accounts and separately disclosed as losses on

the income statement of the period during which the decline in price takes

place.

Answer (A) is correct. A loss is accrued in the income statement on goods subject to a firm

purchase commitment if the market price of these goods declines below

the commitment price. This loss should be measured in the same manner

as inventory losses. Disclosure of the loss is also required.

B. Should be recognized in the accounts and separately disclosed as net

unrealized losses on the balance sheet at the end of the period during which

Gleim 2015 | Part 1 | Online MCQs | Unit 002

www.facebook.com/CMA.Arabwebsoft Page 53

حصري | لمنتدى عرب ويب سوفت

the decline in price takes place.

C. Should be recognized in the accounts and separately disclosed as net

unrealized losses on the balance sheet at the end of the period during which

the contract is executed.

D. Should not be recognized in the accounts until the contract is executed and

need not be separately disclosed in the financial statements.

Question: 239 A liability arising from a loss contingency should be recorded if the

A. Amount of the loss can be reasonably estimated.

B. Contingent future events have a reasonably possible chance of occurring.

C. Contingent future events have a reasonably possible chance of occurring

and the amount of the loss can be reasonably estimated.

D. Contingent future events will probably occur and the amount of the loss

can be reasonably estimated.

Answer (D) is correct. A material contingent loss must be accrued when the following two

conditions are met:

1. It is probable that, at the balance sheet date, an asset has

been impaired or a liability has been incurred.

2. The amount of the loss can be reasonably estimated.

Question: 240 Which one of the following loss contingencies would be accrued as a liability rather than disclosed in the notes to the financial statement?

A. A guarantee of the indebtedness of another.

B. A dispute over additional income taxes assessed for prior years (now in

litigation).

C. A pending lawsuit with an uncertain outcome.

D. Liabilities for service or product warranties made as a regular part of

business.

Answer (D) is correct. Similarly to the guidelines for loss contingencies, a liability for future

warranty costs should be accrued if (1) the incurrence of the expense is

probable and (2) the amount can be reasonably estimated.

Question: 241 Linden Corporation is a defendant in a lawsuit where the plaintiff is seeking $1,000,000 in damages. The company had terminated the plaintiff, George Russell, from his position with Linden after Russell allegedly sold specifications for one of Linden’s new products to a competitor. Linden’s attorney believes that it is quite possible Linden will lose the case and that, if so, damages could range from $100,000 to $200,000. Regardless of the outcome of the case, Linden’s accountants estimate the company will incur an additional $5,000 in unemployment costs because of Russell’s termination. The

Gleim 2015 | Part 1 | Online MCQs | Unit 002

www.facebook.com/CMA.Arabwebsoft Page 54

حصري | لمنتدى عرب ويب سوفت

amount that Linden should accrue because of the contingency in this situation is

A. $200,000

B. $100,000

C. $5,000

D. $0

Answer (D) is correct. Loss contingencies are accrued when the loss is probable. The $5,000 in

unemployment costs that will probably be incurred are a routine cost of

doing business.

Question: 242 Ichabod Company is the plaintiff in two lawsuits. The first suit involves a competitor who has made an exact copy of one of Ichabod’s products, and Ichabod is suing for patent infringement. The attorneys estimate a $5,000,000 award for Ichabod; however, it is anticipated that the case will be in litigation for 2 to 3 years before final resolution.

The second case also involves patent infringement; however, in this instance, the attorneys

do not believe Ichabod has a strong case. It is estimated that the company has a 50%

chance of winning and the award, if any, would be in the $250,000 to $1,000,000 range.

The most appropriate amount to be recorded as a gain contingency is

A. $0

Answer (A) is correct. Gain contingencies are not recorded; they are recognized only when

realized. A gain contingency must be adequately disclosed.

B. $5,000,000

C. $5,125,000

D. $5,250,000

Question: 243 Warren Company is being sued in a wrongful discharge suit for $500,000. The company attorney has advised Warren that the probability of the plaintiff prevailing and receiving the full amount is about 80%. The attorney also indicated that the case would likely be tied up in the courts for 2 to 3 years. The most appropriate financial statement presentation for this loss contingency would be to

A. Record $500,000 as a loss contingency.

Answer (A) is correct. A liability arising from a loss contingency should be recorded if the

contingent future event will probably occur and the amount of the loss

can be reasonably estimated.

B. Record $400,000 as a loss contingency.

C. Disclose the loss contingency in the footnotes.

D. Not record or footnote the loss contingency.

Gleim 2015 | Part 1 | Online MCQs | Unit 002

www.facebook.com/CMA.Arabwebsoft Page 55

حصري | لمنتدى عرب ويب سوفت

Question: 244 In May Year 1, Caso Co. filed suit against Wayne, Inc., seeking $1.9 million in damages for patent infringement. A court verdict in November Year 4 awarded Caso $1.5 million in damages, but Wayne’s appeal is not expected to be decided before Year 6. Caso’s counsel believes it is probable that Caso will be successful against Wayne for an estimated amount in the range between $800,000 and $1.1 million, with $1 million considered the most likely amount. What amount should Caso record as income from the lawsuit in the year ended December 31, Year 4?

A. $0

Answer (A) is correct. Gain contingencies are not recognized until they are realized. Because

the appeal is not expected to be decided before Year 6, Caso should not

record any revenue from the lawsuit in the Year 4 income statement. This

gain contingency should be disclosed; however, care should be taken to

avoid misleading implications as to the likelihood of realization.

B. $800,000

C. $1,000,000

D. $1,500,000

Gleim 2015 | Part 1 | Online MCQs | Unit 003

www.facebook.com/CMA.Arabwebsoft Page 1

حصري | لمنتدى عرب ويب سوفت

Question: 245 Which of the following is not an appropriate basis for measuring the cost of property, plant, and equipment?

A. The purchase price, freight costs, and installation costs of a productive

asset should be included in the asset’s cost.

B. Proceeds obtained in the process of readying land for its intended purpose,

such as from the sale of cleared timber, should be recognized immediately

as income.

Answer (B) is correct. Accordingly, items of property, plant, and equipment (PPE) that meet the

recognition criterion are initially measured at cost. The cost includes the

purchase price (minus trade discounts and rebates, plus purchase taxes)

and the directly attributable costs of bringing the assets to working

condition for their intended use. Directly attributable costs include site

preparation, installation, initial delivery and handling, architect and

equipment fees, costs of removing the assets and restoring the site, etc.

Accordingly, the cost of land includes the cost of obtaining the land and

readying it for its intended uses, but it is inappropriate to recognize the

proceeds related to site preparation immediately in profit or loss. They

should be treated as reductions in the price of the land.

C. The costs of improvements to equipment incurred after its acquisition

should be added to the asset’s cost if they increase future service potential.

D. All costs incurred in the construction of a plant building, from excavation

to completion, should be considered as part of the asset’s cost.

Question: 246 An entity installed an assembly line in Year 1. Four years later, $100,000 was invested to automate the line. The automation increased the market value and productive capacity of the assembly line but did not affect its useful life. Proper accounting for the cost of the automation should be to

A. Report it as an expense in Year 5.

B. Establish a separate account for the $100,000.

C. Allocate the cost of automation between the asset and accumulated

depreciation accounts.

D. Debit the cost to the property, plant, and equipment account.

Answer (D) is correct. Subsequent costs are added to the carrying amount of an item of PPE if it

is probable that, as a result, future economic benefits will be received,

and the costs are reliably measurable. An extended useful life, improved

output quantity or quality, and reduced operating costs are all future

economic benefits.

Gleim 2015 | Part 1 | Online MCQs | Unit 003

www.facebook.com/CMA.Arabwebsoft Page 2

حصري | لمنتدى عرب ويب سوفت

Question: 247 The selected data from statements of financial position on December 31, Year 1, and December 31, Year 2, is presented below:

12/31/Year

1

12/31/Year

2

Property,

plant,

and

equipment

$295,000 $340,000

Accumulated

depreciation (102,000) (119,000)

Additional information for Year 2:

1. Equipment was acquired

for $65,000.

2. Depreciation expense was

$30,000.

The carrying amount (cost minus accumulated depreciation) of property, plant, and equipment disposed of in Year 2 was

A. $7,000

Answer (A) is correct. The Year 2 beginning carrying amount is $193,000 ($295,000 –

$102,000), and the Year 2 ending carrying amount is $221,000

($340,000 – $119,000). The carrying amount of PPE disposed of is

$7,000 ($193,000 beginning balance + $65,000 acquired during Year 2 –

$30,000 depreciation expense – $221,000 ending balance).

B. $17,000

C. $20,000

D. $32,000

Question: 248 A theme park purchased a new, exciting ride and financed it through the manufacturer. The following facts pertain:

Purchase price $800,000

Delivery cost 50,000

Installation cost 70,000

Cost of trial runs 40,000

Interest charges for first year 60,000

The straight-line method is to be used. Compute the depreciation on the equipment for the first year assuming an estimated service life of 5 years.

A. $160,000

B. $184,000

C. $192,000

Gleim 2015 | Part 1 | Online MCQs | Unit 003

www.facebook.com/CMA.Arabwebsoft Page 3

حصري | لمنتدى عرب ويب سوفت

Answer (C) is correct. Under the straight-line method, the annual depreciation expense for an

asset equals the asset’s amount (cost – residual value) divided by the

asset’s estimated useful life. The cost of the asset includes its price and

the directly attributable costs of bringing it to working condition for

intended use. Thus, the depreciation expense is $192,000 [($800,000

purchase price + $50,000 delivery cost + $70,000 installation cost +

$40,000 trial-run cost) ÷ 5-year estimated service life]. Borrowing costs

incurred after the asset is prepared for its intended use are expensed even

if the allowed alternative treatment of such costs is followed, and the

asset otherwise satisfies the criteria for capitalization of such expenses.

D. $204,000

Question: 249 Which of the following is not an appropriate basis for measuring the historical cost of property, plant, and equipment?

A. Delivery and handling costs and installation costs of a productive asset

should be included in the asset’s cost.

B. The cost should include the purchase price without a deduction for trade

discounts.

Answer (B) is correct. An asset classified under property, plant, and equipment is measured

initially at cost. This amount includes the purchase price and any directly

attributable costs of bringing the asset to working condition for its

intended use. Directly attributable costs include costs of, for example,

site preparation, initial delivery and handling, installation, professional

fees (e.g., those of architects and engineers), and dismantling and

removing the asset and restoring the site. The purchase price is

determined by adding any import fees and nonrefundable purchase taxes

and subtracting any trade discounts and rebates.

C. The costs of improvements to equipment incurred after its acquisition

should be added to the asset’s cost if they provide future economic benefits

exceeding the originally assessed standard of performance.

D. All costs incurred in the construction of a plant building, from excavation

to completion, should be considered as part of the asset’s cost.

Question: 250 In making a cash flow analysis of property, plant, and equipment (PPE), the internal auditor discovered that depreciation expense for the period was $10,000. PPE with a cost of $50,000 and related accumulated depreciation of $30,000 was sold for a gain of $1,000. If the carrying amount of PPE increased by $80,000 during the period, how much PPE was purchased this period?

A. $91,000

B. $100,000

C. US $110,000

Gleim 2015 | Part 1 | Online MCQs | Unit 003

www.facebook.com/CMA.Arabwebsoft Page 4

حصري | لمنتدى عرب ويب سوفت

Answer (C) is correct. The carrying amount of the PPE account, net of accumulated

depreciation, is increased by the cost of purchases and decreased by the

carrying amount of items of PPE sold and depreciation. The net PPE

decreased by the carrying amount of items sold, or $20,000 ($50,000 cost

– $30,000 accumulated depreciation), and by the $10,000 of

depreciation. If PPE still increased by $80,000, $110,000 ($30,000 total

decrease + $80,000 increase) of equipment must have been purchased.

D. $119,000

Question: 251 On January 1, Year 1, an entity purchased an abandoned quarry for $1,200,000 to be used as a landfill to service its trash collection contracts with nearby cities for the next 20 years. The entity depletes the quarry using the units-of-production method based on a surveyor’s measurements of volume of the quarry’s pit. This amount was 500,000 cubic yards when purchased and 350,000 cubic yards at year-end Year 5. What is the net amount that should be shown on the entity’s December 31, Year 5, statement of financial position for the quarry?

A. $1,200,000

B. $900,000

C. $840,000

Answer (C) is correct. The units-of-production method allocates cost based on output. The net

amount reported as an asset for the quarry using this method is $840,000

[(350,000 cubic yards ÷ 500,000 total cubic yards) × $1,200,000].

D. $360,000

Question: 252 A new machine has an initial cost of $300,000, an estimated useful life of 2,000 hours of use over a 3-year period, and an estimated residual value of $70,000. Usage rates are estimated as 500 hours in the first year, 700 hours in the second year, and 800 hours in the third year. Depreciation expense in Year 2 under the units-of-production method of depreciation will be

A. $57,500

B. $75,000

C. $80,500

Answer (C) is correct. Depreciation expense equals cost minus residual value, times the

estimated hours of use in Year 2 divided by the total estimated hours of

use. Thus, depreciation expense is $80,500 [($300,000 – $70,000) × (700

hours ÷ 2,000 hours)].

D. $105,000

Gleim 2015 | Part 1 | Online MCQs | Unit 003

www.facebook.com/CMA.Arabwebsoft Page 5

حصري | لمنتدى عرب ويب سوفت

Question: 253 A company uses straight-line depreciation for financial reporting purposes, but uses accelerated depreciation for tax purposes. Which of the following account balances would be lower in the financial statements used for tax purposes than it would be in the general purpose financial statements?

A. Accumulated depreciation.

B. Cash.

C. Retained earnings.

Answer (C) is correct. Because the tax basis uses an accelerated method, depreciation expense

and accumulated depreciation will be greater. Moreover, taxable income

will be lower than financial net income. Consequently, tax-basis retained

earnings will be less than that in the general purpose financial statements.

D. Gross property, plant, and equipment.

Question: 254All of the following would be included as part of the cost of a depreciable

asset except the

A. Costs to level land to make it usable for the company’s purposes.

Answer (A) is correct. Site preparation costs [clearing, draining, filling, leveling the property, and razing existing

buildings, minus any proceeds (such as timber sales)] are costs of the land, not of the

building to be constructed on the land.

B. Freight costs to ship new equipment to the company’s facility.

C. Actual interest costs incurred during the construction of a new building.

D. Costs to construct a driveway on the company’s property.

Question: 255 The board of directors of Ingold Industries, Inc., authorized Don Burger, president of Ingold, to pay as much as $90,000 to purchase a tract of land adjacent to the main factory. Burger negotiated a price of $75,800 for the land, and legal fees for closing costs amounted to $820. A contractor cleared, filled, and graded the land for $6,800, and dug the foundation for a new building for $4,300. A prefabricated building was erected at a cost of $181,000. The building has an estimated useful life of 20 years with no residual value. The contractor’s bill indicated that the cost of the parking lot and driveways was $7,060. The parking lot and the driveways will need to be replaced in 15 years. The proper amount to be recorded in Ingold’s land account is

A. $76,620

B. $83,420

Answer (B) is correct. The cost of acquiring and preparing land for its expected use is

capitalized. The amount to be recorded in the land account is $83,420,

consisting of the $75,800 purchase price, the $820 closing costs, and the

$6,800 site preparation costs.

C. $87,720

D. $90,480

Gleim 2015 | Part 1 | Online MCQs | Unit 003

www.facebook.com/CMA.Arabwebsoft Page 6

حصري | لمنتدى عرب ويب سوفت

Question: 256Basic Brick, Inc., purchased manufacturing equipment for $100,000, with an estimated

useful life of 10 years and a salvage value of $15,000. The second year’s depreciation for this

equipment using the double-declining balance method is

A. $8,500

B. $13,600

C. $16,000

Answer (C) is correct. Under the double-declining balance method, the full cost of the asset, or $100,000, is

depreciated, but not below salvage value. Because the straight-line rate for a 10-year asset

is 10% (100% ÷ 10), the double-declining balance rate is 20% (10% × 2). The first year’s

depreciation is $20,000 ($100,000 × 20%), leaving a carrying amount for the second year

of $80,000 ($100,000 – $20,000). The second year’s depreciation is thus $16,000 ($80,000

× 20%).

D. $20,000

Question: 257 Which one of the following characteristics is not required for an asset to be properly described as property, plant, and equipment?

A. Held for use and not for investment.

B. Newly purchased.

Answer (B) is correct. These assets are known variously as property, plant, and equipment;

fixed assets; or plant assets.

1. PPE are tangible. They have physical existence.

2. PPE may be either personal property (something movable,

e.g., equipment) or real property (such as land or a building).

3. PPE are used in the ordinary operations of an entity and

are not held primarily for investment, resale, or inclusion in

another product. But they are often sold.

4. PPE are noncurrent. They are not expected to be used up

within 1 year or the normal operating cycle of the business,

whichever is longer.

However, an asset need not be newly purchased to be properly described

as property, plant, and equipment.

C. Expected life of more than 1 year.

D. Tangible.

Gleim 2015 | Part 1 | Online MCQs | Unit 003

www.facebook.com/CMA.Arabwebsoft Page 7

حصري | لمنتدى عرب ويب سوفت

Question: 258 Equipment bought by Wilson Steam Generating Company 3 years ago was charged to equipment expense in error. The cost of the equipment was $100,000, with no expected salvage value and a 10-year estimated life. Wilson uses the straight-line depreciation method on similar equipment. The error was discovered at the end of Year 3 prior to the issuance of Wilson’s financial statements. After correction of the error, the correct carrying value of the equipment will be

A. $30,000

B. $70,000

Answer (B) is correct. The straight-line depreciation that should have been charged to the

equipment had it been properly capitalized is $30,000 [$100,000 × (3 ÷

10 years)]. Thus, after correction of the error, the carrying amount of the

equipment will be $70,000 ($100,000 – $30,000).

C. $80,000

D. $100,000

Question: 259 The types of assets that qualify for interest capitalization are

A. Assets that are being used in the earning activities of the reporting entity.

B. Assets that are ready for their intended use in the activities of the reporting

entity.

C. Assets that are constructed for the reporting entity’s own use.

Answer (C) is correct. Interest should be capitalized for (1) assets constructed or otherwise

produced for an entity’s own use, including those constructed or

produced by others; (2) assets intended for sale or lease that are

constructed or produced as discrete projects (e.g., ships); and (3) certain

equity-based investments. An asset constructed for an entity’s own use

qualifies for capitalization of interest if (1) relevant expenditures have

been made, (2) activities necessary to prepare the asset for its intended

use are in progress, and (3) interest is being incurred. The investee must

have activities in progress necessary to commence its planned principal

operations and be expending funds to obtain qualifying assets for its

operations.

D. Inventories that are manufactured in large quantities on a continuing basis.

Question: 260 Wellington Industries has owned its present facilities since 1981, and Mary Dunlap, CEO, has authorized various expenditures to repair and improve the building during the current year. The building was beginning to sag, and without repair, the building would only last another 8 years. To correct the problem, the foundation was reinforced, and several columns were added in the basement area at a cost of $47,200. As a result, engineers estimate that the building will have a remaining useful life of 20 years. To install a new computer local area network (LAN) and be ready for the next generation of computers, the phone lines and electrical systems were updated at a cost of $81,300. Wellington engineers estimate that these improvements should last 25 years. The offices and open work spaces were rearranged to reduce exposure to electronic emissions at a material cost of $31,000. The purchase and installation of the computers and software for the LAN cost $102,700. The LAN hardware and software will have to be replaced in 6 years, but further rearrangement of the offices and work spaces will not be necessary. After the above improvements were completed, the entire building was painted inside and outside at a cost

Gleim 2015 | Part 1 | Online MCQs | Unit 003

www.facebook.com/CMA.Arabwebsoft Page 8

حصري | لمنتدى عرب ويب سوفت

of $9,450.

As controller of Wellington Industries, which one of the following actions would you

recommend to be in conformity with generally accepted accounting principles?

A. Treat all expenditures as expenses in the current year except the cost of

rearrangement ($31,000), which should be amortized over a period not to

exceed 20 years.

B. Capitalize all expenditures because they represent additions, improvements,

and rearrangements.

C. Capitalize all costs with the exception of the upgrade to the phone and

electrical systems and the painting because they represent maintenance

expenses.

D. Capitalize all costs with the exception of the painting because it represents

maintenance expense.

Answer (D) is correct. Expenditures on capital assets that improve the asset’s performance or

extend its useful life are capitalized as part of the asset’s cost.

Accordingly, the building repairs are capitalized. The substitution of a

better computer system is classified as an improvement, and the costs

also should be capitalized. Moreover, the entity capitalizes the costs of a

rearrangement of the configurations of the offices and open work spaces

that (1) requires material outlays, (2) is separable from recurring

expenses, and (3) provides probable future benefits. However,

expenditures that merely maintain the asset at an acceptable level of

productivity are expensed as they are incurred. Thus, the costs of

painting the building are routine, minor outlays that should be expensed

immediately.

Question: 261 Lakeside Electric purchased a truck for $38,600 to transport equipment to various job sites. For this purpose, storage bins were welded to the truck bed at a cost of $1,700. Doug Lombardi, controller of Lakeside, estimates the useful life of the truck to be 5 years and the residual value to be $1,000. Using the double-declining balance method, the depreciation expense on the truck for its second year of use is

A. $9,024

B. $9,264

C. $9,432

D. $9,672

Answer (D) is correct. Under the double-declining balance method, the full cost of the asset, or

$40,300 ($38,600 + $1,700), is depreciated, but not below salvage value.

Because the straight-line rate for a 5-year asset is 20% (100% ÷ 5), the

double-declining balance rate is 40% (20% × 2). The first year’s

depreciation is $16,120 ($40,300 × 40%), leaving a carrying amount for

the second year of $24,180 ($40,300 – $16,120). The second year’s

depreciation is thus $9,672 ($24,180 × 40%).

Gleim 2015 | Part 1 | Online MCQs | Unit 003

www.facebook.com/CMA.Arabwebsoft Page 9

حصري | لمنتدى عرب ويب سوفت

Question: 262 Albright Company uses the sum-of-the-years’-digits (SYD) method of depreciation. On January 1, the company purchased a machine for $50,000. It had an estimated life of 5 years and no residual value. Depreciation for the first year would be

A. $10,000

B. $15,000

C. $16,667

Answer (C) is correct. The SYD method multiplies a constant depreciable base (cost minus

residual value) by a declining fraction. The numerator is the number of

years of the useful life minus the years elapsed (5 – 0 = 5). The

denominator is the sum of the digits of the years in the asset’s useful life

(1 + 2 + 3 + 4 + 5). The first year’s depreciation expense is therefore

$16,667 [$50,000 × (5 ÷ 15)].

D. $20,000

Question: 263 When a fixed plant asset with a 5-year estimated useful life is sold during the second year, how would the use of an accelerated depreciation method instead of the straight-line method affect the gain or loss on the sale of the fixed plant asset?

Gain Loss

A. Increase Increase

B. Increase Decrease

Answer (B) is correct. An accelerated method reduces the carrying amount of the asset more

rapidly in the early years of the useful life than does the straight-line

method. Hence, the effect of an early sale is to increase the gain or

decrease the loss that would have been recognized under the straight-line

method.

C. Decrease Increase

D. Decrease Decrease

Question: 264 Which one of the following methods of depreciation will result in the lowest reported net income in the early life of a depreciable asset?

A. Composite depreciation method.

B. Group depreciation method.

C. Straight-line depreciation method.

D. Sum-of-the-years’-digits depreciation method.

Answer (D) is correct. Sum-of-the-years’-digits depreciation has the highest depreciation

expense in the early years of an asset’s life, resulting in lower net

Gleim 2015 | Part 1 | Online MCQs | Unit 003

www.facebook.com/CMA.Arabwebsoft Page 11

حصري | لمنتدى عرب ويب سوفت

income.

Question: 265 Silken, Inc., a distributor of silk goods, is in its first year of operation. The company has purchased ten computers at $3,500 each with an estimated life of 6 years; five desks at $500 each with an estimated life of 10 years; and two word processors at $300 each, with an estimated life of 4 years. No residual value is anticipated for any of these assets. Silken wants to adopt a depreciation method that will be easy to use and reflect an appropriate depreciation expense for the business each accounting period. The most appropriate method would be

A. Composite depreciation.

Answer (A) is correct. Group and composite depreciation methods use the straight-line

technique for an aggregate of assets. The composite method is used for

dissimilar assets.

B. Group depreciation.

C. Inventory method.

D. Replacement method.

Question: 266 In which of the following situations is the units-of-production method of depreciation most appropriate?

A. An asset’s service potential declines with use.

Answer (A) is correct. The units-of-production depreciation method allocates asset cost based

on the level of production. As production varies, so will the credit to

accumulated depreciation. Consequently, when an asset’s service

potential declines with use, the units-of-production method is the most

appropriate method.

B. An asset’s service potential declines with the passage of time.

C. An asset is subject to rapid obsolescence.

D. An asset incurs increasing repairs and maintenance with use.

Question: 267 Under IFRS, according to the revaluation model, an item of property, plant, and equipment must be carried at

A. Cost minus any accumulated depreciation.

B. Cost minus residual value.

C. Fair value minus any subsequent accumulated depreciation and impairment

losses.

Answer (C) is correct. Under the revaluation model, if the fair value of an item of property,

plant, and equipment can be reliably measured, it must be carried

Gleim 2015 | Part 1 | Online MCQs | Unit 003

www.facebook.com/CMA.Arabwebsoft Page 11

حصري | لمنتدى عرب ويب سوفت

subsequent to initial recognition at a revalued amount. This amount is

fair value at the date of the revaluation minus any subsequent

accumulated depreciation and impairment losses. The revaluation model

is permitted by IFRS, not U.S. GAAP.

D. The lower of cost or net realizable value.

Question: 268 Merry Co. purchased a machine costing $125,000 for its manufacturing operations and paid shipping costs of $20,000. Merry spent an additional $10,000 testing and preparing the machine for use. What amount should Merry record as the cost of the machine?

A. $155,000

Answer (A) is correct. The amount to be recorded as the acquisition cost of a machine includes

all costs necessary to prepare it for its intended use. Thus, the cost of a

machine used in the manufacturing operations of a company includes the

cost of testing and preparing the machine for use and the shipping costs.

The acquisition cost is $155,000 ($125,000 + $20,000 + $10,000).

B. $145,000

C. $135,000

D. $125,000

Question: 269 According to IFRS, which accounting policy may an entity apply to measure investment property in periods subsequent to initial recognition?

A. Cost model or revaluation model.

B. Cost model or fair value model.

Answer (B) is correct. An entity may choose either the cost model or the fair value model as its

accounting policy. But it must apply that policy to all of its investment

property. Under the cost model, investment property is carried at its cost

minus any accumulated depreciation and impairment losses. Under the

fair value model, investment property is measured at fair value, and gain

or loss from a change in its fair value is recognized immediately in profit

or loss.

C. Fair value model only.

D. Fair value model or revaluation model.

Question: 270 An expenditure to install an improved electrical system is a

Capital Expenditure Revenue Expenditure

A. No Yes

B. No No

C. Yes No

Gleim 2015 | Part 1 | Online MCQs | Unit 003

www.facebook.com/CMA.Arabwebsoft Page 12

حصري | لمنتدى عرب ويب سوفت

Answer (C) is correct. A betterment (improvement) occurs when a replacement asset is

substituted for an existing asset, and the result is increased productivity,

capacity, or expected useful life. If the improvement benefits future

periods, it should be capitalized.

D. Yes Yes

Question: 271 Brown Systems began operating January 1 and spent $900,000 in the first month of operations on the following items:

January advertising campaign $ 40,000

Computer equipment 280,000

12-month insurance policy 120,000

January building rent 60,000

January salaries 340,000

Office supplies 10,000

Automobile 30,000

January utilities 20,000

Total $ 900,000

The total cash expenditures that should be capitalized as property, plant, and equipment is

A. $80,000

B. $310,000

Answer (B) is correct. The assets purchased that are capitalized as property, plant, and

equipment (PPE) are the computer equipment and automobile. Therefore,

PPE is $310,000 ($280,000 + $30,000).

C. $370,000

D. $440,000

Question: 272 An entity sells a piece of machinery, for cash, prior to the end of its estimated useful life. The sale price is less than the carrying amount of the asset on the date of sale. The entry that the entity uses to record the sale is

A. Cash

Accumulated depreciation -- machinery

Loss on disposal of machinery

Machinery

Answer (A) is correct.

Gleim 2015 | Part 1 | Online MCQs | Unit 003

www.facebook.com/CMA.Arabwebsoft Page 13

حصري | لمنتدى عرب ويب سوفت

Cash is debited for the amount of the sale proceeds. Machinery and the

related accumulated depreciation are eliminated by a credit and a debit,

respectively. Because the sale price was less than the carrying amount of

the asset on the date of sale, a loss on disposal should be recognized in

net income or loss.

B. Cash

Accumulated depreciation -- machinery

Gain on disposal of machinery

Machinery

C. Cash

Expense -- disposal of machinery

Accumulated depreciation -- machinery

Machinery

D. Cash

Machinery

Accumulated depreciation -- machinery

Gain on disposal of machinery

Question: 273 An entity purchased a machine for $700,000. The machine was depreciated using the straight-line method and had a residual value of $40,000. The machine was sold on December 31, Year 1. The accumulated depreciation related to the machine was $495,000 on that date. The entity reported a gain on the sale of the machine of $75,000 in its income statement for the fiscal year ending December 31, Year 1. The selling price of the machine was

A. $280,000

Answer (A) is correct. The selling price minus the carrying amount of the machine equals the

gain or loss on disposal. The carrying amount equals $205,000 ($700,000

historical cost – $495,000 accumulated depreciation). Thus, the selling

price was $280,000 ($205,000 + $75,000 gain).

B. $240,000

C. $205,000

D. $115,000

Question: 274 What is the journal entry recorded upon the sale of an item of property, plant, and equipment (PPE) that was sold for cash in excess of its carrying amount?

A. No journal entry is required.

B. Debit cash

Debit accumulated depreciation

Debit income on disposal of PPE

Gleim 2015 | Part 1 | Online MCQs | Unit 003

www.facebook.com/CMA.Arabwebsoft Page 14

حصري | لمنتدى عرب ويب سوفت

Credit PPE

C. Debit cash

Debit PPE

Credit accumulated depreciation

Credit income on disposal of PPE

D. Debit cash

Debit accumulated depreciation

Credit PPE

Credit income on disposal of PPE

Answer (D) is correct. The journal entry to record the sale of an item of PPE for cash in excess

of its carrying amount should debit the cash account to record the sale

proceeds received. Accumulated depreciation should be eliminated by

debiting an amount equal to depreciation accumulated up to the start of

the current accounting period plus any depreciation that has accumulated

between the start of the current period and the date of disposal. Finally,

the PPE account should be credited to eliminate the original cost of the

asset. The gain should be recorded as a credit and recognized as income

on the income statement.

Question: 275 An entity sold a depreciable asset in the middle of the fifth year of its estimated 10-year useful life. The original cost of the asset was $100,000, and it was being depreciated on the straight-line basis. If the asset was sold for $80,000, the gain on the sale will be

A. $20,000

B. $25,000

Answer (B) is correct. The gain on the sale is the difference between the sale proceeds and the

carrying amount of the asset (its remaining undepreciated cost).

Depreciation must be taken up to the time of sale. Assuming that residual

value is $0, annual depreciation is $10,000 ($100,000 ÷ 10 years). Thus,

the gain is $25,000 {$80,000 sale proceeds – [$100,000 historical cost –

($10,000 × 4.5 years)]}.

C. $30,000

D. $35,000

Question: 276 To determine whether to recognize the impairment of a depreciable fixed asset, a company must compare the

A. Carrying amount of the asset and the present value of the future cash flows

expected to be generated by the asset.

B. Original cost of the asset and the fair value of the asset.

C. Carrying amount of the asset and the undiscounted future cash flows

expected to be generated by the asset.

Gleim 2015 | Part 1 | Online MCQs | Unit 003

www.facebook.com/CMA.Arabwebsoft Page 15

حصري | لمنتدى عرب ويب سوفت

Answer (C) is correct. A long-lived asset (asset group) is impaired when its carrying amount is

greater than its fair value. However, a loss equal to this excess is

recognized for the impairment only when the carrying amount is not

recoverable. The carrying amount is not recoverable when it exceeds the

sum of the undiscounted cash flows expected from the use and

disposition of the asset (asset group).

D. Original cost of the asset and the carrying amount of the asset.

Fact Pattern: Blake Corporation has determined that one of its machines has experienced an impairment in value. However, the company expects to continue to use the asset for another 3 full years because no active market exists for this machine. Selected information on the impaired asset (on the date that impairment was determined to exist) is provided below.

Original cost of the machine $22,000

Carrying amount of the machine 20,000

Undiscounted future cash flows expected to be generated by the machine 15,000

Fair value of the machine (determined by calculating the present value of the future cash flows expected to be generated by the machine) 12,000

Question: 277 After recognition of the impairment loss, Blake’s carrying amount of the impaired asset will be

A. $0

B. $12,000

Answer (B) is correct. A long-lived asset (asset group) is impaired when its carrying amount is

greater than its fair value. However, a loss equal to this excess is

recognized for the impairment only when the carrying amount is not

recoverable. The carrying amount is not recoverable when it exceeds the

sum of the undiscounted cash flows expected from the use and

disposition of the asset (asset group).

The asset is impaired because its carrying amount ($20,000) exceeds its

fair value ($12,000). This loss ($20,000 – $12,000 = $8,000) is

recognized in full because the carrying amount ($20,000) exceeds the

undiscounted cash flows from the asset ($15,000). Thus, the carrying

amount is reduced to $12,000.

C. $14,000

D. $15,000

Gleim 2015 | Part 1 | Online MCQs | Unit 003

www.facebook.com/CMA.Arabwebsoft Page 16

حصري | لمنتدى عرب ويب سوفت

Fact Pattern: Blake Corporation has determined that one of its machines has experienced an impairment in value. However, the company expects to continue to use the asset for another 3 full years because no active market exists for this machine. Selected information on the impaired asset (on the date that impairment was determined to exist) is provided below.

Original cost of the machine $22,000

Carrying amount of the machine 20,000

Undiscounted future cash flows expected to be generated by the machine 15,000

Fair value of the machine (determined by calculating the present value of the future cash flows expected to be generated by the machine) 12,000

Question: 278 What is the amount of the impairment loss to be recorded by Blake?

A. $3,000

B. $5,000

C. $7,000

D. $8,000

Answer (D) is correct. The impairment loss is the difference between the carrying amount and

fair value of the asset ($20,000 – $12,000 = $8,000).

Question: 279 An entity purchased a machine on January 1, Year 1, for $1,000,000. The machine had an estimated useful life of 9 years and a residual value of $100,000. The company uses straight-line depreciation. On December 31, Year 4, the machine was sold for $535,000. The gain or loss that should be recorded on the disposal of this machine is

A. $35,000 gain.

B. $65,000 loss.

Answer (B) is correct. The accumulated depreciation was $400,000 {[($1,000,000 historical

cost – $100,000 residual value) ÷ 9 years estimated useful life] × 4

years}, so the carrying amount was $600,000 ($1,000,000 – $400,000).

Thus, the loss was $65,000 ($600,000 carrying amount – $535,000 sales

price).

C. $365,000 loss.

D. $465,000 loss.

Question: 280 Which of the following statements is true regarding impairment of long-lived assets?

A. U.S. GAAP requires a one-step impairment test, and IFRS requires a two-

step impairment test.

B. Both IFRS and U.S. GAAP permit reversal of an impairment loss in

subsequent periods.

C. Both IFRS and U.S. GAAP prohibit reversal of an impairment loss in

Gleim 2015 | Part 1 | Online MCQs | Unit 003

www.facebook.com/CMA.Arabwebsoft Page 17

حصري | لمنتدى عرب ويب سوفت

subsequent periods.

D. Under U.S. GAAP, but not IFRS, reversal of an impairment loss in

subsequent periods is prohibited.

Answer (D) is correct. Under IFRS, an impairment loss on an asset may be reversed in

subsequent periods if a change in the estimates used to measure the

recoverable amount has occurred. But an impairment loss recognized for

goodwill must not be reversed. Under U.S. GAAP, a previously

recognized impairment loss must not be reversed.

Question: 281 An entity applies IFRS. On December 31, Year 1, it estimates the following information regarding its headquarters building:

Fair value $100,000

Cost to sell $15,000

Value in use $90,000

Residual value $17,000

Net realizable value $82,000

According to the information above, what is the recoverable amount of the headquarters building on December 31, Year 1?

A. $85,000

B. $90,000

Answer (B) is correct. The recoverable amount of an asset is the higher of its fair value minus

costs to sell and its value in use. Thus, the recoverable amount is $90,000

[$90,000 value in use > ($100,000 – $15,000) FV minus costs to sell].

C. $100,000

D. $82,000

Question: 282 Testing for possible impairment of a long-lived asset (asset group) that an entity expects to hold and use is required

A. At each interim and annual balance sheet date.

B. At annual balance sheet dates only.

C. Periodically.

D. Whenever events or changes in circumstances indicate that its carrying

amount may not be recoverable.

Answer (D) is correct. A long-lived asset (asset group) is tested for recoverability whenever

Gleim 2015 | Part 1 | Online MCQs | Unit 003

www.facebook.com/CMA.Arabwebsoft Page 18

حصري | لمنتدى عرب ويب سوفت

events or changes in circumstances indicate that its carrying amount may

not be recoverable. The carrying amount is not recoverable when it

exceeds the sum of the undiscounted cash flows expected to result from

the use and disposition of the asset (asset group). If the carrying amount

is not recoverable, an impairment loss is recognized equal to the excess

of the carrying amount over the fair value.

Question: 283 A company has a long-lived asset with a carrying value of $120,000, expected future cash flows of $130,000, present value of expected future cash flows of $100,000, and a market value of $105,000. What amount of impairment loss should be reported?

A. $0

Answer (A) is correct. An impairment loss is recognized when a long-lived asset’s carrying

amount exceeds the sum of its undiscounted cash flows. Because the sum

of the undiscounted cash flows ($130,000) exceeds the carrying amount

($120,000), the carrying amount is recoverable. Thus, no impairment is

recognized.

B. $5,000

C. $15,000

D. $20,000

Question: 284 An impairment loss on a long-lived asset (asset group) to be held and used is reported by a business enterprise in

A. Discontinued operations.

B. Extraordinary items.

C. Other comprehensive income.

D. Income from continuing operations.

Answer (D) is correct. An impairment loss is included in income from continuing operations

before income taxes by a business enterprise (income from continuing

operations in the statement of activities by a not-for-profit organization).

When a subtotal for “income from operations” is reported, the

impairment loss is included.

Question: 285 Last year, Katt Co. reduced the carrying amount of its long-lived assets used in operations from $120,000 to $100,000, in connection with its annual impairment review. During the current year, Katt determined that the fair value of the same assets had increased to $130,000. Under U.S. GAAP, what amount should Katt record as restoration of previously recognized impairment loss in the current year’s financial statements?

A. $0

Answer (A) is correct. A previously recognized impairment loss may not be reversed under U.S.

Gleim 2015 | Part 1 | Online MCQs | Unit 003

www.facebook.com/CMA.Arabwebsoft Page 19

حصري | لمنتدى عرب ويب سوفت

GAAP. Under IFRS, an impairment loss (carrying amount > recoverable

amount) on an asset (except goodwill) may be reversed if a change in the

estimates used to measure the recoverable amount has occurred.

Furthermore, IFRS permit an item of property, plant, and equipment to

be carried at a revalued amount if its fair value can be measured reliably.

Thus, an increase in excess of the prior carrying amount is permitted by

IFRS.

B. $10,000

C. $20,000

D. $30,000

Question: 286 Under IFRS, an asset is impaired when its carrying amount exceeds its recoverable amount. The recoverable amount of an asset is

A. The lower of its fair value plus cost to sell or value in use.

B. The greater of its fair value plus cost to sell or value in use.

C. The lower of its fair value minus cost to sell or value in use.

D. The greater of its fair value minus cost to sell or value in use.

Answer (D) is correct. The recoverable amount of an asset is the greater of its fair value minus

cost to sell or value in use. Value in use is the present value of the asset’s

expected cash flows. The recognized impairment loss is the excess of the

asset’s carrying amount over its recoverable amount.

Question: 287 Which of the following is not considered to be an intangible asset?

A. Goods on consignment.

Answer (A) is correct. An intangible asset is an identifiable nonmonetary (nonfinancial) asset

without physical substance. Inventory is a tangible asset. Thus, goods on

consignment are not intangible assets.

B. Patents.

C. Copyrights.

D. Trademarks.

Question: 288 A recognized intangible asset is amortized over its useful life

A. Unless the pattern of consumption of the economic benefits of the asset is

not reliably determinable.

B. If that life is determined to be finite.

Answer (B) is correct.

Gleim 2015 | Part 1 | Online MCQs | Unit 003

www.facebook.com/CMA.Arabwebsoft Page 21

حصري | لمنتدى عرب ويب سوفت

A recognized intangible asset is amortized over its useful life if that

useful life is finite, that is, unless the useful life is determined to be

indefinite. The useful life of an intangible asset is indefinite if no

foreseeable limit exists on the period over which it will contribute,

directly or indirectly, to the reporting entity’s cash flows.

C. Unless the precise length of that life is not known.

D. If that life is indefinite but not infinite.

Question: 289 Hansen, Inc., purchased a patent at the beginning of Year 1 for $22,100 that was to be amortized over 17 years. On July 1 of Year 8, Hansen incurred legal costs of $11,400 to successfully defend the patent. The amount of amortization expense that Hansen should record for Year 8 is

A. $2,500

B. $1,971

C. $1,900

Answer (C) is correct. Hansen will amortize the cost of the patent on a straight-line basis at the

rate of $1,300 per year ($22,100 ÷ 17). The costs of a successful legal

defense of a patent are capitalized and amortized over the shorter of the

remaining legal life or the estimated useful life of the patent. Because the

legal costs to defend the patent were incurred when the patent had

9.5 years of life remaining, they will be amortized at a rate of $1,200 per

year ($11,400 ÷ 9.5). Because Year 8 only includes a half year’s

depreciation for the legal costs, total amortization expense for that year is

$1,900 ($1,300 + $600).

D. $1,300

Question: 290 Which of the following costs associated with an internally developed patent should be capitalized?

Research and Patent Development Registration

A. No Yes

Answer (A) is correct. R&D costs must be expensed as they are incurred. Legal fees and

registration fees are excluded from the definition of R&D. Thus, the

patent registration fees should be capitalized as a cost associated with an

internally developed patent. The patent’s R&D costs should have been

expensed as they were incurred.

B. No No

C. Yes No

D. Yes Yes

Gleim 2015 | Part 1 | Online MCQs | Unit 003

www.facebook.com/CMA.Arabwebsoft Page 21

حصري | لمنتدى عرب ويب سوفت

Question: 291 On July 1, Broadstreet Corporation acquired a patent on its new manufacturing process, which streamlines its production operation. The cost of the patent was $17,000, and Broadstreet expects that the useful life of the new process will be 10 years, although the legal life of the patent is 17 years. Broadstreet is a calendar-year corporation and is preparing its December 31 Statement of Financial Position. At which amount should the patent be reported at December 31 of the year of acquisition?

A. $15,300

B. $16,000

C. $16,150

Answer (C) is correct. A patent is amortized over the shorter of its useful life or legal life, so

annual amortization on this patent is $1,700 ($17,000 ÷ 10 years). The

depreciation expense for the year of acquisition is $850 [$1,700 × (6 ÷ 12

months)]. The patent should therefore be reported at December 31 at

$16,150 ($17,000 – $850).

D. $16,500

Question: 292 Which of the following expenditures qualifies for asset capitalization?

A. Cost of materials used in prototype testing.

B. Costs of testing a prototype and modifying its design.

C. Salaries of engineering staff developing a new product.

D. Legal costs associated with obtaining a patent on a new product.

Answer (D) is correct. Patents may be purchased or developed internally. The initial capitalized

cost of a purchased patent is normally the fair value of the consideration

given, that is, its purchase price plus incidental costs, such as registration

and attorney’s fees. Internally developed patents are less likely to be

capitalized because related R&D costs must be expensed when incurred.

Thus, only relatively minor costs can be capitalized, for example, patent

registration fees and legal fees.

Question: 293 During the year just ended, Jase Co. incurred research and development costs of $136,000 in its laboratories relating to a patent that was granted on July 1. Costs of registering the patent equaled $34,000. The patent’s legal life is 20 years, and its estimated economic life is 10 years. In its December 31 balance sheet, what amount should Jase report for the patent, net of accumulated amortization?

A. $32,300

Answer (A) is correct. R&D costs are expensed as incurred. However, legal work in connection

with patent applications or litigation and the sale or licensing of patents

are specifically excluded from the definition of R&D. Hence, the legal

costs of filing a patent should be capitalized. The patent should be

amortized over its estimated economic life of 10 years. Amortization for

Gleim 2015 | Part 1 | Online MCQs | Unit 003

www.facebook.com/CMA.Arabwebsoft Page 22

حصري | لمنتدى عرب ويب سوفت

the year equals $1,700 [($34,000 ÷ 10) × (6 ÷ 12)]. Thus, the reported

amount of the patent at year end equals $32,300 ($34,000 – $1,700).

B. $33,150

C. $161,500

D. $165,000

Question: 294 A purchased patent has a remaining legal life of 15 years. It should be

A. Expensed in the year of acquisition.

B. Amortized over 15 years regardless of its useful life.

C. Amortized over its useful life if less than 15 years.

Answer (C) is correct. The amortization period for an intangible asset distinct from goodwill is

the shorter of its useful life or the legal life remaining after acquisition.

D. Amortized over 40 years.

Question: 295 Under IFRS, an entity that acquires an intangible asset may use the revaluation model for subsequent measurement only if

A. The useful life of the intangible asset can be reliably determined.

B. An active market exists for the intangible asset.

Answer (B) is correct. An intangible asset is carried at cost minus any accumulated amortization

and impairment losses, or at a revalued amount. The revaluation model is

similar to that for items of PPE (initial recognition of an asset at cost).

However, fair value must be determined based on an active market.

C. The cost of the intangible asset can be measured reliably.

D. The intangible asset is a monetary asset.

Question: 296 Legal fees incurred by a company in defending its patent rights should be capitalized when the outcome of litigation is

Successful Unsuccessful

A. Yes Yes

B. Yes No

Answer (B) is correct. Legal fees incurred in the successful defense of a patent should be

capitalized as part of the cost of the patent and then amortized over its

remaining useful life because that useful life is finite. Legal fees incurred

in an unsuccessful defense should be expensed as the costs are incurred.

C. No No

Gleim 2015 | Part 1 | Online MCQs | Unit 003

www.facebook.com/CMA.Arabwebsoft Page 23

حصري | لمنتدى عرب ويب سوفت

D. No Yes

Question: 297 Gray Co. was granted a patent on January 2, Year 5, and appropriately capitalized $45,000 of related costs. Gray was amortizing the patent over its estimated useful life of 15 years. During Year 8, Gray paid $15,000 in legal costs in successfully defending an attempted infringement of the patent. After the legal action was completed, Gray sold the patent to the plaintiff for $75,000. Gray’s policy is to take no amortization in the year of disposal. In its Year 8 income statement, what amount should Gray report as gain from sale of patent?

A. $15,000

B. $24,000

Answer (B) is correct. The patent was capitalized at $45,000 in Year 5. Annual amortization of

$3,000 ($45,000 ÷ 15 years) for Year 5, Year 6, and Year 7 reduced the

carrying amount to $36,000. The $15,000 in legal costs for successfully

defending an attempted infringement may be capitalized, which increases

the carrying amount of the patent to $51,000 ($36,000 + $15,000).

Accordingly, the gain from the sale is $24,000 ($75,000 – $51,000).

C. $27,000

D. $39,000

Question: 298 Which of the following assets, if any, acquired this year in an exchange transaction is(are) potentially amortizable?

Goodwill Trademarks

A. No No

B. No Yes

Answer (B) is correct. Goodwill is tested for impairment at least annually but is never

amortized. Trademarks, however, may be amortized but only if they have

finite useful lives.

C. Yes Yes

D. Yes No

Question: 299 Goodwill should be tested for value impairment at which of the following levels?

A. Each identifiable long-term asset.

B. Each reporting unit.

Answer (B) is correct. The cost of an acquired entity minus the net amount assigned to assets

acquired and liabilities assumed is goodwill. Goodwill is not amortized.

However, goodwill is assigned to a reporting unit that benefited from the

business combination for the purpose of testing impairment. Testing

Gleim 2015 | Part 1 | Online MCQs | Unit 003

www.facebook.com/CMA.Arabwebsoft Page 24

حصري | لمنتدى عرب ويب سوفت

occurs each year at the same time, but different reporting units may be

tested at different times. Furthermore, additional testing also may be

indicated. Potential impairment of goodwill is deemed to exist only if the

carrying amount (including goodwill) of a reporting unit is greater than

its fair value. Thus, accounting for goodwill is based on the units of the

combined entity into which the acquired entity was absorbed. A reporting

unit is an operating segment or one of its components, that is, one level

below an operating segment. A component qualifies as a reporting unit if

(1) it is a business for which discrete financial information is available,

and (2) segment management regularly reviews its operating results.

However, similar components are aggregated. These provisions,

including the determination of operating segments, apply even if the

reporting entity is not required to report segment information.

C. Each acquisition unit.

D. Entire business as a whole.

Question: 300 A company should recognize goodwill in its balance sheet at which of the following points?

A. Costs have been incurred in the development of goodwill.

B. Goodwill has been created in the purchase of a business.

Answer (B) is correct. Goodwill can be recognized only in a business combination. Goodwill is

an asset representing the future economic benefits arising from other

assets acquired in a business combination that are not individually

identified and separately recognized.

C. The company expects a future benefit from the creation of goodwill.

D. The fair market value of the company’s assets exceeds the book value of

the company’s assets.

Question: 301 Which one of the following statements is correct about the reconciliation of U.S. GAAP and International Financial Reporting Standards (IFRS)?

A. The costs of development must be expensed under U.S. GAAP, but are

capitalized under IFRS if they meet specific criteria.

Answer (A) is correct. Under IFRS, (1) costs incurred during the research phase of an internal

project are expensed as incurred since the company cannot demonstrate

that an intangible asset exists that will generate probable future economic

benefits; and (2) costs incurred during the development phase of an

internal project can be capitalized and recognized as an intangible asset

if, and only if, the company can demonstrate all of the following:

1. The technical feasibility to complete the intangible asset

2.

Gleim 2015 | Part 1 | Online MCQs | Unit 003

www.facebook.com/CMA.Arabwebsoft Page 25

حصري | لمنتدى عرب ويب سوفت

3. Its intention to complete and use or sell the intangible

asset

4. Its ability to sell or use the intangible asset

5. Availability of resources to complete and use or sell the

intangible asset

6. The way in which the asset will generate probable future

economic benefits

7. Its ability to reliably measure expenditures attributable to

the asset

B. The costs of research must be expensed under U.S. GAAP, but are

capitalized under IFRS if they meet specific criteria.

C. All costs of research and development must be expensed under both U.S.

GAAP and IFRS.

D. Internally generated goodwill may not be capitalized under U.S. GAAP,

but it may be capitalized under IFRS.

Question: 302 Howell Corporation, a publicly traded corporation, is the lessee in a leasing agreement with Brandon, Inc. to lease land and a building. If the lease contains a bargain purchase option, Howell should record the land and the building as a(n)

A. Operating lease and capital lease, respectively.

B. Capital lease and operating lease, respectively.

C. Capital lease but recorded as a single unit.

D. Capital lease but separately classified.

Answer (D) is correct. A lessee records a lease as a capital lease if it meets any one of four

criteria. Existence of a bargain purchase option is one of these criteria. If

a lease involving land and a building contains a bargain purchase option

or if the lease transfers ownership to the lessee at the end of its term, the

lessee separately capitalizes the land and the building.

Gleim 2015 | Part 1 | Online MCQs | Unit 003

www.facebook.com/CMA.Arabwebsoft Page 26

حصري | لمنتدى عرب ويب سوفت

Fact Pattern:

On January 1, Plantation Restaurant is planning to enter as the lessee into the two lease agreements described in the columns to the right. Each lease is noncancelable, and Plantation does not receive title to either leased property during or at the end of the lease term. All payments required under these agreements are due on January 1 each year.

Lessor Hadaway,

Inc.

Cutter

Electronics

Type of property Oven Computer

Yearly rental $15,000 $4,000

Lease term 10 years 3 years

Economic life 15 years 5 years

Purchase option None $3,000

Renewal option None None

Fair market value

at

inception of lease $125,000 $10,200

Unguaranteed

residual value None $2,000

Lessee’s

incremental

borrowing rate 10% 10%

Executory costs

paid by

Lessee Lessor

Annual executory

costs

$800 $500

Present value

factor at

10% (of an

annuity due)

6.76 2.74

Question: 303 Plantation Restaurant should treat the lease agreement with Hadaway, Inc. as a(n)

A. Capital lease with an initial asset value of $101,400.

B. Operating lease, charging $14,200 in rental expense and $800 in executory

costs to annual operations.

C. Operating lease, charging the present value of the yearly rental expense to

annual operations.

D. Operating lease, charging $15,000 in rental expense and $800 in executory

costs to annual operations.

Answer (D) is correct. The Hadaway lease is an operating lease with a $15,000 annual rental

expense with annual executory costs of $800 to be paid by the lessee. An

operating lease does not transfer the rights and risks of ownership to the

lessee. The Hadaway lease is nothing more than a rental arrangement. If

any one of the following criteria is met, the lease is a capital lease: the

lease transfers title to the lessee, the lease has a bargain purchase option,

the lease term is 75% or more of the useful life of the leased asset, or the

present value of the minimum lease payments is 90% or more of the

Gleim 2015 | Part 1 | Online MCQs | Unit 003

www.facebook.com/CMA.Arabwebsoft Page 27

حصري | لمنتدى عرب ويب سوفت

asset’s fair value. The Hadaway lease meets none of these four criteria.

Fact Pattern:

On January 1, Plantation Restaurant is planning to enter as the lessee into the two lease agreements described in the columns to the right. Each lease is noncancelable, and Plantation does not receive title to either leased property during or at the end of the lease term. All payments required under these agreements are due on January 1 each year.

Lessor Hadaway,

Inc.

Cutter

Electronics

Type of property Oven Computer

Yearly rental $15,000 $4,000

Lease term 10 years 3 years

Economic life 15 years 5 years

Purchase option None $3,000

Renewal option None None

Fair market value

at

inception of lease $125,000 $10,200

Unguaranteed

residual value None $2,000

Lessee’s

incremental

borrowing rate 10% 10%

Executory costs

paid by

Lessee Lessor

Annual executory

costs

$800 $500

Present value

factor at

10% (of an

annuity due)

6.76 2.74

Question: 304 Plantation Restaurant should treat the lease agreement with Cutter Electronics as a(n)

A. Capital lease with an initial asset value of $10,960.

B. Capital lease with an initial asset value of $10,200.

C. Operating lease, charging $3,500 in rental expense and $500 in executory

costs to annual operations.

D. Capital lease with an initial asset value of $9,590.

Answer (D) is correct. A capital lease is one in which many of the benefits and risks of

ownership are transferred to the lessee. For accounting purposes, the

lessee treats a capital lease as similar to the purchase of an asset. If the

present value of the minimum lease payments (excluding executory

costs) is 90% or more of the asset’s fair value, the lease should be

accounted for as a capital lease. Given that the executory costs associated

with the lease are to be paid by the lessor, a portion of the lease rental

Gleim 2015 | Part 1 | Online MCQs | Unit 003

www.facebook.com/CMA.Arabwebsoft Page 28

حصري | لمنتدى عرب ويب سوفت

price is for those costs, not for the asset. Executory costs include

insurance, maintenance, and similar expenses. Consequently, the annual

minimum lease payment equals the annual payment minus the executory

costs, or $3,500 ($4,000 yearly rental – $500). The present value of the

minimum lease payments is therefore $9,590 ($3,500 × 2.74), which is

greater than 90% of the fair value of the asset. Thus, the lease should be

capitalized. The appropriate amount of the initial asset value is the

present value of the minimum lease payments calculated above.

Question: 305 Which of the following statements about a capital lease is false?

A. The lessor capitalizes the net investment in the lease.

B. The lessor records the leased item as an asset.

Answer (B) is correct. When a lease is capitalized, the lessor derecognizes the leased item and

records lease payments receivable. The lessee records and depreciates the

leased item.

C. The lessee records depreciation or capital cost allowance on the leased

asset.

D. The lease arrangement represents a form of financing.

Question: 306 If a company uses off-balance-sheet financing, assets have been acquired

A. For cash.

B. With operating leases.

Answer (B) is correct. With an operating lease, no long-term liability need be reported on the

face of the balance sheet.

C. With financing leases.

D. With a line of credit.

Question: 307 Which one of the following statements with respect to leases is correct?

A. An operating lease is treated like a rental contract between the lessor and

lessee.

Answer (A) is correct. An operating lease is a transaction in which the lessee rents the right to

use the lessor’s assets without acquiring a substantial portion of the

benefits and risks of ownership. Thus, an operating lease is treated like a

rental contract between the lessor and lessee.

B. A lease that does not transfer ownership from the lessor to the lessee by the

end of the lease is automatically an operating lease.

C. Sales and direct financing leases pertain more to lessees than lessors.

Gleim 2015 | Part 1 | Online MCQs | Unit 003

www.facebook.com/CMA.Arabwebsoft Page 29

حصري | لمنتدى عرب ويب سوفت

D. Unpredictability of lease revenues or expenses can transform what would

otherwise be a capital lease for the lessee into an operating lease for the

lessee.

Fact Pattern: Neary Company has entered into a contract to lease computers from Baldwin Company starting on January 1, Year 1. Relevant information pertaining to the lease is provided below.

Lease term 4 Years

Useful life of computers 5 Years

Present value of future lease payments $100,000

Fair value of leased asset on date of lease 105,000

Baldwin’s implicit rate 10%

At the end of the lease term, ownership of the asset transfers from Baldwin to Neary. Neary has properly

n line depreciation o-classified this lease as a capital lease on its financial statements and uses straight

comparable assets.

Question: 308 At January 1, Year 1, the lease would be reported on Neary’s books as a(n)

A. Asset only.

B. Asset and a liability.

Answer (B) is correct. The lease is classified as a capital lease, since the ownership of the leased

asset is transferred to the lessee at the end of the lease term. The lessee

must record a capital lease as an asset and as an obligation at an amount

equal to the present value of the minimum lease payments.

C. Liability only.

D. Expense and a liability.

Fact Pattern: Neary Company has entered into a contract to lease computers from Baldwin Company starting on January 1, Year 1. Relevant information pertaining to the lease is provided below.

Lease term 4 Years

Useful life of computers 5 Years

Present value of future lease payments $100,000

Fair value of leased asset on date of lease 105,000

Baldwin’s implicit rate 10%

At the end of the lease term, ownership of the asset transfers from Baldwin to Neary. Neary has properly

line depreciation on -s a capital lease on its financial statements and uses straightclassified this lease a

comparable assets.

Gleim 2015 | Part 1 | Online MCQs | Unit 003

www.facebook.com/CMA.Arabwebsoft Page 31

حصري | لمنتدى عرب ويب سوفت

Question: 309 What is the annual depreciation expense that Neary will record on the leased computers?

A. $20,000

Answer (A) is correct. Under a capital lease, the lessee recognizes a leased asset at an amount

equal to the present value of the minimum lease payments ($100,000).

Since the lease provides for the transfer of ownership, Neary should

depreciate the computers using the straight-line method over their

estimated useful life (5 years). Annual depreciation expense on the

computers is $20,000 ($100,000 ÷ 5 years).

B. $21,000

C. $25,000

D. $26,250

Question: 310 On January 1, Rosewater Company leased a computer for 4 years at a monthly rent of $80, payable at the end of each month. Due to the rate of technical change, the computer is expected to become obsolete within 5 years. At the inception of the lease, the computer was retailing for $3,450. Had Rosewater chosen to purchase the computer instead of leasing it, they could have borrowed the funds at 10%. At a 10% interest rate, the present value of the lease payments is $3,154. Rosewater does not know the rate implicit in the lease. For the month of January, Rosewater should report (to the closest dollar) interest expense of

A. $26 and depreciation expense of $66.

Answer (A) is correct. Interest expense will be recognized in the amount of $26 [$3,154 × 10%

× (1 ÷ 12 months)]. Since Rosewater does not know the lessor’s implicit

rate, it is appropriate to use Rosewater’s own incremental borrowing rate

to determine whether the lease should be classified as a capital lease.

Since the present value of the lease payments is greater than 90% of the

fair value of the computer ($3,154 ÷ $3,450 = 91.4%), the lease is

appropriately classified as a capital lease and Rosewater will recognize

depreciation expense. Since the lease agreement neither provides for

transfer of ownership nor contains a bargain purchase option, the

computers are depreciated over the lease term (4 years). Monthly

depreciation expense will be $66 ($3,154 ÷ 48 months).

B. $0 and rent expense of $80.

C. $29 and depreciation expense of $58.

D. $29 and rent expense of $80.

Question: 311 Keller Corporation signed a 3-year lease for an automobile on December 1. The automobile had a list price of $17,000 and an estimated useful life of 8 years. The lease called for payments of $500 per month for 36 months. The present value of the $500 payments was $15,054 at Keller’s incremental borrowing rate and $15,496 at the lessor’s implicit rate, which is known to the lessee. Based on the above information, Keller should record the lease as a(n)

A. Capital lease.

Gleim 2015 | Part 1 | Online MCQs | Unit 003

www.facebook.com/CMA.Arabwebsoft Page 31

حصري | لمنتدى عرب ويب سوفت

Answer (A) is correct. A lessee must report a lease as a capital lease if the present value of the

minimum lease payments (MLP) is at least 90% of the fair value of the

asset. If the lessor’s implicit rate is known to the lessee, that is the

appropriate rate for discounting the MLP. Dividing the present value of

the MLP by the list price of the automobile yields a result > 90%

($15,496 ÷ $17,000 = 91.2%). Thus, this lease must be classified by

Keller as a capital lease.

B. Operating lease.

C. Sale-leaseback.

D. Sales-type lease.

Question: 312 Lease M does not contain a bargain purchase option, but the lease term is equal to 90% of the estimated economic life of the leased property. Lease P does not transfer ownership of the property to the lessee by the end of the lease term, but the lease term is equal to 75% of the estimated economic life of the leased property. How should the lessee classify these leases?

Lease M Lease P

A. Capital lease Operating lease

B. Capital lease Capital lease

Answer (B) is correct. For a lease to be classified as a capital lease by the lessee, any one of

four criteria must be met. One of these criteria is that the lease term equal

75% or more of the estimated remaining economic life of the leased

property. Both leases meet the 75% criterion and should be properly

classified as capital leases.

C. Operating lease Capital lease

D. Operating lease Operating lease

Question: 313 Bain Co. entered into a 10-year lease agreement for a new piece of equipment worth $500,000. At the end of the lease, Bain will have the option to purchase the equipment. Which of the following would require the lease to be accounted for as a capital lease?

A. The lease includes an option to purchase stock in the company.

B. The estimated useful life of the leased asset is 12 years.

Answer (B) is correct. A lease is classified as a capital lease by the lessee if, at its inception, any

of the following four criteria are satisfied: (1) the lease provides for the

transfer of ownership of the leased property, (2) the lease contains a

bargain purchase option, (3) the lease term is 75% or more of the

estimated economic life of the leased property, or (4) the present value of

the minimum lease payments is at least 90% of the fair value of the

leased property to the lessor. Because the lease is for 83 1/3% (10 ÷ 12)

Gleim 2015 | Part 1 | Online MCQs | Unit 003

www.facebook.com/CMA.Arabwebsoft Page 32

حصري | لمنتدى عرب ويب سوفت

of the estimated economic life of the leased property, Bain must

capitalize the lease.

C. The present value of the minimum lease payments is $400,000.

D. The purchase option at the end of the lease is at fair market value.

Question: 314 Which of the following is a criterion for a lease to be classified as a capital lease in the books of a lessee?

A. The lease contains a bargain purchase option.

Answer (A) is correct. A lease is classified as a capital lease by the lessee if, at its inception, any

of the following four criteria is satisfied: (1) the lease provides for the

transfer of ownership of the leased property, (2) the lease contains a

bargain purchase option, (3) the lease term is 75% or more of the

estimated economic life of the leased property, or (4) the present value of

the minimum lease payments (excluding executory costs) is at least 90%

of the fair value of the leased property to the lessor.

B. The lease does not transfer ownership of the property to the lessee.

C. The lease term is equal to 65% or more of the estimated useful life of the

leased property.

D. The present value of the minimum lease payments is 70% or more of the

fair market value of the leased property.

Question: 315 Which of the following is a characteristic of a capital lease?

A. The lease term is substantially less than the estimated economic life of the

leased property.

B. The lease contains a bargain-purchase option.

Answer (B) is correct. A lessee capitalizes a lease that contains a BPO. A lessor capitalizes a

lease that contains a BPO if (1) collectibility of the remaining payments

is reasonably predictable and (2) no material uncertainties exist regarding

unreimbursable costs to be incurred by the lessor.

C. The present value of the minimum lease payments at the beginning of the

lease term is 75% or more of the fair value of the property at the inception

of the lease.

D. The future obligation does not appear in the balance sheet of the lessee.

Gleim 2015 | Part 1 | Online MCQs | Unit 003

www.facebook.com/CMA.Arabwebsoft Page 33

حصري | لمنتدى عرب ويب سوفت

Question: 316 The present value of minimum lease payments should be used by the lessee in determining the amount of a lease liability under a lease classified by the lessee as a(n)

Capital Lease Operating Lease

A. Yes Yes

B. Yes No

Answer (B) is correct. The lessee must record a capital lease as an asset and an obligation at an

amount equal to the present value of the minimum lease payments.

Under an operating lease, the lessee records no liability except for rental

expense accrued at the end of an accounting period. The accrual is at

settlement value rather than present value.

C. No No

D. No Yes

Question: 317 Koby Co. entered into a capital lease with a vendor for equipment on January 2 for 7 years. The equipment has no guaranteed residual value. The lease required Koby to pay $500,000 annually on January 2, beginning with the current year. The present value of an annuity due for seven years was 5.35 at the inception of the lease. What amount should Koby capitalize as leased equipment?

A. $500,000

B. $825,000

C. $2,675,000

Answer (C) is correct. The lessee records a capital lease as an asset and an obligation at the

present value of the minimum lease payments. These payments include

the initial payment at the inception of the lease. Thus, the annual

payments constitute an annuity due. In the absence of a bargain purchase

option, guaranteed residual value, or nonrenewal penalty, the amount

capitalized as leased equipment is $2,675,000 ($500,000 annual payment

× 5.35 present value of an annuity due for 7 years).

D. $3,500,000

Question: 318 Quick Company’s lease payments are made at the end of each period. Quick’s liability for a capital lease will be reduced periodically by the

A. Minimum lease payment less the portion of the minimum lease payment

allocable to interest.

Answer (A) is correct. The lease liability consists of the present value of the minimum lease

payments. The lease liability is reduced by the portion of the lease

payment attributable to the lease liability. This amount is the lease

payment less the interest component of the payment. Thus, the liability is

decreased by the minimum lease payment each period less the portion of

Gleim 2015 | Part 1 | Online MCQs | Unit 003

www.facebook.com/CMA.Arabwebsoft Page 34

حصري | لمنتدى عرب ويب سوفت

the payment allocable to interest.

B. Minimum lease payment plus the amortization of the related asset.

C. Minimum lease payment less the amortization of the related asset.

D. Minimum lease payment.

Question: 319 On January 1, Year 4, Harrow Co., as lessee, signed a 5-year noncancelable equipment lease with annual payments of $100,000 beginning December 31, Year 4. Harrow treated this transaction as a capital lease. The five lease payments have a present value of $379,000 at January 1, Year 4, based on interest of 10%. What amount should Harrow report as interest expense for the year ended December 31, Year 4?

A. $37,900

Answer (A) is correct. Under the effective-interest method, interest expense for the first year is

$37,900 ($379,000 lease obligation × 10% effective interest rate).

B. $27,900

C. $24,200

D. $0

Question: 320 On January 2, Cole Co. signed an 8-year noncancelable lease for a new machine, requiring $15,000 annual payments at the beginning of each year. The machine has a useful life of 12 years with no salvage value. Title passes to Cole at the lease expiration date. Cole uses straight-line depreciation for all of its plant assets. Aggregate lease payments have a present value on January 2 of $108,000, based on an appropriate rate of interest. For the current year, Cole should record depreciation (amortization) expense for the leased machine at

A. $0

B. $9,000

Answer (B) is correct. This lease qualifies as a capital lease because title passes to the lessee at

the end of the lease term. When a lease is capitalized because title passes

to the lessee at the end of the lease term or because the lease contains a

bargain purchase option, the depreciation (amortization) period is the

estimated economic life of the asset. The asset should be depreciated

(amortized) in accordance with the lessee’s normal depreciation policy

for owned assets. Cole normally uses the straight-line method. Thus,

depreciation (amortization) expense is $9,000 [($108,000 leased asset –

$0 salvage value) ÷ 12-year economic life].

C. $13,500

D. $15,000

Gleim 2015 | Part 1 | Online MCQs | Unit 003

www.facebook.com/CMA.Arabwebsoft Page 35

حصري | لمنتدى عرب ويب سوفت

Question: 321 Leases should be classified by the lessee as either operating leases or capital leases. Which of the following statements best characterizes operating leases?

A. The benefits and risks of ownership are transferred from the lessor to the

lessee.

B. The lessee records leased property as an asset and the present value of the

lease payments as a liability.

C. Operating leases transfer ownership to the lessee, contain a bargain

purchase option, are for more than 75% of the leased asset’s useful life, or

have minimum lease payments with a present value in excess of 90% of the

fair value of the leased asset.

D. The lessor records lease revenue, asset depreciation, maintenance, etc., and

the lessee records lease payments as rental expense.

Answer (D) is correct. Operating leases are transactions whereby lessees rent the right to use

lessor assets without acquiring a substantial portion of the benefits and

risks of ownership of those assets.

Question: 322 Which one of the following temporary differences will result in a deferred tax asset?

A. Use of the straight-line depreciation method for financial statement

purposes and the Modified Accelerated Cost Recovery System (MACRS)

for income tax purposes.

B. Installment sale profits accounted for on the accrual basis for financial

statement purposes and on a cash basis for income tax purposes.

C. Advance rental receipts accounted for on the accrual basis for financial

statement purposes and on a cash basis for tax purposes.

Answer (C) is correct. A deferred tax asset records the deferred tax consequences attributable to

deductible temporary differences and carryforwards. Advance rental

receipts accounted for on the accrual basis for financial statement

purposes and on a cash basis for tax purposes would give rise to a

deferred tax asset. The financial statements would report no income and

no related tax expense because the rental payments apply to future

periods. The tax return, however, would treat the rent as income when

the cash was received, and a tax would be due in the year of receipt.

Because the tax is paid prior to recording the income for financial

statement purposes, it represents an asset that will be recognized as an

expense when income is finally recorded.

D. Investment gains accounted for under the equity method for financial

statement purposes and under the cost method for income tax purposes.

Gleim 2015 | Part 1 | Online MCQs | Unit 003

www.facebook.com/CMA.Arabwebsoft Page 36

حصري | لمنتدى عرب ويب سوفت

Fact Pattern:

Bearings Manufacturing Company, Inc. purchased a new machine on January 1, Year 1, for $100,000. The company uses the straight-line depreciation method with an estimated equipment life of 5 years and a zero salvage value for financial statement purposes, and uses the 3-year, Modified Accelerated Cost Recovery System (MACRS) with an estimated equipment life of 3 years for income tax reporting purposes. Bearings is subject to a 35% marginal income tax rate.

Assume that the deferred tax liability at the beginning of the year is zero and that Bearings has a positive earnings tax position. The MACRS depreciation rates for 3-year equipment are shown below.

Year Rate

1 33.33%

2 44.45

3 14.81

4 7.41

Question: 323 What is the deferred tax liability at December 31, Year 1 (rounded to the nearest whole dollar)?

A. $7,000

B. $33,330

C. $11,666

D. $4,666

Answer (D) is correct. For financial reporting purposes, the reported amount (cost –

accumulated depreciation) of the machine at year-end, assuming straight-

line depreciation and no salvage value, will be $80,000 [$100,000 cost –

($100,000 ÷ 5 years)]. The tax basis of this asset will be $66,670

[$100,000 – ($100,000 × 33.33%)]. A taxable temporary difference has

arisen because the excess of the reported amount over the tax basis will

result in a net future taxable amount over the recovery period. A taxable

temporary difference requires recognition of a deferred tax liability.

Assuming the 35% rate applies during the asset’s entire life, the deferred

tax liability equals the applicable enacted tax rate times the temporary

difference, or $4,666 [($80,000 – $66,670) × 35%].

Question: 324 On a statement of financial position, all of the following should be classified as current liabilities except

A. Advances from customers for services to be performed.

B. Salaries payable for work performed during the previous month.

C. Deferred income taxes for differences based on depreciation methods.

Answer (C) is correct. Deferred tax amounts are classified as current or noncurrent based on the

classification of the related asset or liability (assuming such an asset or

liability exists). Because depreciable assets are noncurrent, a deferred tax

liability for differences based on depreciation methods is noncurrent.

D. Accounts payable for inventory items to be shipped on consignment.

Gleim 2015 | Part 1 | Online MCQs | Unit 003

www.facebook.com/CMA.Arabwebsoft Page 37

حصري | لمنتدى عرب ويب سوفت

Question: 325 A liability that represents the accumulated difference between the income tax expense reported on the firm’s books and the income tax actually paid is

A. Capital gains tax.

B. Deferred taxes.

Answer (B) is correct. Deferred tax liabilities arise when temporary differences in book and

taxable income result in future taxable amounts. Deferred tax assets arise

when temporary differences in book and taxable income result in future

deductible amounts.

C. Taxes payable.

D. Value-added taxes.

Question: 326 Harrison Corporation entered into a 3-year contract, using the percentage-of-completion method for financial income and the completed contract method for taxable income. Harrison expected the project to be profitable throughout the construction period. The effect on Harrison’s financial statements for the third year of this contract would be a(n)

A. Decrease in the deferred tax asset account.

B. Decrease in the deferred tax liability account.

Answer (B) is correct. For the first two years of the contract, Harrison reports more revenue for

financial reporting purposes than for tax purposes, giving rise to a

deferred tax liability. Upon completion of the contract, Harrison reports

all the revenue on its tax return, thereby decreasing the deferred tax

liability.

C. Increase in the deferred tax asset account.

D. Increase in the deferred tax liability account.

Question: 327 A tax rate other than the current tax rate may be used to calculate the deferred income tax amount on the statement of financial position if a(n)

A. Future tax rate has been enacted into law.

Answer (A) is correct. A tax rate other than the current tax rate may be used to calculate the

deferred income tax amount on the statement of financial position if a

future tax rate has been enacted into law.

B. Future tax rate change is considered more likely than not to occur.

C. Election has been made to apply past tax rates.

D. Net operating loss carryback exists.

Gleim 2015 | Part 1 | Online MCQs | Unit 003

www.facebook.com/CMA.Arabwebsoft Page 38

حصري | لمنتدى عرب ويب سوفت

Question: 328 Selected financial information for Windham, Inc., for the year just ended is shown below.

Pretax income $5,000,000

Interest received on municipal bonds 600,000

Gain on the sale of land reported this

year but not taxable until next year 1,000,000

Tax rate for all years 40%

Beginning balances:

Income taxes payable -0-

Deferred tax liability $50,000

The total income tax expense reported on Windham’s income statement for the year just ended should be

A. $960,000

B. $1,360,000

C. $1,760,000

Answer (C) is correct. Taxable income consists of pretax income adjusted for those items that

give rise to tax differences. Taxable income is therefore $3,400,000

($5,000,000 – $600,000 – $1,000,000), and current tax expense is

$1,360,000 ($3,400,000 × 40%). The interest on municipal bonds is a

permanent difference because it is tax-exempt, i.e., it is recognized in

GAAP income but never in taxable income. Permanent differences have

no deferred tax effects. However, the gain on the sale of land is a

temporary difference because it is included in GAAP income this year

and is included in taxable income in the future. This temporary

difference gives rise to a future taxable amount, specifically, a $400,000

deferred tax liability ($1,000,000 × 40%). This credit to the deferred tax

liability account is balanced by a debit to income tax expense. Total

income tax expense for the year is therefore $1,760,000 ($1,360,000

current portion + $400,000 deferred portion).

D. $2,640,000

Fact Pattern: Lucas Company computed the following deferred tax balances for the 2 most recent years. Deferred tax assets are considered fully realizable.

Year 1 Year 2

Current deferred tax assets $3,000 $10,000

Noncurrent deferred tax assets 6,000 7,000

Current deferred tax liabilities 8,000 9,000

Noncurrent deferred tax liabilities 5,000 14,000

Gleim 2015 | Part 1 | Online MCQs | Unit 003

www.facebook.com/CMA.Arabwebsoft Page 39

حصري | لمنتدى عرب ويب سوفت

Question: 329 If Lucas calculates taxable income of $1,000,000 for Year 2 and is taxed at an effective income tax rate of 40%, how much income tax expense will be reported on Lucas’s income statement for Year 2?

A. $400,000

B. $402,000

Answer (B) is correct. Deferred tax expense or benefit is the net change during the year in the

entity’s deferred tax liabilities and assets. It is aggregated with the

current tax expense or benefit to determine total income tax expense for

the year. The amount of income taxes payable (current tax expense) is

$400,000 ($1,000,000 × 40%). The deferred tax assets increased by

$8,000 ($10,000 – $3,000 + $7,000 – $6,000) and the deferred tax

liabilities increased by $10,000 ($9,000 – $8,000 + $14,000 – $5,000).

Thus, Lucas’s income tax expense for Year 2 is $402,000 ($400,000

current tax expense – $8,000 increase in the deferred tax assets +

$10,000 increase in the deferred tax liabilities).

C. $404,000

D. $406,000

Fact Pattern: Lucas Company computed the following deferred tax balances for the 2 most recent years. Deferred tax assets are considered fully realizable.

Year 1 Year 2

Current deferred tax assets $3,000 $10,000

Noncurrent deferred tax assets 6,000 7,000

Current deferred tax liabilities 8,000 9,000

Noncurrent deferred tax liabilities 5,000 14,000

Question: 330 What deferred tax amounts will appear on Lucas’s statement of financial position at the end of Year 2?

Assets Liabilities

Current Noncurrent Current Noncurrent

A. $0 $1,000 $5,000 $0

B. $7,000 $1,000 $1,000 $9,000

C. $1,000 $0 $0 $7,000

Answer (C) is correct. Current deferred tax amounts are netted for financial reporting purposes.

Likewise, noncurrent amounts are also netted. At the end of Year 2,

Gleim 2015 | Part 1 | Online MCQs | Unit 003

www.facebook.com/CMA.Arabwebsoft Page 41

حصري | لمنتدى عرب ويب سوفت

Lucas nets its $10,000 of current deferred tax assets and $9,000 of

current deferred tax liabilities for a reported current deferred tax asset of

$1,000. Similarly, the $7,000 of noncurrent deferred tax assets and

$14,000 of noncurrent deferred tax liabilities are netted to produce a

reported noncurrent deferred tax liability of $7,000.

D. $10,000 $7,000 $9,000 $14,000

Question: 331 At the end of its first year in business, Pebbles Corporation reported pretax financial statement income of $50,000. Included in pretax income were $10,000 of revenue from installment sales and depreciation expense of $12,000. On the tax return, $5,000 of installment sales revenue was reported, and depreciation expense of $16,000 was deducted. The income tax rate was 40%. Pebbles reports installment sales receivables as current assets. On its year-end statement of financial position, Pebbles should report deferred tax balances of

A. $2,000 as a current liability and $1,600 as a current asset.

B. $4,000 as a current asset and $5,000 as a noncurrent asset.

C. $2,000 as a current liability and $1,600 as a noncurrent liability.

Answer (C) is correct. Temporary differences arise when the GAAP basis and the tax basis of

an item of income or expense differ. Of the installment sales, all $10,000

was recognized for financial reporting, but only $5,000 was recognized

for tax purposes, producing a temporary difference of $5,000. Since this

amount will be recognized later for tax purposes than for financial

reporting, it constitutes a deferred tax liability in the amount of $2,000

($5,000 × 40%). The depreciation expense will also result in a deferred

tax liability; since more expense was recognized for tax purposes than for

GAAP reporting ($16,000 – $12,000 = $4,000), a deferred tax liability of

$1,600 ($4,000 × 40%) results. A deferred tax asset or liability is

classified as current or noncurrent depending on the classification of the

related asset or liability. The installment revenue is thus properly

classified as current and, since depreciation expense relates to property,

plant, and equipment, it is classified as noncurrent.

D. $4,000 as a noncurrent liability and $5,000 as a current liability.

Question: 332 Moore Corporation’s income tax computations gave rise to the following accounts.

Deferred tax asset -- current $20,000

Deferred tax asset -- noncurrent 30,000

Deferred tax liability -- current 10,000

Deferred tax liability -- noncurrent 80,000

The account(s) relating to Moore’s taxes that should appear on the statement of financial position is (are)

Gleim 2015 | Part 1 | Online MCQs | Unit 003

www.facebook.com/CMA.Arabwebsoft Page 41

حصري | لمنتدى عرب ويب سوفت

A. A noncurrent deferred tax liability of $40,000.

B. A noncurrent deferred tax liability of $90,000 and a noncurrent deferred tax

asset of $50,000.

C. A current deferred tax asset of $10,000 and a noncurrent deferred tax

liability of $50,000.

Answer (C) is correct. Current deferred tax amounts are netted for financial reporting purposes.

Likewise, noncurrent amounts are also netted. At the end of the year,

Moore nets its $20,000 of current deferred tax assets and $10,000 of

current deferred tax liabilities for a reported current deferred tax asset of

$10,000. Similarly, the $30,000 of noncurrent deferred tax assets and

$80,000 of noncurrent deferred tax liabilities are netted to produce a

reported noncurrent deferred tax liability of $50,000.

D. A current deferred tax asset of $20,000, a noncurrent deferred tax asset of

$30,000, a current deferred tax liability of $10,000, and a noncurrent

deferred tax liability of $80,000.

Question: 333Intraperiod income tax allocation arises because

A. Items included in the determination of taxable income may be presented in different sections

of the financial statements.

Answer (A) is correct. To provide a fair presentation, GAAP require that income tax expense for the period be

allocated among continuing operations, discontinued operations, extraordinary items, other

comprehensive income, and items debited or credited directly to equity.

B. Income taxes must be allocated between current and future periods.

C. Certain revenues and expenses appear in the financial statements either before or after they

are included in taxable income.

D. Certain revenues and expenses appear in the financial statements but are excluded from

taxable income.

Question: 334 Income-tax-basis financial statements differ from those prepared under GAAP because they

A. Do not include nontaxable revenues and nondeductible expenses in

determining income.

B. Include detailed information about current and deferred income tax

liabilities.

C. Contain no disclosures about capital and operating lease transactions.

D. Recognize certain revenues and expenses in different reporting periods.

Answer (D) is correct. Financial statements prepared under the income tax basis of accounting

and financial statements prepared under GAAP differ when the tax basis

of an asset or a liability and its reported amount in the GAAP-based

Gleim 2015 | Part 1 | Online MCQs | Unit 003

www.facebook.com/CMA.Arabwebsoft Page 42

حصري | لمنتدى عرب ويب سوفت

financial statements are not the same. The result will be taxable or

deductible amounts in future years when the reported amount of the asset

is recovered or the liability is settled. Thus, certain revenues and

expenses are recognized in different periods. An example is subscriptions

revenue received in advance, which is recognized in taxable income

when received and recognized in financial income when earned in a later

period. Another example is a warranty liability, which is recognized as

an expense in financial income when a product is sold and recognized in

taxable income when the expenditures are made in a later period.

Question: 335 Temporary differences arise when expenses are deductible for tax purposes

After They Are Before They Are Recognized in Recognized in Financial Income Financial Income

A. No No

B. No Yes

C. Yes Yes

Answer (C) is correct. A temporary difference exists when (1) the reported amount of an asset

or liability in the financial statements differs from the tax basis of that

asset or liability, and (2) the difference will result in taxable or deductible

amounts in future years when the asset is recovered or the liability is

settled at its reported amount. A temporary difference may also exist

although it cannot be identified with a specific asset or liability

recognized for financial reporting purposes. Temporary differences most

commonly arise when either expenses or revenues are recognized for tax

purposes either earlier or later than in the determination of financial

income.

D. Yes No

Question: 336 When accounting for income taxes, a temporary difference occurs in which of the following scenarios?

A. An item is included in the calculation of net income but is neither taxable

nor deductible.

B. An item is included in the calculation of net income in one year and in

taxable income in a different year.

Answer (B) is correct. A temporary difference results when the GAAP basis and the tax basis of

an asset or liability differ. The effect is that a taxable or deductible

amount will occur in future years when the asset is recovered or the

liability is settled. But some temporary differences are not related to an

Gleim 2015 | Part 1 | Online MCQs | Unit 003

www.facebook.com/CMA.Arabwebsoft Page 43

حصري | لمنتدى عرب ويب سوفت

asset or liability for financial reporting. Thus, temporary differences

occur when revenues or gains, or expenses or losses, are used to calculate

net income under GAAP in a year before or after being used to calculate

taxable income.

C. An item is no longer taxable due to a change in the tax law.

D. The accrual method of accounting is used.

Question: 337 The best advantage of a zero-coupon bond to the issuer is that the

A. Bond requires a low issuance cost.

B. Bond requires no interest income calculation to the holder or issuer until

maturity.

C. Interest can be amortized annually by the APR method and need not be

shown as an interest expense to the issuer.

D. Interest can be amortized annually on a straight-line basis but is a noncash

outlay.

Answer (D) is correct. Zero-coupon bonds do not pay periodic interest. The bonds are sold at a

discount from their face value, and the investors do not receive interest

until the bonds mature. The issuer does not have to make annual cash

outlays for interest. However, the discount must be amortized annually

and reported as interest expense.

On January 1, Evangel Company issued 9% bonds in the face amount of $100,000, which Fact Pattern:

sulting in a bond discount of $3,793. mature in 5 years. The bonds were issued for $96,207 to yield 10%, re

Evangel uses the effective interest method of amortizing bond discount. Interest is payable annually on

December 31.

Question: 338 What is the amount of interest Evangel will pay at the end of the first year?

A. $8,659

B. $9,000

Answer (B) is correct. The annual cash payment is the face amount of the bonds times the stated

rate ($100,000 × 9% = $9,000).

C. $9,621

D. $10,000

Gleim 2015 | Part 1 | Online MCQs | Unit 003

www.facebook.com/CMA.Arabwebsoft Page 44

حصري | لمنتدى عرب ويب سوفت

Question: 339 A premium on bonds payable arises when

A. The semiannual bond interest becomes due.

B. The prevailing interest rate after the bond issuance falls below the nominal

rate of the bonds.

C. The amount received from sale of the bonds at issuance exceeds the face

value of the bonds.

Answer (C) is correct. A premium on bonds payable arises when the amount received from sale

of the bonds at issuance exceeds the face value of the bonds. This

situation occurs if, at the time the bonds are sold, their stated rate is

greater than the current market rate.

D. The cost of issuing the bonds is capitalized.

On January 1, Evangel Company issued 9% bonds in the face amount of $100,000, which Fact Pattern:

mature in 5 years. The bonds were issued for $96,207 to yield 10%, resulting in a bond discount of $3,793.

ive interest method of amortizing bond discount. Interest is payable annually on Evangel uses the effect

December 31.

Question: 340 What is the amount of interest expense that should be reported on Evangel’s income statement for the second year?

A. $8,779

B. $9,000

C. $9,559

D. $9,683

Answer (D) is correct. An amortization schedule for the first 2 years of Evangel’s bonds can be

prepared as follows:

Year

Beginning

Carrying

Amount

Times:

Effective

Rate

Equals:

Interest

Expense

Minus:

Cash

Paid

Equals:

Discount

Amortized

Ending

Carrying

Amount

1 $96,207 10% $9,621 $9,000 $621 $96,828

2 96,828 10% 9,683 9,000 683 97,510

On January 1, Evangel Company issued 9% bonds in the face amount of $100,000, which Fact Pattern:

ere issued for $96,207 to yield 10%, resulting in a bond discount of $3,793. mature in 5 years. The bonds w

Evangel uses the effective interest method of amortizing bond discount. Interest is payable annually on

December 31.

Gleim 2015 | Part 1 | Online MCQs | Unit 003

www.facebook.com/CMA.Arabwebsoft Page 45

حصري | لمنتدى عرب ويب سوفت

Question: 341 What is the amount of Evangel’s unamortized bond discount at the end of the first year?

A. $621

B. $2,452

C. $3,172

Answer (C) is correct. Total interest expense for the year equals the carrying amount of the

bonds times the effective rate (yield), or $9,621 ($96,207 × 10%).

Subtracting the cash interest payment from this leaves the amount of

discount amortized, or $621 ($9,621 – $9,000). Subtracting this amount

from the previous unamortized discount ($3,793) leaves a remaining

unamortized discount at the end of Year 1 of $3,172.

D. $3,793

On January 1, Evangel Company issued 9% bonds in the face amount of $100,000, which ct Pattern:Fa

mature in 5 years. The bonds were issued for $96,207 to yield 10%, resulting in a bond discount of $3,793.

zing bond discount. Interest is payable annually on Evangel uses the effective interest method of amorti

December 31.

Question: 342 The net carrying amount of Evangel’s bonds payable at the end of the first year is

A. $94,866

B. $95,586

C. $96,828

Answer (C) is correct. Total interest expense for the year equals the carrying amount of the

bonds times the effective rate (yield), or $9,621 ($96,207 × 10%).

Subtracting the cash interest payment from this leaves the amount of

discount amortized ($9,621 – $9,000 = $621). Subtracting this amount

from the previous unamortized discount ($3,793) leaves a remaining

unamortized discount at the end of Year 1 of $3,172. Subtracting this

amount from the face amount of the bonds ($100,000) provides a

carrying amount of $96,828.

D. $97,548

y 1, Evangel Company issued 9% bonds in the face amount of $100,000, which On Januar Fact Pattern:

mature in 5 years. The bonds were issued for $96,207 to yield 10%, resulting in a bond discount of $3,793.

terest is payable annually on Evangel uses the effective interest method of amortizing bond discount. In

December 31.

Question: 343 What is the amount of interest expense that should be reported on Evangel’s income statement at the end of the first year?

A. $8,659

B. $9,000

C. $9,621

Gleim 2015 | Part 1 | Online MCQs | Unit 003

www.facebook.com/CMA.Arabwebsoft Page 46

حصري | لمنتدى عرب ويب سوفت

Answer (C) is correct. Total interest expense for the year equals the carrying amount of the

bonds times the effective rate (yield), or $9,621 ($96,207 × 10%).

D. $10,000

Question: 344 On January 1, bonds with a face amount of $200,000, an 8% annual effective yield, and a 7% annual coupon rate were sold by Thomas Dynamics, Inc., for $180,000. The bonds pay interest on January 1 and July 1. Using the effective interest method, the company’s interest expense for the first 6 months ended July 1 will be

A. $7,000

B. $7,200

Answer (B) is correct. Total interest expense for the year equals the carrying amount of the

bonds times the effective rate (yield), or $14,400 ($180,000 × 8%). Half

of this amount is $7,200.

C. $14,000

D. $14,400

Question: 345 Debentures are

A. Income bonds that require interest payments only when earnings permit.

B. Subordinated debt and rank behind convertible bonds.

C. Bonds secured by the full faith and credit of the issuing firm.

Answer (C) is correct. Debentures are unsecured bonds. Although no assets are mortgaged as

security for the bonds, debentures are secured by the full faith and credit

of the issuing firm. Debentures are a general obligation of the borrower.

Only companies with the best credit ratings can issue debentures because

only the company’s credit rating and reputation secure the bonds.

D. A form of lease financing similar to equipment trust certificates.

Question: 346 Which one of the following characteristics distinguishes income bonds from other bonds?

A. The bondholder is guaranteed an income over the life of the security.

B. By promising a return to the bondholder, an income bond is junior to

preferred and common stock.

C. Income bonds are junior to subordinated debt but senior to preferred and

common stock.

D. Income bonds pay interest only if the issuing company has earned the

interest.

Answer (D) is correct. An income bond is one that pays interest only if the issuing company has

Gleim 2015 | Part 1 | Online MCQs | Unit 003

www.facebook.com/CMA.Arabwebsoft Page 47

حصري | لمنتدى عرب ويب سوفت

earned the interest, although the principal must still be paid on the due

date. Such bonds are riskier than normal bonds.

Question: 347 Serial bonds are attractive to investors because

A. All bonds in the issue mature on the same date.

B. The yield to maturity is the same for all bonds in the issue.

C. Investors can choose the maturity that suits their financial needs.

Answer (C) is correct. Serial bonds have staggered maturities; that is, they mature over a period

(series) of years. Thus, investors can choose the maturity date that meets

their investment needs. For example, an investor who will have a child

starting college in 16 years can choose bonds that mature in 16 years.

D. The coupon rate on these bonds is adjusted to the maturity date.

Question: 348 A construction company has signed $1,000,000 in new contracts. During the current year, 10% of the required work for these contracts was performed. Historically, the controller has recognized revenue when the contract work was completed using the completed contract method. This year, the company’s auditors are requiring the new contracts to be recognized under the percentage of completion method. The change in revenue recognition methods will result in a revenue change of

A. $0

B. $(900,000)

C. $100,000

Answer (C) is correct. When the outcome of a transaction involving the rendering of services

(e.g., a construction project) cannot be estimated reliably, revenue must

be recognized only to the extent of the expenses recognized that are

recoverable. If it is probable that the entity will recover the transaction

costs incurred, revenue is recognized only to the extent of those costs that

are expected to be recoverable. Thus, $100,000, 10% of the $1,000,000

contract, should be recognized as revenue compared with $0 of revenue

recognized under the completed contract method, since the contract is not

fully completed.

D. $1,000,000

Question: 349 Visor Co. maintains a defined benefit pension plan for its employees. The service cost component of Visor’s pension expense is measured using the

A. Unfunded accumulated benefit obligation.

B. Unfunded vested benefit obligation.

C. Projected benefit obligation.

Gleim 2015 | Part 1 | Online MCQs | Unit 003

www.facebook.com/CMA.Arabwebsoft Page 48

حصري | لمنتدى عرب ويب سوفت

Answer (C) is correct. Service cost is the actuarial present value of benefits attributed by the

pension benefit formula to services rendered during the accounting

period. It is a component of the projected benefit obligation (PBO). The

PBO as of a date is equal to the actuarial present value of all benefits

attributed by the pension benefit formula to employee service rendered

prior to that date. The PBO is measured using assumptions as to future

salary levels.

D. Expected return on plan assets.

Question: 350 Which of the following components must be included in the calculation of pension expense recognized for a period by an employer sponsoring a defined benefit pension plan?

Expected Return

Interest Cost on Plan Assets

A. Yes No

B. Yes Yes

Answer (B) is correct. The required minimum pension expense consists of the following

elements:

+ Service cost

+ Interest cost

– Expected return on plan assets

± Amortization of net gain or loss

± Amortization of prior service cost of credit

Pension expense

Thus, both interest cost and expected return on plan assets are

components of pension expense.

C. No Yes

D. No No

Question: 351 The following information pertains to Gali Co.’s defined benefit pension plan for Year 1:

Fair value of plan assets, beginning of year $350,000

Fair value of plan assets, end of year 525,000

Employer contributions 110,000

Benefits paid 85,000 In computing pension expense, what amount should Gali use as actual return on plan assets?

Gleim 2015 | Part 1 | Online MCQs | Unit 003

www.facebook.com/CMA.Arabwebsoft Page 49

حصري | لمنتدى عرب ويب سوفت

A. $65,000

B. $150,000

Answer (B) is correct. The actual return on plan assets is based on the fair value of plan assets at

the beginning and end of the accounting period adjusted for contributions

and payments during the period. The actual return for Gali is $150,000

($525,000 – $350,000 – $110,000 + $85,000).

C. $175,000

D. $260,000

Question: 352 Interest cost included in the pension expense recognized for a period by an employer sponsoring a defined benefit pension plan represents the

A. Shortage between the expected and actual return on plan assets.

B. Increase in the projected benefit obligation resulting from the passage of

time.

Answer (B) is correct. The interest cost component of pension expense is defined as the increase

in the PBO resulting from the passage of time.

C. Increase in the fair value of plan assets resulting from the passage of time.

D. Amortization of the discount on prior service cost.

Fact Pattern: Selected financial information for Jory Company for the current year ended December 31 is shown below.

Plan assets at January 1 $6,000,000

Projected benefit obligation at January 1 5,000,000

Accumulated benefit obligation at January 1 4,000,000

Interest cost 400,000

Service cost 700,000

Actual return on plan assets 500,000

Expected return on plan assets 500,000

Gleim 2015 | Part 1 | Online MCQs | Unit 003

www.facebook.com/CMA.Arabwebsoft Page 51

حصري | لمنتدى عرب ويب سوفت

Employer’s contribution 800,000

Benefits paid to retirees 300,000

Accrued pension cost at January 1 -0-

Question: 353 Jory’s net pension expense for the year ended December 31 is

A. $600,000

Answer (A) is correct. Jory’s net pension expense for the year ended December 31 can be

calculated as follows:

Current service cost $700,000

Interest cost 400,000

Expected return on plan assets (500,000)

Net periodic pension cost $600,000

B. $900,000

C. $1,600,000

D. $1,900,000

Fact Pattern: Selected financial information for Jory Company for the current year ended December 31 is shown below.

Plan assets at January 1 $6,000,000

Projected benefit obligation at January 1 5,000,000

Accumulated benefit obligation at January 1 4,000,000

Interest cost 400,000

Service cost 700,000

Actual return on plan assets 500,000

Gleim 2015 | Part 1 | Online MCQs | Unit 003

www.facebook.com/CMA.Arabwebsoft Page 51

حصري | لمنتدى عرب ويب سوفت

Expected return on plan assets 500,000

Employer’s contribution 800,000

Benefits paid to retirees 300,000

Accrued pension cost at January 1 -0-

Question: 354 The plan assets at December 31 for Jory should be valued at

A. $6,500,000

B. $6,800,000

C. $7,000,000

Answer (C) is correct. The fair value of Jory’s plan assets at December 31 can be calculated as

follows:

Fair value, January 1 $6,000,000

Add: Actual return 500,000

Add: Employer contribution 800,000

Less: Retirement benefits paid (300,000)

Fair value, December 31 $7,000,000

D. $7,300,000

Fact Pattern: Selected financial information for Jory Company for the current year ended December 31 is shown below.

Plan assets at January 1 $6,000,000

Projected benefit obligation at January 1 5,000,000

Accumulated benefit obligation at January 1 4,000,000

Interest cost 400,000

Gleim 2015 | Part 1 | Online MCQs | Unit 003

www.facebook.com/CMA.Arabwebsoft Page 52

حصري | لمنتدى عرب ويب سوفت

Service cost 700,000

Actual return on plan assets 500,000

Expected return on plan assets 500,000

Employer’s contribution 800,000

Benefits paid to retirees 300,000

Accrued pension cost at January 1 -0-

Question: 355 Jory’s projected benefit obligation at December 31 is

A. $5,400,000

B. $5,700,000

C. $5,800,000

Answer (C) is correct. Jory’s projected benefit obligation at December 31 can be calculated as

follows:

PBO, January 1 $5,000,000

Add: Current service cost 700,000

Add: Interest cost 400,000

Less: Retirement benefits paid (300,000)

PBO, December 31 $5,800,000

D. $6,100,000

Fact Pattern: Brown Industries operates a defined benefit pension plan. Information received from the actuary and the trustee related to the Year 2 pension plan includes the following:

Projected benefit obligation, January 1, Year 2 $1,889,000

Service cost 105,000

Interest cost 190,000

Gleim 2015 | Part 1 | Online MCQs | Unit 003

www.facebook.com/CMA.Arabwebsoft Page 53

حصري | لمنتدى عرب ويب سوفت

Retirement benefits paid 182,000

Employer contribution 155,000

Actual return on plan assets 215,000

Amortization of prior service cost 122,000

Amortization of prior-year net pension loss 37,000

Fair value -- pension plan assets, December 31, Year 1 1,825,000

Question: 356 Brown’s Year 2 net pension cost is

A. $190,000

B. $239,000

Answer (B) is correct. Assuming that the actual return on plan assets is equal to the expected

return, the calculations are:

Current service cost $105,000

Interest cost 190,000

Actual return on plan assets (215,000)

Amortization of prior service cost 122,000

Amortization of prior-year net pension loss 37,000

Net periodic pension cost $239,000

C. $454,000

D. $299,000

Fact Pattern: Brown Industries operates a defined benefit pension plan. Information received from the actuary and the trustee related to the Year 2 pension plan includes the following:

Projected benefit obligation, January 1, Year 2 $1,889,000

Service cost 105,000

Interest cost 190,000

Retirement benefits paid 182,000

Employer contribution 155,000

Actual return on plan assets 215,000

Gleim 2015 | Part 1 | Online MCQs | Unit 003

www.facebook.com/CMA.Arabwebsoft Page 54

حصري | لمنتدى عرب ويب سوفت

Amortization of prior service cost 122,000

Amortization of prior-year net pension loss 37,000

Fair value -- pension plan assets, December 31, Year 1 1,825,000

Question: 357 The fair value of Brown’s plan assets at December 31, Year 2, is

A. $1,790,000

B. $1,798,000

C. $2,005,000

D. $2,013,000

Answer (D) is correct. The calculations are:

Fair value, Dec. 31, Year 1 $1,825,000

Add: Actual return 215,000

Add: Employer contribution 155,000

Less: Retirement benefits paid (182,000)

Fair value, Dec. 31, Year 2 $2,013,000

Fact Pattern: Brown Industries operates a defined benefit pension plan. Information received from the actuary and the trustee related to the Year 2 pension plan includes the following:

Projected benefit obligation, January 1, Year 2 $1,889,000

Service cost 105,000

Interest cost 190,000

Retirement benefits paid 182,000

Employer contribution 155,000

Actual return on plan assets 215,000

Amortization of prior service cost 122,000

Amortization of prior-year net pension loss 37,000

Fair value -- pension plan assets, December 31, Year 1 1,825,000

Gleim 2015 | Part 1 | Online MCQs | Unit 003

www.facebook.com/CMA.Arabwebsoft Page 55

حصري | لمنتدى عرب ويب سوفت

Question: 358 Brown’s projected benefit obligation at December 31, Year 2, is

A. $1,787,000

B. $1,969,000

C. $2,002,000

Answer (C) is correct. The calculations are:

PBO, Jan. 1, Year 2 $1,889,000

Add: Current service cost 105,000

Add: Interest cost 190,000

Less: Retirement benefits paid (182,000)

PBO, Dec. 31, Year 2 $2,002,000

D. $2,029,000